Você está na página 1de 90

PEOPLE OF THE PHILIPPINES, petitioner, vs.HERMOGENES MARIANO and HON.

AMBROSIO M. GERALDEZ, in his capacity as Presiding Judge of the Court of


First Instance of Bulacan, Branch V
G.R. No. L-40527 June 30, 1976

Facts:
Hermogenes Mariano was charged with estafa before the CFI of Bulacan. Being then
appointed as Liaison Officer by Mayor Nolasco, he was authorized to receive and be
receipted for US excess property of USAID/NEC for the use and benefit of said
municipality, electric cables and cable power with a total value of $717.50 or P4,797.35.
Once in possession of the said items and far from complying with his obligation to deliver
it to the mayor, and in spite of repeated demands, he misappropriated, misapplied and
converted the said items for his own personal use and benefit.

He filed a motion to quash on the ground that the CFI has no jurisdiction of the offense
charged or of the person of the defendant, among others. He claimed that the items which
were the subject matter of the Information against him were the same items for which
Mayor Nolasco of San Jose del Monte, Bulacan, was indicted before a Military
Commission under a charge of malversation of public property, and for which Mayor
Nolasco had been found guilty, and that inasmuch as the case against Mayor Nolasco
had already been decided by the Military Tribunal, the Court of First Instance of Bulacan
had lost jurisdiction over the case against him.

Respondent Judge issued an Order granting the motion to quash on the ground of lack
of jurisdiction, considering that the Military Commission had already taken cognizance of
the malversation case against Mayor Nolasco involving the same subject matter in its
concurrent jurisdiction with CFI.

The people now seeks a review of the aforesaid Order.

Issue:

Whether or not civil courts and military commissions exercise concurrent jurisdiction over
the offense of estafa of goods valued at not more than six thousand pesos and allegedly
committed by a civilian.

Ruling:
----------------------------------------------------------------------------------------------------------------
"Jurisdiction" is the basic foundation of judicial proceedings. The word "jurisdiction" is
derived from two Latin words "juris" and "dico" — "I speak by the law" — which means
fundamentally the power or capacity given by the law to a court or tribunal to entertain,
hear, and determine certain controversies.

"Jurisdiction is the right of a Judge to pronounce a sentence of the law in a case or issue
before him, acquired through due process of law;" it is "the authority by which judicial
officers take cognizance of and decide cases."
"Jurisdiction" is the authority to hear and determine a cause the right to act in a case.
"Jurisdiction" has also been aptly described as the right to put the wheels of justice in
notion and to proceed to the final determination of a cause upon the pleadings and
evidence.

"Criminal Jurisdiction" is necessarily the authority to hear and try a particular offense and
impose the punishment for it.
----------------------------------------------------------------------------------------------------------------

The Judiciary Act of 1948 where in its Section 44 (f) provided that Courts of First Instance
shall have original jurisdiction in all criminal cases in which the penalty provided by law is
imprisonment for more than six months,or a fine of more than two hundred pesos.

The offense of estafa charged against respondent Mariano is penalized with


imprisonment from four (4) months and one (1) day to two (2) years and four (4) months.
By reason of the penalty imposed which exceeds six (6) months imprisonment, the
offense alleged to have been committed by the accused, Mariano, falls under the original
jurisdiction of courts of first instance.

The SC reiterated the settled rule that the jurisdiction of a court is determined by the
statute in force at the time of the commencement of the action. In the case at bar, the law
in force vesting jurisdiction upon said court was the Judiciary Act of 1948. The Military
Commission is not vested with jurisdiction over the crime of estafa.

Respondent court therefore gravely erred when it ruled that it lost jurisdiction over the
estafa case against respondent Mariano with the filing of the malversation charge against
Mayor Nolasco before the Military Commission. Estafa falls within the sole exclusive
jurisdiction of civil courts.

PREMISES CONSIDERED, the appealed Order is set aside and respondent Judge is
directed to proceed with the trial of Criminal Case No. SM- 649 without further delay.

US Vs JUEVES
(G.R. No. 6992, August 30, 1912)

FACTS: Seven persons were charged with the crime of brigandage or highway
robbery.
1. A band of armed men entered into a municipal building, where they abducted the
presidente, in the town of Alabat, Tayabas Province. They also killed the justice of
peace of that town.
2. A band of some twenty men, entered the house of one Doroteo
Maraver, situated in the sitio of Capalohan (formerly a part of the town of
Capalongan, Ambos Camarines, but at the date of the commencement of
this action a part of the town of Calauag, Tayabas), wherein they tied and
bound four men therein. They ate first before they, together with the prisoners,
went out to the house of Francis Ambas (one of the prisoners) and took several of
his things. They then proceeded to a river, where they liberated two of their
prisoners. About a year later the widow of Martin Ambas attempted to
identify two skulls exhibited to her in the Court of First Instance at Daet,
Ambos Camarines, as being those of the two missing men.
3. A band of armed men, went to the barrio of Basiad in the town of
Capalongan, Ambos Camarines, and took as prisoners three men (Juan
Talento escaped and reported the crime to the authorities), conducting them from
place to place.
4. Two of the accused, Agustin Jueves and Felix Jueves, accompanied by their
younger brother, Esteban Jueves, all armed, entered the house of Serapio
Juego, situated near the sea in the sitio of Pangas, municipality of Calauag,
Tayabas. The two first named placed themselves in the doorway while the
last named took possession of a small quantity of rice. All of them left
afterwards.
Counsel of defendants insisted that the court of Tayabas had no jurisdiction to try
these appellants for the reason that the territory where the acts complained of
were committed in the Province of Ambos Camarines, although it has since been
transferred to the Province of Tayabas. The record showing nothing to the contrary,
we assume that the complaint was filed subsequent to the transfer of territory in
question.
Issue: Whether or not the court have jurisdiction of crimes committed in a particular
locality prior to the time such locality was included within the jurisdiction of
such court
Held: The general rule is that the dispose of a court is determined (1) by the
geographical limits of the territory over which it presides, and (2) the actions (civil
and criminal) it is empowered to hear and decide.
Questions of jurisdiction do not rise and cannot be decided until the initial pleadings
in an action presented to a court. A court has an inchoate right of jurisdiction over
all crimes committed within its jurisdiction which is perfected on the institution of
the action. If, however, it loses jurisdiction over a particular action because its
territorial limits are restricted prior to the institution of the action, it also loses this
inchoate right to jurisdiction in favor, of the court to which the territory is transferred.
Were the rule otherwise, it would be necessary to prolong a court's existence
indefinitely after being legally abolished or after its authority had been legally
restricted or diminished on the ground that it must hear possible actions arising
sometime in the future.
The territory where the acts complained of in the case at bar were committed having
been transferred to the Province of Tayabas prior to the institution of this action,
the court of that province had jurisdiction to hear and determine this case.
T he change of the territory after the crime was committed and before the institution
of this action does no touch the offense nor change the punishment therefor.
It only includes the place of the commission of the offense within another judicial
district, and subjects the appellants to trial in that district.
THE MANILA RAILROAD COMPANY vs. THE ATTORNEY-GENERAL
(G.R. No. L-6287, December 1, 1911)

Facts:
The plaintiff began an action in the Court of First Instance of the Province of Tarlac for
the condemnation of certain real estate, stated by the plaintiff in his complaint that such
property is located in the Province of Tarlac. It is alleged in the complaint that the
plaintiff is authorized by law to construct a railroad line "from Paniqui to Tayug in the
Province of Tarlac," and it is for the purpose of condemning lands for the construction of
such line that this action is brought. The defendants in one action are all of the different
owners of or persons otherwise interested in the 69,910 square meters of land to be
condemned. After filing and duly serving the complaint the plaintiff, pursuant to law and
pending final determination of the action, took possession of and occupied the lands
described in the complaint. Subsequently, plaintiff gave notice to the defendants that a
motion would be made to the court to dismiss the action upon the ground that the court
had no jurisdiction of the subject matter, it having just been ascertained by the plaintiff
that the land sought to be condemned was situated in the Province of Nueva Ecija,
instead of the Province of Tarlac, as alleged in the complaint. The trial court dismissed
the action upon the ground presented by the plaintiff. This appeal is taken from said
judgment of dismissal.

Issue: WON the Court of First Instance of the province of Tarlac has no jurisdiction for
an action in condemnation of real estate located in Nueva Ecija.

Ruling:
No, the Court of First Instance of the province of Tarlac has jurisdiction. The contention
of the plaintiff lies under Section 377 of the Code of the Civil Procedure which is a
procedural regarding the place and method of trial. It touched not the authority of the
court over the land but, rather, the powers which it may exercise over the parties. They
relate not to the jurisdictional power of the court over the subject matter but to the place
where that jurisdiction is to be exercised. In other words, the jurisdiction is assured,
whatever the place of its exercise. The jurisdiction is the thing; the place of exercise
its incident.

The Court reiterated that Sections 55 and 56 of Act No. 136 of the Philippine
Commission confer jurisdiction upon the Courts of First Instance of the Philippine
Islands with respect to real estate which provides that all civil actions which involve the
title to or possession of real property, or any interest therein, or the legality of any tax,
impost, or assessment, except actions of forcible entry into, and detainer of lands or
buildings, original jurisdiction of which is by this Act conferred upon courts of justice of
the peace.

It is the intention of the Philippine Commission to give to the Courts of First Instance the
most perfect and complete jurisdiction possible over the subject matters mentioned in
connection therewith. Such jurisdiction is not made to depend upon locality. There is no
suggestion of limitation.

The fact that section 377 is not such Act, that it is found in code of Procedure rather
than in the substantive law, that it deals with the relative procedural rights of parties
rather than the power of the court, that it relates to the place rather than to the thing,
that it composes the whole of a chapter headed simply "Venue," lead us to hold that the
Court of First Instance of Tarlac had full jurisdiction of the subject matter of this action at
the time when it was dismissed.

The Court, therefore, holds that the terms of section 377 providing that actions affecting
real property shall be brought in the province where the land involved in the suit, or
some part thereof, is located, do not affect the jurisdiction of Courts of First Instance
over the land itself but relate simply to the personal rights of parties as to the place of
trial.

Case is remanded to the trial court.

SEC. 377 of the Code of Civil Procedure states that actions to confirm title to real
estate, or to secure a partition of real estate, or to cancel clouds, or remove doubts from
the title to real estate, or to obtain possession of real estate, or to recover damages for
injuries to real estate, or to establish any interest, right, or title in or to real estate, or
actions for the condemnation of real estate for public use, shall be brought in the
province were the lands, or some part thereof, is situated; actions against executors,
administrators, and guardians touching the performance of their official duties, and
actions for account and settlement by them, and actions for the distribution of the
estates of deceased persons among the heirs and distributes, and actions for the
payment of legacies, shall be brought in the province in which the will was admitted to
probate, or letters of administration were granted, or the guardian was appointed. And
all actions not herein otherwise provided for may be brought in any province where the
defendant or any necessary party defendant may reside or be found, or in any province
where the plaintiff, except in cases were other special provision is made in this Code. In
case neither the plaintiff nor the defendant resides within the Philippine Islands and the
action is brought to seize or obtain title to property of the defendant within the Philippine
Islands and the action is brought to seize or obtain title to property of the defendant
within the Philippine Islands, the action shall be brought in the province where the
property which the plaintiff seeks to seize or to obtain title to is situated or is
found: Provided, that in an action for the foreclosure of a mortgage upon real estate,
when the service upon the defendant is not personal, but is by publication, in
accordance with law, the action must be brought in the province where the land lies.
And in all cases process may issue from the court in which an action or special
proceeding is pending, to be enforced in any province to bring in defendants and to
enforce all orders and decrees of the court. The failure of a defendant to object t the
venue of the action at the time of entering his appearance in the action shall be deemed
a waiver on his part of all objection to the place or tribunal in which the action is brought,
except in the actions referred to in the first sixteen lines of this section relating to real
estate, and actions against executors, administrators, and guardians, and for the
distribution of estates and payment of legacies.

Reyes v Diaz
Facts: Art VIII, Sec 2 No. 3 of the Constitution confers upon the SC any question where
jurisdiction of the trial court is in issue. In this case, Diaz accuses Reyes of not duly
filing his certificate of candidacy. The case was elevated to the Supreme Court for
decision.
Issue: Does the RTC have jurisdiction over the case at hand?
Held: It depends on the filing of the candidacy. The Supreme Court ruled that if Reyes in
fact did not file his candidacy, then the RTC has jurisdiction to try the case. However, if
the same is not proven, the RTC has no jurisdiction but to dismiss the case.
It was explained that jurisdiction over the subject-matter is the power to hear and
determine the general class to which the proceeding in question
belongs. Therefore, it is not determined until the question has been settled.
G.R. No. 71855 January 20, 1988
RIZALITO VELUNTA, petitioner,
vs.
THE CHIEF, PHILIPPINE CONSTABULARY AND COLONEL SIMEON KEMPIS JR.,
PRESIDENT GCM, Recom. VIII, Palo, Leyte, respondents.
On April 16, 1982 at about 6:00 o'clock in the evening, while directing traffic at the
intersection of Burgos-Tarcela-Lucente Streets, Tacloban City, the petitioner, a
patrolman apprehended Romeo Lozano, a motorized tricycle driver, for violations of
traffic rules and regulations. An altercation occurred between them which resulted in the
shooting and death of Romeo Lozano.
On October 30,1982, Mrs. Anacorita Lozano, widow of the victim, filed an administrative
complaint against the petitioner with the NAPOLCOM, Region VIII, Tacloban City for
grave misconduct and rendered a decision dated August 9,1984 finding the petitioner
guilty of grave misconduct and meted the penalty of "Dismissal from the Service." Upon
motion, it is modified finding the petitioner guilty only of Less Grave Misconduct and
suspension from service for six months without pay.
During the pendency of the administrative case, Mrs. Lozano also filed a complaint for
homicide with the City Fiscal's Office of Tacloban which issued a resolution finding the
existence of prima facie evidence that the petitioner is "with deliberate intent and with
intent to kill," shot with his service pistol one Romeo Lozano.
The case was referred to the military authorities pursuant to P.D. 1850 which authorizes
the Chief of the Philippine Constabulary to convene court martials to try, hear, and
decide cases for criminal acts committed by members of the Integrated National Police.
Petitioner challenges the assumption of jurisdiction by the General Court Martial over
the criminal case for homicide against him. He contends that the General Court Martial
has no more jurisdiction to continue the hearing against him as a result of the provisions
of Executive Order No. 1040, in relation to Executive Order No. 1012, which became
effective last July 10, 1985 whereby supervision and control over all units and members
of the Integrated National Police have been transferred to NAPOLCOM and placed
directly under the Office of the President of the Philippines, thereby removing police
officers from the supervision and control of the Chief of the Philippine
Issue: Whether or not the GCM still has jurisdiction to her the case against the
petitioner.
Ruling: Yes. Jurisdiction is the power with which courts are invested for administering
justice, that is, for hearing and deciding cases. As early as 1914, it was declared that
the courts of the Philippine Islands have no common law jurisdiction or power, but only
those expressly conferred by the Constitution and statutes and those necessarily
implied to make the express powers effective.
In the instant case, P.D. No, 1850 which vests jurisdiction on courts martial over
criminal cases involving the members of the Integrated National Police and the term
uniformed members of the Integrated National Police used in Sec. 1 shall refer to police
officers, policemen, firemen and jail guards.
Under Executive Order No. 1012 it is only the "operational supervision and direction"
over all units of the Integrated National Police force stationed or assigned in the
different cities and municipalities that was transferred from the Philippine Constabulary
to the city or municipal government concerned. Likewise, under Executive Order No.
1040 it is the exercise of "administrative control and supervision" over all units of the
Integrated National Police forces throughout the country that was transferred to the
President of the Philippines.
It is specifically stated under Executive Order No. 101 2 that it is only the "operational
supervision and direction" over all units of the Integrated National Police force stationed
or assigned in the different cities and municipalities that was transferred from the
Philippine Constabulary to the city or municipal government concerned. Likewise, under
Executive Order No. 1040 it is the exercise of "administrative control and supervision"
over all units of the Integrated National Police forces throughout the country that was
transferred to the President of the Philippines.
The distinction between operational supervision and direction over the Integrated
National Police and jurisdiction or authority of a court-martial to hear, try and decide a
criminal proceeding against a police officer so that the appropriate penalty for the
commission of a crime or offense may be imposed is easily discernible. One refers to
how the police will perform their functions and who shall direct such performance while
the other refers to the tribunals vested with power to try criminal cases against them.
The allegation of the petitioner that P.D. 1850 has been expressly repealed by the clear
and precise provision of Section 3 of Executive Order No. 1040 is inaccurate, Section 3
of the executive order provides:
All laws, decrees, executive orders, rules and regulations and other enactments, or
parts thereof, inconsistent with the provisions of tills Executive Order are hereby
repealed, amended and modified accordingly.
The aforecited provision does not repeal in express terms, P.D. No. 1850. Neither is
there any inconsistency between P.D. No. 1850, which confers upon courts-martial,
jurisdiction over crimes and offenses involving members of the Integrated National
Police, and Executive Order No. 1040 which gives the city and municipal governments,
operational supervision and direction over members of the Integrated National Police.
Repeals by implication are not favored and will not be so declared unless the intent of
the legislators is manifest.
When the case was filed in 1982, there can be no question that the respondent General
Court Martial had jurisdiction. Since jurisdiction had properly been exercised from the
start, it remains with the military court martial unless a law expressly divests it of that
jurisdiction. It is an established rule that jurisdiction once acquired remains until validly
transferred by the proper authority according to law.
The provision of the Constitution, Article XVI, Section 6, on the State maintaining a
police force national and civilian in character is still in the process of being implemented.
Police forces continue to remain part of the PC-INP until the civilian police force is finally
set-up as contemplated by the fundamental law. (Barcellano v. Major General Renato
de Villa, et al., G.R. No. 75952, October 20,1987)
Petition is DISMISSED for lack of merit.
UY V SANDIGANBAYAN
(G.R. Nos. 105965-70, August 09, 1999)
FACTS:

George Uy have six (6) informations charging him with violation of Section 3 (e), R.A.
No. 3019 (Anti-Graft and Corrupt Practices Act). Sandiganbayan filed a resolution
denying Uy’s motion to quash the informations. In this petition, he seeks to annul and
set aside the said resolution.
Uy was Deputy Comptroller of the Philippine Navy. He was designated by his immediate
supervisor, Assistant Chief of Naval Staff for Comptrollership, to act on the latter's
behalf, during his absence, on matters relating to the activities of the Fiscal Control
Branch, O/NG. This included the authority to sign disbursement vouchers relative to the
procurement of equipment needed by the Philippine Navy.
July 2, 1991: six informations for estafa through falsification of official documents and
one (1) information for violation of Section 3 (e), R.A. No. 3019 were filed with the
Sandiganbayan against Uy and 19 co-accused.
September 20, 1991: the Sandiganbayan issued an Order directing a comprehensive
re-investigation of the cases against all the twenty (20) accused.
After conducting the re-investigation, the Special Prosecutor issued an Order dated
November 14, 1991 recommending that the informations for estafa through falsification
of official documents be withdrawn and in lieu thereof, informations for violation of
Section 3 (e) of R.A. No. 3019 be filed against eleven (11) accused, which included Uy.
The number of those charged under RA 3019 reduced to 5, including Uy.
The number of accused reduced to 3 upon filing of motions for reconsideration, but still
including Uy.
The amended informations filed by the special prosecutor states that in 1985, Metro
Manila, Uy and 2 other public officials, committed the offense through evident bad faith
or gross inexcusable negligence in relation to their office, causing undue injury to the
Government.
The 3 accused prepared and signed documents for purchasing seal rings-- which
documents said accused had the duty to check/verify/examine, thereby `acting or
omitting to act' in a situation where there is a duty to act. However, only 100 seal rings
were received at a unit price of P98.70, yet 1,000 pieces appear to have been sold with
total price of P98,700.00, hence there was gross error in multiplication as shown on the
face of the aforesaid PO and other supporting documents, resulting to an overpayment
of P88,930.00 thereby depriving the Government/Philippine Navy of the use thereof
until its remittance/return to the Government/Philippine Navy in December, 1991.
Uy filed (with the Sandiganbayan) a motion to quash the informations on the grounds
that:
The Sandiganbayan has no jurisdiction over the offense charged or the person of the
accused.
The officer who has filed the informations had no authority to do so.
The facts charged do not constitute an offense.
More than one (1) offense is charged.
Sandiganbayan: denied Uy’s motion to quash for lack of merit.
Uy: it is the court- martial, not the Sandiganbayan, which has jurisdiction to try him. He
emphasized the fundamental doctrine that the jurisdiction of a court is determined by
the statute in force at the time of the commencement of the action. They claim that at
the time the amended informations were filed, the controlling law on the jurisdiction over
members of the AFP is P.D. 1850, "Providing for the trial by courts-martial of members
of the Integrated National Police and further defining the jurisdiction of courts- martial
over members of the AFP"
ISSUE/S:
W/N the Sandiganbayan has jurisdiction over the subject criminal cases or the person of
the petitioner;
W/N the respondents Ombudsman and Special Prosecutor have the authority to file the
questioned amended information;

HELD:

The Sandiganbayan has NO jurisdiction over Uy, at the time of the filing of the
informations, and as now prescribed by law. The Sandiganbayan Law creating the
Sandiganbayan provides for the scope of its jurisdiction. In cases of violation of RA
3019, the Sandiganbayan has exclusive original jurisdiction over “Philippine army and
air force colonels, naval captains, and all officers of higher rank”. In the case at bar, Uy
is not included in the scope of Sandiganbayan’s jurisdiction. He is a rank lower than
“naval captains and all officer of higher rank”. Therefore, his rank falls within the
jurisdiction of the regular courts which is the RTC because the punishment for the
offense committed is not less than 6 years and 1 month or 15 years.
It is the prosecutor, not the Ombudsman, who has the authority to file the corresponding
information/s against Uy in the RTC. The Ombudsman exercises prosecutorial powers
only in cases cognizable by the Sandiganbayan.
PILIPINAS SHELL PETROLEUM CORP. and PETRON CORP. VS ROMARS
INTERNATIONAL
G.R. 189669 February 16, 2015
FACTS:
Petitioner received information that respondent was selling and distributing LPG by
illegally refilling the steel cylinders these companies manufactured and bearing their
registered trademark and device by Petron. Investigators were sent to probe on the
activities and the refilled cylinders were brought back and through this it was verified
that respondent was not authorized to distribute and/or sell, deal with Petrol LPG
products nor use any of their trademarks. The petitioners then requested the NBI to
investigate the activities to apprehend and prosecute establishment who conduct illegal
refilling, distributing, and/or sale of LPG products. On behalf of Petron and Shell, the
NBI filed with the Naga RTC two separate applications for search warrant and violation
of the Intellectual Property Code.
It was only when a new counsel for respondents that they raised for the first time the
issue of the impropriety of the filing of the Search Warrant when the alleged crime was
committed within the Iriga City RTC jurisdiction.
ISSUE:
Was the issuance of the search warrants within the jurisdiction of RTC Naga when the
crime was committed beyond its territorial jurisdiction?
RULING:
YES. An application for a search warrant is not a criminal action. Proceedings for said
applications are not criminal in nature. Hence, the rule that venue is jurisdictional does
not apply thereto. Evidently, the issue of whether the application should have been filed
in RTC-Iriga City or Naga RTC is not one involving jurisdiction because the power to
issue criminal processes is inherent in all courts.
The concept of venue in criminal cases is jurisdictional. The place where the crime was
committed determines not only the venue of the action but an essential element of
jurisdiction. It is a fundamental rule that for jurisdiction to be acquired by courts, the
offense should have been committed or any of its essential ingredients should have
taken place within the territorial jurisdiction of the court.
However, in a jurisprudence, the SC ruled that an application for a search warrant is a
“special criminal process” rather than a criminal action. It is a basic flaw to erroneously
equate it with the institution and prosecution of a criminal action in a trial court. The
requisites, procedure and purposes for the issuance of a search warrant are completely
different from the institution of a criminal action. In special criminal process, the power
to issue is inherent in all courts as compared to criminal action, jurisdiction over which is
reposed to specific courts of indicated competence. A search warrant is merely a
process, generally issued by the court in the exercise of its ancillary jurisdiction, and not
a criminal action to be entertained by the court pursuant to its original jurisdiction.

People vs Alejo Taroy


G.R No. 192466 September 7, 2011
FACTS: Mila married Taroy in 1997 upon the death of her first husband. Des, aged 10,
was the eldest daughter of Mila by her first marriage. Mila’s children lived with her and
her second husband in Pucsusan Barangay, Itogon, Benguet, at the boundary of Baguio
City. According to Des, on August 10, 1997 she was alone in the house doing some
cleaning since her mother was at work and her two siblings were playing outside. When
Taroy entered the house, he locked the door, closed the windows, removed his clothes,
and ordered Des to remove hers. When Des resisted, Taroy poked a knife at her head
and forced her to submit to his bestial or inhuman desires. Afterwards, Taroy warned
her not to tell anyone about it, otherwise Mila and her siblings will suffer harm. Taroy
sexually abused Des again in September 1998. 4 years later or on November 1, 2002,
when Des was 15, she told her aunt and Mila about what had happened between Taroy
and her. They accompanied Des to the NBI to complain. Mila and a certain Alumno
testified that they later accompanied Des to the hospital for examination. The doctor
who examined Des testified that Des had two narrow notches in her hymen at three
o’clock and five o’clock positions. The Doctor explained that these notches or V-shaped
or sharp indentions over the hymenal edges suggested a history of previous blunt force
or trauma possibly caused by the insertion of an erect male penis. Taroy denied raping
Des. He asserted that the testimony was made only because of Des’ aunt disliked him.
The public prosecutor charged Taroy with two counts of rape before the Regional Trial
Court of La Trinidad, Benguet.
Benguet RTC: Found Taroy guilty of two counts of rape and sentenced him to suffer the
penalty of reclusion perpetua. It also ordered him to pay Des for each count:
₱75,000.00 as civil indemnity, ₱75,000.00 as moral damages, and ₱25,000.00 as
exemplary damages.
Taroy challenged the Benguet RTC’s jurisdiction over the crimes charged, he testified
that their residence when the alleged offenses took place was in Pucsusan Barangay,
Baguio City. The RTC held, however, that Taroy’s testimony about his residence did not
strip the court of its jurisdiction since he waived the jurisdictional requirement.
CA: Court of Appeals affirmed the decision of the RTC. It held that the prosecution has
sufficiently established the jurisdiction of the RTC through the testimony of Mila, Des,
and Alumno.
ISSUE: Whether or not the RTC of La Trinidad, Benguet, has jurisdiction to hear and
decide the cases of rape against Taroy.
RULING: Yes. The information filed with the RTC of La Trinidad states that the crimes
were committed in the victim and the offender’s house in City Limit, Tuding, Municipality
of Itogon, Province of Benguet. This allegation conferred territorial jurisdiction over the
subject offenses on the RTC of La Trinidad, Benguet. Taroy’s unsupported assertion
that the subject offenses took place in Baguio City is not entitled to belief. Besides,
Taroy admitted during the pre-trial in the case that it was the RTC of La Trinidad that
had jurisdiction to hear the case. Taken altogether, that RTC’s jurisdiction to hear the
case is outside dispute. Under the law, venue is jurisdictional in criminal cases. It can
neither be waived nor subjected to stipulation. The right venue must exist as a matter of
law. Thus, for territorial jurisdiction to attach, the criminal action must be instituted and
tried in the proper court of the municipality, city, or province where the offense was
committed or where any of its essential ingredients took place.
Court dismisses the appeal and affirms the Court of Appeals decision dated January 19,
2010 with the modification that the award of exemplary damages be increased from
₱25,000.00 to ₱30,000.00.

HECTOR TRENAS vs PEOPLE OF THE PHILIPPINES


G.R. No. 195002, January 25, 2012

FACTS: Sometime in December 1999, Margarita Alocilja (Margarita) wanted to buy a


house-and-lot in Iloilo City covered by TCT No. 109266. It was then mortgaged with
Maybank. The bank manager Joselito Palma recommended the appellant Hector
Treñas (Hector) to private complainant Elizabeth, who was an employee and niece of
Margarita, for advice regarding the transfer of the title in the latter’s name. Hector
informed Elizabeth that for the titling of the property in the name of her aunt Margarita,
the following expenses would be incurred: P20,000.00- Attorney’s fees, P90,000.00-
Capital Gains Tax, P24,000.00- Documentary Stamp, P10,000.00- Miscellaneous
Expenses. Thereafter, Elizabeth gave P150,000.00 to Hector who issued a
corresponding receipt dated December 22, 1999 and prepared a Deed of Sale with
Assumption of Mortgage. Subsequently, Hector gave Elizabeth Revenue Official
Receipt Nos. 00084370 for P96,000.00 and 00084369 for P24,000.00. However, when
she consulted with the BIR, she was informed that the receipts were fake. When
confronted, Hector admitted to her that the receipts were fake and that he used the
P120,000.00 for his other transactions. Elizabeth demanded the return of the money. To
settle his accounts, appellant Hector issued in favor of Elizabeth a Bank of Commerce
check No. 0042856 dated November 10, 2000 in the amount of P120,000.00, deducting
from P150,000.00 the P30,000.00 as attorney’s fees. When the check was deposited
with the PCI Bank, Makati Branch, the same was dishonored for the reason that the
account was closed. Notwithstanding repeated formal and verbal demands, appellant
failed to pay. Thus, the instant case of Estafa was filed against him. On 29 October
2001, an Information was filed by the Office of the City Prosecutor before the Regional
Trial Court, both of Makati City. During arraignment on 26 April 2002, petitioner, acting
as his own counsel, entered a plea of "Not Guilty." Allegedly due to old age and poor
health, and the fact that he lives in Iloilo City, petitioner was unable to attend the pre-
trial and trial of the case. The RTC rendered a decision finding the accused guilty for
estafa under section 1 paragraph b of Article 315 of the RPC. The RTC denied the
motion for reconsideration filed by the petitioner. The Court of Appeals affirmed the
decision of the RTC also denying its motion for reconsideration. Hence the petition.

Petitioner, in his contention, states the case should not be tried in Makati since nowhere
in the evidence presented by the prosecution does it show that ₱ 150,000 was given to
and received by petitioner in Makati City. Instead, the evidence shows that the receipt
issued by petitioner for the money was dated 22 December 1999, without any indication
of the place where it was issued. As a matter of fact, the Deed of Sale with Assumption
of Mortgage prepared by petitioner was signed and notarized in Iloilo City, also on 22
December 1999. Petitioner claims that the money was actually delivered to him in Iloilo
and that absence of proof as to the place of delivery, one must rely on the disputable
presumption that things happened according to the ordinary course of nature and the
ordinary habits of life. putable presumption that things happened according to the
ordinary course of nature and the ordinary habits of life. The only time Makati City was
mentioned was with respect to the time when the check provided by petitioner was
dishonored by Equitable-PCI Bank in its De la Rosa-Rada Branch in Makati. Petitioner
asserts that the prosecution witness failed to allege that any of the acts material to the
crime of estafa had occurred in Makati City. Therefore, the trial court failed to acquire
jurisdiction over the case.
ISSUE: WON the case should be dismissed for lack of jurisdiction.
RULING: Yes, the case should be dismissed. In criminal cases, a court cannot exercise
jurisdiction over a person charged with an offense committed outside its limited territory.
The place where the crime was committed determines not only the venue of the action
but is an essential element of jurisdiction. It is a fundamental rule that for jurisdiction to
be acquired by courts in criminal cases, the offense should have been committed or any
one of its essential ingredients should have taken place within the territorial jurisdiction
of the court. It cannot take jurisdiction over a person charged with an offense allegedly
committed outside of that limited territory. If there is evidence showing that the crime
was committed elsewhere, the court should dismiss the criminal action.

In Fukuzune v. People, the Supreme Court dismissed the complaint for estafa because
the prosecution failed to prove that the essential elements of the offense took place in
the trial court’s jurisdiction.

In the case at bar, the prosecution also failed to prove that the offense of estafa was
committed within the Jurisdiction of Makati City. Under Article 315, par. 1 (b) of the
RPC, the elements of estafa are as follows: (1) that money, goods or other personal
property is received by the offender in trust or on commission, or for administration, or
under any other obligation involving the duty to make delivery of or to return the same;
(2) that there be misappropriation or conversion of such money or property by the
offender, or denial on his part of such receipt; (3) that such misappropriation or
conversion or denial is to the prejudice of another; and (4) there is a demand by the
offended party to the offender. Although it was mentioned that the check was
dishonored in a bank located in Makati, the fact is immaterial because such dishonor is
not an element of estafa.

Furthermore, Section 15 (a) of Rule 110 of the Revised Rules on Criminal Procedure of
2000 provides that subject to existing laws, the criminal action shall be instituted and
tried in the court of the municipality or territory where the offense was committed or
where any of its essential ingredients occurred. This principle is to ensure that the
accused is not compelled to move to, and appear in, a different court from that of the
province where the crime was committed as it may cause him great inconvenience in
looking for his witnesses and other evidence in another place. This principle echoes
more strongly in this case, where, due to distance constraints, coupled with his
advanced age and failing health, petitioner was unable to present his defense in the
charges against him. PETITION IS GRANTED. CASE DISMISSED
THE UNITED STATES, plaintiff-appellee, vs.
MARIANO M. GALLEGOS, ET AL., defendants. MARIANO M. GALLEGOS,
appellant.
[G.R. No. L-12739, December 8, 1917, JOHNSON, J.]

FACTS:
From the month of November, 1914, to the month of October, 1916, inclusive, in the
municipality of Cebu, Province of Cebu, P. I., Mariano Gallegos did lie with the accused
Benita Antioquia several times, within and without the offended party's own dwelling;
and said Benita Antioquia, on her part, being legally married to the undersigned, said
marriage not having dissolved up to the present time, voluntarily, illegally, and
criminally, did lie with the Gallegos; that as a result of their illicit relation the said
accused begot a daughter; in violation of the law.
Mariano M. Gallegos was duly arraigned, tried, found guilty of the crime charged in the
complaint and sentenced by the Honorable Adolph Wislizenus to be imprisoned for a
period of 3 years 6 months and 21 days of prision correccional, and to pay one-half part
of the costs. In the same decision the lower court ordered that a new order of arrest be
issued for the arrest of the codefendant Benita Antioquia.
At the opening of the trial, the attorney for the defendant Mariano M. Gallegos
presented a motion asking that the trial against him be suspended until his codefendant
could be arrested and brought into the court, which motion was denied by the lower
court. The appellant now alleges that the lower court committed an error in compelling
him to proceed to trial alone in the absence of his codefendant.

ISSUE:

(1) Whether or not, in a criminal action for the crime of adultery, one of the defendants
may be tried alone, where the complaint includes them both, and when for some reason
or other one of the codefendants has not been arrested and brought to trial.
(2) Whether or not the Court may require evidence that the crime was committed in its
jurisdiction

RULING: Yes, the defendant may be tried alone.


Yes, the Court may require evidence of the place of the commission of the
crime.

While the law provides that the complaint must be presented, in a criminal action for
adultery, (a) by the offended person and (b) against both of the alleged culprits, there is
no provision of law requiring that they shall be tried jointly. Not only is there no law
requiring that they should be tried jointly, but there is a positive provision of law
permitting them to be tried separately. (Section 33 of General Orders No. 58)
The mere fact that article 434 of the Penal Code requires the offended husband to
institute the criminal action for adultery against both of the alleged guilty parties does
not necessarily prevent either of them from obtaining a separate trial, nor prohibit the
court from dismissing the complaint as to one or the other under certain conditions.
When the complaint was filed by the offended husband against both of the guilty parties,
the proceedings then passed into the hands of the prosecuting officer who may move
for a dismissal of the complaint as to one if he is satisfied that he cannot established
guilty knowledge, on married, and such dismissal would not of itself require the court to
acquit the woman.
There are numerous cases, after the complaint has been properly presented, where one
or the other of the alleged guilty parties in a criminal action for adultery may be tried and
sentenced separate from the codefendant. For example, where one of the parties died
after the commencement of the action, or where the man was ignorant of the fact that
the woman was a married woman at the time of the commission of the alleged criminal
act.
(U. S. vs. Asuncion, 22 Phil. Rep., 358; U. S. vs. De la Torre and Gregorio, 25 Phil.
Rep., 36; decision of the supreme court of Spain, January 17, 1889; decision of the
supreme court of Spain, October 24, 1894.)
The appellant further alleges that the lower court committed an error in permitting the
case to be tried by a private attorney and in excusing the prosecuting attorney from
attending the hearing. Said permission was granted and no objection or protest was
made by the defendant nor by his attorney. We believe, however, that even if the
defendant had objected to the permission granted by the court allowing the prosecuting
attorney to be absent, that the error assigned could not in any was affect the sentence
of the lower court. We see nothing objectionable in this practice, provided always that
the fiscal retains control of the prosecution, and assumes full responsibility therefor.
The appellant further contends that the lower court committed an error in permitting the
prosecution, after it had closed its case, to present additional proof relating to the
jurisdiction of the court.

Section 31 of General Orders No. 58 provides, among other things (a) That the counsel
for the United States must offer evidence in support of the charge; (b) that the
defendant, or his counsel, may offer evidence in support of the defense, and (c) that the
parties may then respectively offer rebutting testimony, but rebutting testimony only,
unless the court, in furtherance of justice, permit them to offer new and additional
evidence bearing upon the main issue in question.

The question of the jurisdiction of the court is always a question of importance; and if
evidence is necessary to prove that fact, as it is in all criminal cases, so far as the place
of the commission of the crime is concerned, and the prosecution fails to prove that fact,
in the interest of justice the court may always permit it to present additional evidence, if
that fact appears before the trial of the case is closed. And, moreover, an examination
of the record shows that, without the additional proof, the court had right to take judicial
notice of the fact, from the evidence adduced, that the crime committed, if committed at
all, was within its jurisdiction.
THE UNITED STATES, vs.GUMERSINDO DE LA SANTA,
G.R. No. L-3181 October 10, 1907

FACTS: The defendant (Gumersindo dela Santa) seduced Teofila Sevilla under promise
of marriage early in the year 1902, at which time she was less than 21 years of age.
The complaint was not filed until February, 1906, when she was more than 24 though
less than 25 years of age, and was signed, sworn, and submitted by one Esteban
Sevilla, at whose "instance" these proceedings were had, he appearing as the private
prosecutor and alleging that he is the father of the said Teofila Sevilla. Gumersindo was
charged of Seduction.
Article 448 of the Penal Code is as follows:
Criminal proceedings for seduction can only be instituted on the complaint of the
offended person or her parents, grandparents, or guardian.
In order to proceed in cases of rape and in those of abduction committed with unchaste
design, the denunciation of the interested party, her parents, grandparents, or
guardians, shall suffice even though they do not present a formal petition to the judge.
If the person injured should, by reason of her age or mental condition, lack the requisite
personality to appear in court, and should, besides, be wholly unprotected, not having
parents, grandparents, brothers, or guardian of person or property to denounce the
crime, the procurador sindico or the or the public prosecutor may do so, acting on the
strength of public rumor.
In all the cases of this article the express or implied pardon of the offended party shall
extinguish penal action or the penalty, if it should have been already imposed on the
culprit.
The pardon shall never be presumed, except by the marriage of the offended party with
the offender.

ISSUE: WON the father has the authority to institute criminal action in case of seduction
when the child is already of legal age.

RULING: No. If the father does not institute such proceedings until after his daughter
has attained full age, we are of opinion that he loses the right so to do, and that this
right vests exclusively in the offended party, unless, of course, there is some legal
impediment, not arising out of nonage, which prevents her from maintaining such
criminal action.
The right to institute criminal proceedings in cases of seduction could not be reposed in
the offended person, her parents, grandparents, and guardian, at one and the same
time, without occasioning grave difficulties in the administration of justice, resulting from
the attempts of some of these persons to institute criminal proceedings contrary to the
wish and desire of the others.
Under the provisions of the Civil Code, a woman 23 years old is of age. From that
period she is in the full possession of her civil rights, save only in certain exceptional
cases expressly prescribed in the code. The right to appear and prosecute or defend an
action in the courts is not one of these exceptions, and indeed, it is inherent to the full
exercise of civil rights.
Under the provisions of the above-cited article 448 of the Penal Code, jurisdiction over
the crime of seduction is expressly denied the trial court unless such jurisdiction be
conferred by one of certain persons specified in the law; in this case, as we have seen,
by the offended person herself. The objection in this case is not, strictly speaking, to the
sufficiency of the complaint, but goes directly to the jurisdiction of the court over the
crime with which the accused was charged. It has been frequently held that a lack of
jurisdiction over the subject-matter is fatal, and subject to objection at any stage of the
proceedings, either in the court below or on appeal, and, indeed, where the subject
matter is not within the jurisdiction, the court may dismiss the proceeding ex mero motu.
Jurisdiction over the subject-matter in a judicial proceeding is conferred by the
sovereign authority which organizes the court; it is given only by law and in the manner
prescribed by law and an objection based on the lack of such jurisdiction can not be
waived by the parties. Hence, the accused in a criminal case can not, by express waiver
or otherwise, confer jurisdiction on a court over an offense as to which such jurisdiction
has not been conferred upon such court by law. Counsel further contends that since the
offended party appeared in court and testified, she may be said to have instituted the
proceedings, although the complaint is signed and sworn to by her father. It may be
sufficient answer to this contention to point out that there is nothing in the record to
indicate that the proceedings were, in fact, had at the instance of the daughter rather
than the father, the fact that she appeared and gave testimony not justifying such
conclusion because, being duly subpoenaed, she would have been compelled so to do
whether she appeared voluntarily or otherwise; but, as has been shown before, the
provisions of article 448 are so explicit and so positive that even though it appears that
she had, in fact, taken an active part in all the proceedings, this would not be sufficient
unless the complaint was submitted and the action formally maintained by her. That this
is the meaning of the provisions of the said article becomes clear upon a comparison of
the language used as to proceedings for seduction and proceedings in a case of rape.
In proceedings for seduction the language used expressly provides that they can only
be instituted and maintained on the complaint (a instancia) of the persons therein
mentioned; while in cases of rape and those of abduction committed with dishonest
designs, the denunciation (la denuncia) of the interested party, or her parents,
grandparents, or guardian, shall suffice, "though they do not present a formal complaint
to the judge.
THE UNITED STATES, complainant-appellant, vs. JOSE REYES, defendant-
appellee

FACTS:
Jose Reyes, employee of Manila-Dagupan Railway, collected from a passenger 1 peso
and 22 cents for a trip from Manila to Malolos. He issued a ticket worth 18 cents
simulating that said trip is only from Manila to Bocaue . He then appropriated the
difference. Upon reaching Tarlac, he rendered a sum of 6 pesos and 48 cents (which
includes the 18 cents) to the station master.
The crimes of estafa and falsification was filed in the Court of Instance of Manila.
ISSUE:
Whether or not Manila has jurisdiction over the case.
RULING:
No. Under article 20 of the General Compilation of Laws upon Criminal Procedure,
jurisdiction to try the offense charged is vested in the court of Tarlac, not only because it
was within the territory of that court that the appropriation constituting the crime of
estafa charged was committed, but also because within the same territory the accused
made use of the document alleged to be false.
REPUBLIC V. ASUNCION
G.R. NO. 108208, 11 Mach 1994
FACTS

An information was filed with RTC Quezon City accusing Manio, a member of the PNP,
of the crime of homicide for the shooting of a certain T/Sgt. Romeo Sadang during an
operation in response to a complaint that a person was creating trouble at Dumalay
Street in Novaliches.

RTC Judge Asuncion required the prosecution and the defense to comment on whether
the court should proceed with the trial of the case in view of the case of Deloso v.
Domingo where the SC rules that the Sandiganbayan has jurisdiction over offenses
committed by public officials when the penalty prescribe by law is higher than prision
correccional. On the ground of lack of jurisdiction, Judge Asuncion dismissed the
criminal case for the refiling of the same with Sandiganbayan. The motion for
reconsideration was also denied.

The Office of the Ombudsman, as intervenor, argued that the term “regular courts” in
R.A. No. 6975 refers to “civilian courts”, which include Sandiganbayan. The petitioner
insisted the contrary, and claimed that if it were the intention of R.A. No. 6975 to grant
to Sandiganbayan the jurisdiction over PNP members, then it would have explicitly used
the term “civil courts” as distinguished from “court martial”.
ISSUE
Whether or not Sandiganbayan has jurisdiction over the criminal case for homicide
against Manio, who was a PNP member during the time of the alleged commission of
the crime.

RULING
The Court clarified its ruling in the case of Deloso v. Domingo, and explained that for
Sandiganbayan to acquire jurisdiction over the subject matter, the penalty prescribed for
the crime charged must be higher than prision correccional and the fact that the crime
charged was committed in relation to their office must be alleged in the Information,
considering the rule that jurisdiction is determined by the allegations contained in the
Information. It is settled that an offense may be considered “committed in relation to the
office” if the offense is intimately connected with the office of the offender and
perpetuated while he was in the performance of his official functions.

In this case, there is no doubt that the penalty for the crime charged is higher than
prision correccional. However, it was not established in the Information that it was
committed by Manio in relation to his office.

Hence, the SC directed to conduct a preliminary hearing in this case to determine


whether the crime charged was committed by Manio in relation to his office. If in the
affirmative, the RTC shall order the transfer of the case to Sandiganbayan. Otherwise,
the RTC shall proceed with the trial and render judgment thereon.

MA. LUISA G. DAZON v. KENNETH Y. YAP


GR No. 157095, January 15, 2010
Facts:
Respondent Kenneth Y. Yap was the president of Primetown Property Group, Inc.,
(Primetown) the developer of Kiener Hills Mactan Condominium, a low-rise
condominium project. In November 1996, petitioner Ma. Luisa G. Dazon entered into a
contract with Primetown for the purchase of Unit No. C-108 of the said condominium
project. Petitioner made a downpayment and several installment payments. Primetown,
however, failed to finish the condominium project. Petitioner filed a criminal complaint
with the Office of the City Prosecutor of Lapu-Lapu City against respondent
Meanwhile, respondent, in connection with the resolution finding probable cause filed a
Petition for Review with the Department of Justice (DOJ). On June 14,2002, the DOJ
rendered a Resolution ordering the trial prosecutor to cause the withdrawal of the
Information. Hence, the prosecutor filed a Motion to Withdraw Information With the
RTC.
Issues:
Whether or not a regional trial court has jurisdiction over a criminal action arising from
violation of PD 957
Petitioner contends that jurisdiction is conferred by law and that there is no law
expressly vesting on the HLUKB exclusive jurisdiction over criminal actions arising from
violations of PD 957.
Ruling:
Jurisdiction over criminal actions arising from violations of PD 957 is vested in the
regular courts.
Noticeably, cases that are criminal in nature are not mentioned in the enumeration
quoted above. The primordial function of the HLURB, after all, is the regulation of the
real estate trade and business and not the conviction and punishment of criminals. "It
may be conceded that the legislature may confer on administrative boards or bodies,
quasi-judicial powers involving the exercise of judgment and discretion, as incident to
the performance of administrative functions. But in so doing, the legislature must state
its intention in express terms that would leave no doubt, as even such quasi-judicial
prerogatives must be limited, if they are to be valid, only to those incidental to or in
connection with the performance of administrative duties, which do not amount to
conferment of jurisdiction over a matter exclusively vested in the courts".
Administrative agencies being tribunals of limited jurisdiction can only wield such
powers as are specifically granted to them by their enabling statutes. PD 957 makes the
following specific grant of powers to the NHA (now HLURB) for the imposition of
administrative fines, and it also mentions penalties for criminal cases... the power in
relation to criminal liability mentioned in the immediately succeeding provision, to
impose, upon conviction, fines above ten thousand pesos and/or imprisonment, was not
conferred on it
Not having been specifically conferred with power to hear and decide cases which are
criminal in nature, as well as to impose penalties therefor, we find that the HLURB has
no jurisdiction over criminal actions arising from violations of PD 957.
The DOJ made no reversal of such finding of probable cause. Instead, it directed the
withdrawal of the information on the erroneous premise that it is the HLURB which has
jurisdiction over the case
VICENTE FOZ, JR. and DANNY G. FAJARDO, Petitioners, vs.
PEOPLE OF THE PHILIPPINES, Respondent.
G.R. No. 167764 October 9, 2009
FACTS:
Information filed:
That on or about the 5th day of July, 1994 in the City of Iloilo, Philippines and within the
jurisdiction of this court, both the accused as columnist and Editor-Publisher,
respectively, of Panay News, a daily publication with a considerable circulation in the
City of Iloilo and throughout the region, did then and there willfully, unlawfully and
feloniously with malicious intent of impeaching the virtue, honesty, integrity and
reputation of Dr. Edgar Portigo, a physician and medical practitioner in Iloilo City, and
with the malicious intent of injuring and exposing said Dr. Edgar Portigo to public hatred,
contempt and ridicule, write and publish in the regular issue of said daily publication on
July 5, 1994, a certain article entitled "MEET DR. PORTIGO, COMPANY PHYSICIAN,"
quoted verbatim hereunder, to wit: xxx
The article was about Dr. Portigo, company physician of the Local San Miguel
Corporation (SMC) office who sworn to help others to do their best to promote the
health of their patients, but the opposite was happening. Lita Payunan, wife of an SMC
employee consulted Dr Portigo about her illness which took the latter seven months to
conclude that she had rectum myoma. Payunan sought help from a clinic where Dr
Portigo maintained his clinic as well. Dr Portigo got angry after knowing that the
Payunans chose a surgeon without his nod. Lita was operated but by another doctor
and was released from the hospital after three weeks. Lita complained of difficulty in
urinating and defecating and had a second operation by a surgeon recommended by Dr
Portigo. The patient woke up to find her anus and vagina closed and a hole with
catheter punched on her right side. It was followed by bad news of cancer which Dr
Portigo recommended another operation, but another doctor had advised that it was
futile because the illness was on its terminal stage.
Petitioners were found guilty beyond reasonable doubt of the crime of libel by the
Regional Trial Court of Iloilo which was later affirmed by the Court of Appeals hence the
petition.
Petitioners raised for the first time the issue that the information charging them with libel
did not contain allegations sufficient to vest jurisdiction in the RTC of Iloilo City.
ISSUE:
WON RTC Iloilo has jurisdiction over the offense charged.
HELD:
No, RTC Iloilo has no jurisdiction over the offense charged.
Venue in criminal cases is an essential element of jurisdiction. The Court held in
Macasaet v. People that: xxx the jurisdiction of a court over the criminal case is
determined by the allegations in the complaint or information. And once it is so shown,
the court may validly take cognizance of the case.
Article 360. Persons responsible.—Any person who shall publish, exhibit or cause the
publication or exhibition of any defamation in writing or by similar means, shall be
responsible for the same.
The author or editor of a book or pamphlet, or the editor or business manager of a daily
newspaper, magazine or serial publication, shall be responsible for the defamations
contained therein to the same extent as if he were the author thereof.
The criminal action and civil action for damages in cases of written defamations, as
provided for in this chapter shall be filed simultaneously or separately with the court of
first instance of the province or city where the libelous article is printed and first
published or where any of the offended parties actually resides at the time of the
commission of the offense.
In Agbayani v. Sayo, the rules on venue in Article 360 were restated as follows:
1. Whether the offended party is a public official or a private person, the criminal action
may be filed in the Court of First Instance of the province or city where the libelous
article is printed and first published.
2. If the offended party is a private individual, the criminal action may also be filed in the
Court of First Instance of the province where he actually resided at the time of the
commission of the offense.
3. If the offended party is a public officer whose office is in Manila at the time of the
commission of the offense, the action may be filed in the Court of First Instance of
Manila.
4. If the offended party is a public officer holding office outside of Manila, the action may
be filed in the Court of First Instance of the province or city where he held office at the
time of the commission of the offense
Applying the foregoing law to this case, since Dr. Portigo is a private individual at the
time of the publication of the alleged libelous article, the venue of the libel case may be
in the province or city where the libelous article was printed and first published, or in the
province where Dr. Portigo actually resided at the time of the commission of the offense.
The allegations in the Information that "Panay News, a daily publication with a
considerable circulation in the City of Iloilo and throughout the region" only showed that
Iloilo was the place where Panay News was in considerable circulation but did not
establish that the said publication was printed and first published in Iloilo City.
Article 360 of the Revised Penal Code as amended provides that a private individual
may also file the libel case in the RTC of the province where he actually resided at the
time of the commission of the offense. The Information filed against petitioners failed to
allege the residence of Dr. Portigo. While the Information alleges that "Dr. Edgar Portigo
is a physician and medical practitioner in Iloilo City," such allegation did not clearly and
positively indicate that he was actually residing in Iloilo City at the time of the
commission of the offense. It is possible that Dr. Portigo was actually residing in another
place.
Settled is the rule that jurisdiction of a court over a criminal case is determined by the
allegations of the complaint or information, and the offense must have been committed
or any one of its essential ingredients took place within the territorial jurisdiction of the
court. Considering that the Information failed to allege the venue requirements for a libel
case under Article 360, the Court finds that the RTC of Iloilo City had no jurisdiction to
hear this case. Thus, its decision convicting petitioners of the crime of libel should be
set aside for want of jurisdiction without prejudice to its filing with the court of competent
jurisdiction.
SOLEMNIDAD M. BUAYA, petitioner, vs. THE HONORABLE WENCESLAO M.
POLO, Presiding Judge, Branch XIX, Regional Trial Court of Manila and the
COUNTRY BANKERS INSURANCE CORPORATION, respondents.

FACTS
Solemnidad Buaya was charged with the crime of Estafa. She alleges that she lives in
Cebu and the alleged misappropriation of funds occurred in cebu. She questions the
jurisdiction of the court of Manila over the case.
Solemnidad was an agent of Country Bankers Insurance Corporation tasked to collect
the insurance premiums and to remit it as soon as possible to the corporation’s office in
Manila. She failed and refused to comply with her obligation and with intent to defraud,
misappropriated with the whole amount and converted the said amount of P358,850.00
to her own personal use and benefit, to the damage and prejudice of said CBCI.
She sought to annul and set aside the orders of denial issued by the respondent Judge
of the Regional Trial Court of Manila, Branch XIX, on her Motion to Quash/Dismiss and
Motion for Reconsideration in Criminal Case No. L-83-22252 entitled "People of the
Philippines vs. Solemnidad M. Buaya." based on the following grounds (a) the court has
no jurisdiction over the case and (b) the subject matter is purely civil in nature.

ISSUE
W/N the City of Manila had proper jurisdiction over the case.
RULE (DISMISSED)

Yes, It is well-settled that the averments in the complaint or information characterize the
crime to be prosecuted and the court before which it must be tried.
(JURISPRUDENCE) In order to determine the jurisdiction of the court in criminal cases,
the complaint must be examined for the purpose of ascertaining whether or not the facts
and the punishment provided for by law fall within the jurisdiction of the court where the
complaint is filed. The jurisdiction of courts in criminal cases is determined by the
allegations of the complaint or information, and not by the findings the court may make
after the trial
(LAW) Section 14(a), Rule 110 of the Revised Rules of Court provides: In all criminal
prosecutions the action shall be instituted and tried in the court of the municipality or
province wherein the offense was committed or any of the essential elements thereof
took place.
Besides, the crime of estafa is a continuing or transitory offense which may be
prosecuted at the place where any of the essential elements of the crime took place.
One of the essential elements of estafa is damage or prejudice to the offended party.
The Corporation has its principal place of business and office at (Kalaw) Manila. The
failure of the petitioner to remit the insurance premiums she collected allegedly caused
damage and prejudice to CBIC in Manila.

The other contention that the subject matter is purely civil in nature is still yet to be
proved.
SUBIDO vs. SANDIGANBAYAN

Bayani Subido Jr. and Rene Parina, petitioners vs. Sandiganbayan, respondent
(G.R. No. 122641, 20 | January 1997)

The reckoning point for the Sandiganbayan to have jurisdiction over the accused and
the offense charged is the time of the commission of the crime. The information filed in
Criminal
A procedural and curative statute may validly be given retroactive effect, there being no
impairment of contractual or vested rights.

FACTS: Petitioners Subido and Parina were charged with Arbitrary Detention (penalized
under Revised Penal Code) in an information filed on 28 July 1995. The case was
docketed as Criminal Case 22825. The information alleged that then Commissioner of
Immigration and Deportation Bayani Subido Jr. and then BID Special Agent Rene
Parina willfully, unlawfully and feloniously caused the issuance and implementation of
an arrest warrant dated 25 June 1992 against James J. Maksimuk, in conspiracy with
each other and while performing their official functions. The said
warrant caused Maksimuk’s detention for forty-three (43) days.

The petitioners then filed on 28 August 1995 a Motion to Quash, where they alleged that
the Sandiganbayan had no jurisdiction over their person and the offense charged and
they should be tried in RTC of Manila, as Arbitary Detention (penalized under the
Revised Penal Code) is not covered under R.A. No. 7975. Also, they argued that R.A.
7975 should be given prospective application because when the case was filed, Subido
was already a private person since he separated from government service on 28
February 1995 while Parina did not hold a position corresponding to Salary Grade 27.

The prosecution filed their opposition to the Motion to Quash on 28 September 1995,
contending
that under Sec. 4(b) of R.A. 7975, the Sandiganbayan had jurisdiction over the
petitioners and the offense charged and that the basis of Sandiganbayan’s jurisdiction is
“the position of the accused in the government service when the offense charged was
committed and not the nature
of the offense charged, provided the offense committed by the accused was in the
exercise of his
duties and in relation to his office.”

The petitioners’ Motion to Quash was denied by the Sandiganbayan (in its Resolution
dated 25 October 1995). The Sandiganbayan ruled that it has jurisdiction over the case,
as stated in Sec. 4 (a)e of R.A. 7975.

As the arraignment was 10 November 1995, the petitioners filed on 9 November 1995
Motion
for Reconsideration, but the Sandiganbayan that motion through its Order dated 10
November
1995. Hence, this petition for Certiorari under Rule 65 on ground of grave abuse of
discretion
amounting to lack of jurisdiction.

ISSUES: W/N the Sandiganbayan had jurisdiction over the petitioners and the offense
of
Arbitrary Detention charged against them?

HELD: The petition of Subido and Parina was DISMISSED by the Supreme Court.
Ruling – The Court ruled that as per Sections 2 and 7 of Rep. Act 7975, the
Sandiganbayan has jurisdiction over the offense, as the information in Criminal Case
22825 stated that the petitioners “willfully, unlawfully and feloniously caused the
issuance and implementation of an arrest warrant dated 25 June 1992 against James J.
Maksimuk, in conspiracy with each other and while performing their official functions.”
Moreover, the penalty for the Arbitrary Detention due to Maksimuk’s detention of forty-
three (43) days is prision mayor (6 years 1 day – 12 years), and under E.O. 184, the
Sandiganbayan has jurisdiction over such offense. It also stated that the petitioners
overlooked that the reckoning point for the Sandiganbayan to have jurisdiction over
them and the offense charged is the time of the commission of the crime. Moreover,
Subido never denied in the lower court that at time when he committed the offense, he
was classified under Salary Grade 27 for the position of Commissioner of Immigration
and Deportation. Contrary to the petitioner’s argument, the Court also ruled that the
Sandiganbayan Law (PD 1606, as amended by RA 7975) is not a penal law which
defines crimes and provide punishment for such acts, but a procedural law which
prescribes rules and forms of procedure of enforcing rights or obtaining redress for their
invasion. Thus, as a procedural and curative statute, R.A. No. 7975 may validly be
given retroactive effect, there being no impairment of contractual or vested rights
G.R. No. 137017-18 February 8, 2000
RAMON G. CUYCO, petitioner,
vs.
THE HONORABLE SANDIGANBAYAN, Fifth Division and THE HONORABLE
OFFICE OF THE SPECIAL PROSECUTOR, respondents.

Facts:
On 1995, Graft Investigation Officer Ma. Lourdes M. Vilaria-Yap found probable cause
for the indictment of petitioner Ramon G. Cuyco, and two others for the violation of the
Anti-Graft and Corruption Act (RA 3019) sections (a) and (e).
Ombudsman approved the recommendation, and thereafter, the prosecution filed with
the Sandiganbayan two informations against petitioner for the offenses aforesaid.
On 1998, petitioner filed a motion to quash the informations on the ground of lack of
jurisdiction of the Sandiganbayan under RA 7975 which was subsequently amended by
RA 8249 (Act defining the Jurisdiction of Sandiganbayan).
Prosecutor filed a comment with Sandiganbayan interposing no objection to the
remanding of the case to RTC of Zamboanga.
Sandiganbayan issued a resolution denying petitioner’s motion to quash and issued a
preventive suspension of petitioner and co-accused for (90) days. Motion for
reconsideration was likewise denied.
Hence, this petition for certiorari.
Issue:
Whether or not at the time of the filing of the informations, on 1995, the Sandiganbayan
had jurisdiction over the cases against petitioner for violation of Anti Graft and
Corruption Act Sections 3(a) and (e), (RA 3019), as amended.

Ruling:
No. Sandiganbayan has no jurisdiction over cases where the accused is occupying a
position with a Salary Grade lower than 27. In this case, at the time of commission of
offense in 1992, the accused was occupying a position with Salary Grade 26, hence,
jurisdiction falls with the RTC.
The Sandiganbayan has no jurisdiction over violations of Section 3(a) and (e), Republic
Act No. 3019, as amended, unless committed by public officials and employees
occupying positions of regional director and higher with Salary Grade "27" or higher,
under the Compensation and Position Classification Act of 1989 (Republic Act No.
6758) in relation to their office.
DP: Wherefore grants the petition, annuls the resolutions of Sandiganbayan and
ordered dismissal of Criminal Cases for lack of jurisdiction.
G.R. No. L-38634 June 20, 1988
REPUBLIC OF THE PHILIPPINES, (PEOPLE OF THE PHILIPPINES), petitioner,
vs.
HON. DELFIN VIR. SUNGA, as Presiding Judge, CFI Branch I, Camarines Sur,
ARISTON ANADILLA, RAFAEL ANADILLA and JOSE ANADILLA, respondents.

I. Facts: An information for Attempted Homicide was filed on August 10, 1964 by the
Provincial Fiscal of Camarines Sur against accused-private respondents Rafael
Anadilla, Ariston Anadilla and Jose Anadilla. But the trial was set only on March 11 and
12, 1974, ten years after the information was filed. After a week, on March 20, 1974, the
court dismissed the case relying on the affidavit of desistance executed by the offended
party and subscribed to and sworn by the branch clerk of court.
The provincial fiscal then moved to reconsider the order of dismissal, which was denied
by the court.
II. Issue: Whether or not the court a quo may dismiss a criminal case on the basis of an
affidavit of desistance executed by the offended party, but without a motion to dismiss
filed by the prosecuting fiscal.
III. Ruling: No. The Crespo doctrine provides “The rule therefore in this jurisdiction is
that once a complaint or information is filed in Court any disposition of the case as its
dismissal or the conviction or acquittal of the accused rests in the sound discretion of
the Court. Although the fiscal retains the direction and control of the prosecution of
criminal cases even while the case is already in Court he cannot impose his opinion on
the trial court. The Court is the best and sole judge on what to do with the case before it.
The determination of the case is within its exclusive jurisdiction and competence. A
motion to dismiss the case filed by the fiscal should be addressed to the Court who has
the option to grant or deny the same. It does not matter if this is done before or after the
arraignment of the accused or that the motion was filed after a reinvestigation or upon
instructions of the Secretary of Justice who reviewed the records of the investigation."
The Court takes the view that, while the Crespo doctrine has settled that the trial court is
the sole judge on whether a criminal case should be dismissed (after the complaint or
information has been filed in court), still, any move on the part of the complainant or
offended party to dismiss the criminal case, even if without objection of the accused,
should first be referred to the prosecuting fiscal for his own view on the matter. He is,
after all, in control of the prosecution of the case and he may have his own reasons why
the case should not be dismissed. It is only after hearing the prosecuting fiscal's view
that the Court should exercise its exclusive authority to continue or dismiss the case.
REPUBLIC V. REGALARIO
G.R. NO. 101451, 23 March 1993

FACTS

RTC Lucena City found Regalario, et. al., (appellants) guilty of murder for the death of
one Menardo Garcio. The judgment of conviction was promulgated in open court on 17
January 1991. On 31 January 1991, or 14 days after judgment, the appellants filed a
motion for reconsideration. The same was denied by the RTC on 22 February 1991. On
4 March 1991, or 10 days after the motion for reconsideration was denied, the
appellants filed a notice of appeal. This was denied due course by the RTC for having
been filed out of time.

ISSUE
Whether or not the notice of appeal should be given due course.

RULING
Yes, the notice of appeal should be given due course, even if filed out of time, on the
ground of estoppel by laches.

The Rules of Court provides that an appeal must be taken within 15 days from the
promulgation of judgment. This period shall be interrupted from the time a motion for
reconsideration (or a new trial) is filed, and shall run again after notice of order denying
said motion.

In this case, the notice of appeal was filed on 4 March 1991 or 10 days after the denial
of the motion for reconsideration on 22 February 1991. Since judgment was delivered
on 31 January 1991, the appellants actually have until 02 February 1991 to file the
notice of appeal. But the counting was interrupted by the filing of the motion for
reconsideration on 31 January 1991, which was 14 days after the promulgation of
judgment. The period began to run again after the denial of the motion for
reconsideration on 22 February 1991, which means that the notice of appeal should
have been filed on 23 February 1991. However, in this case, the notice of appeal was
filed only on 4 March 1991, or 10 days after the denial of the motion of reconsideration.
The SC ruled that the appeal could have been dismissed outright for being time-barred.
However, it was not shown that neither of the public or private prosecutor moved for the
dismissal of the appeal or objected to the order of the trial court to elevate the records to
the SC. It was only after the appellants had filed their brief that the appellee raised the
issue of belated appeal.

To obviate miscarriage of justice and in view of the gravity of the offense and the
penalty involved, the Court decided to review the case and render judgment on the
merits thereof.

DELA CRUZ vs. MOYA


(G.R. No. L-65192, April 27, 1988)

FACTS: Rodolfo Dela Cruz, a member of the Armed Forces of the


Philippines, together with other PC men, received a mission order to proceed to
Barangay Pangi, Maco, Sto. Tomas, Davao for the purpose of verifying and
apprehending persons who were allegedly engaged in illegal cockfighting. Dela
Cruz and company proceeded and caught in flagrante the operators of said illegal
cockfighting, but said operators resisted arrest. The soldiers left the place but
they brought with them to the PC Headquarters the evidence of the crime, such
as gaffs and fighting cocks. The operators of the illegal cockfights, including the
deceased Eusebio Cabilto, followed the soldiers on their way back to the PC
Headquarters, catching up with them on the Tagum-Mati National Highway.
Fighting ensued and in the scuffle, Dela Cruz shot Cabilto.
Dela Cruz was charged with homicide in the Court of First Instance of Davao.
While the case was pending trial, Presidential Decree Nos. 1822 and 1822-A
were promulgated by the President of the Philippines on January 16, 1981,
vesting in courts-martial jurisdiction over crimes committed by members of the
Armed Forces or of the Philippine Constabulary in performance of their duties.
Claiming that the crime for which he was charged was committed in relation to
the performance of his duties, Dela Cruz filed with the Court of First Instance of
Davao a motion to transfer the case to the military authorities so he could be
tried by court martial. The motion was denied.
Issue: Whether or not the civil courts have jurisdiction over the subject matter of
Criminal Case.
Held: One of the essential requisites of a valid court proceeding is that the court
hearing the case must have jurisdiction over the subject matter of the case. If
the court is acting without jurisdiction, then the entire proceedings are null and
void.
Jurisdiction over the subject matter is determined by the statute in force at the
time of the commencement of the action. And once jurisdiction is vested in the
court, it is retained up to the end of the litigation.
In the instant case, the information was filed on August 2, 1979. On such date,
by virtue of General Order No. 59, dated June 24, 1977, published in Official
Gazette in July 11, 1977, military tribunals created under General Order No. 8
exercised exclusive jurisdiction over all offenses committed by military personnel
of the Armed Forces of the Philippines while in the performance of their official
duty or which arose out of any act or omission done in the performance of their
official duty; Provided, that for the purpose of determining whether an offense
was committed while in the performance of official duty or whether it arose out
of an act or omission done in the performance of official duty, a certificate
issued by the Secretary of National Defense to that effect shall be conclusive
unless modified or revoked by the President. . . " (Section 1.) As no
amendatory law was ever published in the Official Gazette between the time
G.R. No. 59 was published until the information in Criminal Case was filed on
August 2,1979, then said General Order No. 59 remained in force on said date.
In the case at bar, it is not disputed that at the time of the commission of the
alleged offense, petitioner Dela Cruz was a member of the Philippine
Constabulary, and that the shooting of the deceased Cabilto was committed
while petitioner was executing the Mission Order.
The evidence of the prosecution presented in court likewise shows that Cabilto
was shot while petitioner was executing the mission order. These undisputed
facts compel this Court to declare that respondent court was without jurisdiction
to try the case against petitioner Dela Cruz. The Solicitor General points out that
at the time the information was filed, Presidential Decrees Nos. 1822 and 1822A
which vest in the courts-martial jurisdiction over offenses committed by members
of the AFP in the performance of their duties were not yet in effect, the same
having been promulgated only in 1981. Truly, PD 1822 and 1822-A are
inapplicable to the case at bar. However, General Order No. 59 cited above
applies.
SERAFIN TIJAM, ET AL., plaintiffs-appellees, vs.MAGDALENO SIBONGHANOY
alias GAVINO SIBONGHANOY and LUCIA BAGUIO, defendants, MANILA SURETY
AND FIDELITY CO., INC. (CEBU BRANCH) bonding company and defendant-
appellant.
G.R. No. L-21450 April 15, 1968

Facts:
On July 19, 1948 — barely one month after the effectivity of Republic Act No. 296
known as the Judiciary Act of 1948 — the spouses Serafin Tijam and Felicitas Tagalog
commenced a civil case in the Court of First Instance of Cebu against the spouses
Magdaleno Sibonghanoy and Lucia Baguio to recover from them the sum of P1,908.00,
with legal interest thereon from the date of the filing of the complaint until the whole
obligation is paid, plus costs.

A writ of attachment was issued by the court against defendants' properties, but the
same was soon dissolved upon the filing of a counter-bond by defendants and the
Manila Surety and Fidelity Co., Inc., on the 31st of the same month. The Court rendered
judgment in favor of the plaintiffs and issued a writ of execution against the defendants.
The writ having been returned unsatisfied, the plaintiffs moved for the issuance of a writ
of execution against the Surety's bond, against which the Surety filed a written
opposition upon two grounds, namely, (1) Failure to prosecute and (2) Absence of a
demand upon the Surety for the payment of the amount due under the judgment.

Surety moved to quash the writ on the ground that the same was issued without the
required summary hearing provided for in Section 17 of Rule 59 of the Rules of Court.
As the Court denied the motion, the Surety appealed to the Court of Appeals without
raising the issue on lack of jurisdiction.

The CA affirmed the order appealed from. Surety filed a Motion to dismiss, alleging: that
appellee’s action was filed in the CFI for the recovery of the sum of P1,908.00 only; that
a month before that date Republic Act No. 296, otherwise known as the Judiciary Act of
1948, had already become effective, Section 88 of which placed within the original
exclusive jurisdiction of inferior courts all civil actions where the value of the subject-
matter or the amount of the demand does not exceed P2,000.00, exclusive of interest
and costs; that the CFI therefore had no jurisdiction to try and decide the case.
CA set aside its earlier decision and referred the case to SC since it has exclusive
jurisdiction over "all cases in which the jurisdiction of any inferior court is in issue.”

Issue:
Whether or not Surety bond is estopped from questioning the jurisdiction of the CFI
Cebu for the first time upon appeal.

Ruling:
Yes, the SC held that the Surety is now barred by laches after almost fifteen years
before the Surety filed its motion to dismiss raising the question of lack of jurisdiction for
the first time. A party may be estopped or barred from raising a question in different
ways and for different reasons.

It has been held that a party can not invoke the jurisdiction of a court to sure affirmative
relief against his opponent and, after obtaining or failing to obtain such relief, repudiate
or question that same jurisdiction. It was further said that the question whether the court
had jurisdiction either of the subject-matter of the action or of the parties was not
important in such cases because the party is barred from such conduct not because the
judgment or order of the court is valid and conclusive as an adjudication, but for the
reason that such a practice can not be tolerated — obviously for reasons of public
policy. Furthermore, it has also been held that after voluntarily submitting a cause and
encountering an adverse decision on the merits, it is too late for the loser to question
the jurisdiction or power of the court.

The facts of this case show that from the time the Surety became a quasi-party on July
31, 1948, it could have raised the question of the lack of jurisdiction of the Court of First
Instance of Cebu to take cognizance of the present action by reason of the sum of
money involved which, according to the law then in force, was within the original
exclusive jurisdiction of inferior courts. It failed to do so. Instead, at several stages of the
proceedings in the court a quo as well as in the Court of Appeals, it invoked the
jurisdiction of said courts to obtain affirmative relief and submitted its case for a final
adjudication on the merits. It was only after an adverse decision was rendered by the
Court of Appeals that it finally woke up to raise the question of jurisdiction. Were we to
sanction such conduct on its part, We would in effect be declaring as useless all the
proceedings had in the present case since it was commenced on July 19, 1948 and
compel the judgment creditors to go up their Calvary once more. The inequity and
unfairness of this is not only patent but revolting. The Court said that it is not right for a
party who has affirmed and invoked the jurisdiction of a court in a particular matter to
secure an affirmative relief, to afterwards deny that same jurisdiction to escape a
penalty.

UPON ALL THE FOREGOING, the orders appealed from are hereby affirmed, with
costs against the appellant Manila Surety and Fidelity Company, Inc.

PEOPLE vs. HON. DEMOSTHENES L. MAGALLANES


G.R. Nos. 118013-14 October 11, 1995

Facts:
A team of Police Officers together with civilian agents arrested and abducted Rufino
Gargar and Danilo Lumangyao who were allegedly members of the group that had
swindled the Dumancas spouses. The two suspects were then taken to the Ceres
Compound, where Jeanette Dumancas identified Lumangyao as a member of the group
that had swindled her. She then asked about the money that the group had received
from her. Lumangyao said that the money had already been divided among his partners
long time ago. Thereafter, they were brought to D' Hacienda Motel where the two were
shot and killed by the team.

On 13 January 1994, two information for kidnapping for ransom with murder were filed
in RTC Bacolod against the team, five of whom are members of the PNP. Several
accused filed a motion for the inhibition of Judge Garvilles, RTC Judge of Bacolod, who
subsequently voluntarily inhibited himself from further hearing both cases, which were
thereafter re-raffled to another RTC branch, presided by herein public respondent Judge
Demosthenes L. Magallanes.

On the other hand, the private prosecutors of respondents moved for the transmittal of
the records of the cases to the Sandiganbayan on the ground that the trial court has no
jurisdiction over the cases because the offenses charged were committed in relation to
the office of the accused PNP officers.

The trial court ruled that the Sandiganbayan does not have jurisdiction over the subject
cases because the information do not state that the offenses were committed in relation
to the office of the accused PNP officers.
Issue: WON Sandiganbayan has jurisdiction over the two criminal cases wherein some
of the accused implicated as principals are members of the Philippine National Police
(PNP).

Ruling:
No. There is no dispute that the prescribed penalties for the offenses charged in the
cases are higher than prision correcional or imprisonment for more than six years
however for the Sandiganbayan to have exclusive original jurisdiction over offenses
committed by public officers or employees, Section 4 of P.D. No. 1606, as amended by
P.D. No. 1861 provides that it is not enough that the penalty prescribed therefor is
higher than prision correccional or imprisonment for six years, or a fine of P6,000.00; it
is also necessary that the offenses or felonies were committed in relation to their office.
An offense may be considered as committed in relation to the office if it cannot exist
without the office, or if the office is a constituent element of the crime. The offense must
be intimately connected with the office of the offender which must be alleged in the
information.

Further, the two information filed indicate that the accused arrested and investigated the
victims and then killed the two in the course of the investigation. The information merely
allege that the accused, for the purpose of extracting or extorting the sum of money,
abducted, kidnapped, and detained the two victims, and failing in their common
purpose, they shot and killed the said victims. For the purpose of determining
jurisdiction, it is these allegations that shall control, and not the evidence presented by
the prosecution at the trial.

Accordingly, for lack of an allegation in the information that the offenses were committed
in relation to the office of the accused PNP officers or were intimately connected with
the discharge of the functions of the accused, the subject cases come within the
jurisdiction of the Regional Trial Court and not of the Sandiganbayan as insisted by the
petitioner.

Assuming then for the sake of argument that the information in the said cases allege
that the crimes charged were committed by the five PNP officers in relation to their
office, it would appear that the cases would still fall within the jurisdiction of the trial
court. Amendments to Section 4 of P.D. No. 1606, as amended, introduced by R.A. No.
7975, which was approved on 30 March 1995, enumerated those which government
positions will fall within its jurisdiction and among of which are the PNP chief
superintendent and PNP officers of higher rank. However we may note that on 13
January 1994 is when the information of the herein case was filed and that the
jurisdiction of a court is determined by the law in force at the time of the commencement
of the action. Hence, it is Section 4 of P.D. No. 1606 which applies to this case.

Jurisdiction once acquired is not affected by subsequent legislative enactment placing


jurisdiction in another tribunal. It remains with the court until the case is finally
terminated. Hence, the Sandiganbayan or the courts, as the case may be, cannot be
divested of jurisdiction over cases filed before them by reason of R.A. No. 7975. They
retain their jurisdiction until the end of the litigation.

UY VS CA
(UNAVAILABLE)
G.R. Nos. 120681-83 October 1, 1999
JEJOMAR C. BINAY, petitioner,
vs.
HON. SANDIGANBAYAN (Third Division) and the DEPARTMENT OF INTERIOR
AND LOCAL GOVERNMENT, respondents.
The Office of the Ombudsman filed before the Sandiganbayan three separate
informations against petitioner Jejomar Binay, one for violation of Article 220 (Illegal Use
of Public Funds) of the Revised Penal Code, and two for violation of Section 3 (e) of
R.A. No. 3019 (Violation of Anti-Graft and Corrupt Practices Act) and alleged that the
acts constituting these crimes were committed in 1987 during petitioner's incumbency
as Mayor of Makati.
Binay moved to quash the informations contending that the six-year delay from the time
the charges were filed in the Office of the Ombudsman on July 27, 1988 to the time the
informations were filed in the Sandiganbayan on September 7, 1994 constituted a
violation of his right to due process. Arraignment of the accused was held in abeyance
pending the resolution of this motion.
SB then issued a resolution denying petitioner’s motion to quash and further the latter’s
motion for reconsideration. In the meantime, the prosecution filed a motion to suspend
the accused ‘pendente lite’ (benefits) which was later granted and ordered for a 90-day
suspension. Petition for certiorari was filed by Mayor Binay in the SC praying that the
resolution denying his motion for reconsideration be set aside and claimed that he was
denied of his rights when the suspension was ordered even before he could file his
reply to the petitioner’s opposition. SC then, directed the SB to permit petitioner to file
said reply.
The SB nonetheless reiterated its previous resolutions and order after the submission of
the reply. Meanwhile, RA 7975 redefining the jurisdiction of SB took effect on May 1995
so much so that the petitioner filed before SB a motion to refer his cases to the RTC of
Makati alleging that the SB has no jurisdiction over said cases when it issued its
resolutions and suspension order on June 1995. The SB in a follow-up resolution
denied the petitioner’s motion. Hence this present petition, prohibition and mandamus
questioning the jurisdiction of SB over the criminal cases.
Issue: Whether or not the Sandiganbayan been ousted of its jurisdiction over the case
of municipal mayor after the passage of Republic Act No. 7975.
Ruling: No. The Court rules that it is the Sandiganbayan which has jurisdiction over the
subject cases. Sec. 4. Jurisdiction. — The Sandiganbayan shall exercise:
(a) Exclusive original jurisdiction in all cases involving:
(1) Violations of Republic Act No. 3019, as amended, otherwise known as the Anti-Graft
and Corrupt Practices Act, Republic Act No. 1379, and Chapter II, Section 2, Title VII of
the Revised Penal Code;
(2) Other offenses or felonies committed by public officers and employees in relation to
their office, including those employed in government-owned or controlled corporations,
whether simple or complexed with other crimes, where the penalty prescribed by law is
higher than prision correccional or imprisonment for six (6) years, or a fine of P6,000.00;
PROVIDED, HOWEVER, that offenses or felonies mentioned in this paragraph where
the penalty prescribed by law does not exceed prision correccional or imprisonment for
six (6) years or a fine of P6,000.00 shall be tried by the proper Regional Trial Court,
Metropolitan Trial Court and Municipal Circuit Trial Court.
xxx xxx xxx
On May 16, 1995, R.A. No. 7975 took effect. At this time, Mayor Binay had not yet been
arraigned in the Sandiganbayan.
In cases where none of the accused are occupying positions corresponding to salary
grade "27" or higher, as prescribed in the said Republic Act No. 6758, or military and
PNP officers mentioned above, exclusive original jurisdiction thereof shall be vested in
the proper regional trial court, metropolitan trial court, municipal trial court, and
municipal circuit trial court, as the case may be, pursuant to their respective jurisdictions
as provided in Batas Pambansa Blg. 129, as amended.
Petitioners contend that they do not come under the exclusive original jurisdiction of the
Sandiganbayan because:
(1) At the alleged time of the commission of the crimes charged, petitioner municipal
mayors were not classified as Grade 27. (2) Municipal mayors are not included in the
enumeration in Section 4a(1) of P.D. No. 1606, as amended by R.A. No. 7975. (3)
Congressional records reveal that the law did not intend municipal mayors to come
under the exclusive original jurisdiction of the Sandiganbayan.
(1)The SC ruled that to determine whether an official is within the exclusive original
jurisdiction of the Sandiganbayan, therefore, reference should be made to R.A. No.
6758 and the Index of Occupational Services, Position Titles and Salary Grades.
Petitioner mayor are "local officials classified as Grade "27" and higher under the
Compensation and Position Classification Act of 1989.
(2)The law is clear in this case. As stated earlier, Section 4a(1) of P.D. No. 1606, as
amended by R.A. No. 7975, speaks of "[o]fficials of the executive branch occupying the
positions of regional director and higher, otherwise classified as grade "27" and higher,
of the Compensation and Position Classification Act of 1989."The Court fails to see how
a different interpretation could arise even if the plain meaning rule were disregarded and
the law subjected to interpretation.
(3) The resort to congressional records to determine the proper application of the law in
this case is unwarranted in this case for the same reason that the resort to the rule of
inclusio unius est expressio alterius is inappropriate. Verily, the interpretation of the law
desired by the petitioner may be more humane but it is also an elementary rule in
statutory construction that when the words and phrases of the statute are clear and
unequivocal, their meaning must be determined from the language employed and the
statute must be taken to mean exactly what it says.
From the foregoing discussion, it is clear that the cases against petitioner Binay cannot
be referred to the regular courts under Section 7 of R.A. No. 7975, which provides:
Sec. 7. Upon effectivity of this Act, all criminal cases in which trial has not begun in the
Sandiganbayan shall be referred to the proper courts.
R.A. No. 7975, by virtue of Section 7, belongs to the exception rather than the rule. The
provision is transitory in nature and expresses the legislature's intention to apply its
provisions on jurisdiction to "criminal cases in which trial has not begun in the
Sandiganbayan." To this extent, R.A. 7975 is retroactive. Thus, under both R.A. Nos.
7975 and 8429, the Sandiganbayan retains jurisdiction over said cases.
WHEREFORE, the consolidated petition is DISMISSED.
Separate Opinions
PANGANIBAN, J., separate opinion;
I concur with the majority that, as a rule, the Sandiganbayan retains jurisdiction over
criminal cases involving municipal mayors.
Due to their peculiar factual circumstances, however, Petitioner Binay's cases, I believe,
should be deemed exceptions and referred to the "proper courts," that is, the regional
trial courts. These factual circumstances are simple: (1) the Informations charging Binay
were filed in the Sandiganbayan on July 7, 1994, prior the enactment of RA 7975; and
(2) when RA 7975 took effect on May 16, 1995, trial in the anti-graft court had not yet
commenced. In fact, Binay had not been arraigned yet. These undisputed facts are
plainly governed by the unambiguous provision of Section 7, RA 7975, which reads:
Sec. 7. Upon the effectivity of this Act, all criminal cases in which trial has not begun in
the Sandiganbayan shall be referred to the proper courts.
The majority, however, complicates the above syllogistic application of the law by ruling
that before Section 7 could be used, a prior determination as to which court has
jurisdiction over the cases should first be undertaken. Since the aforesaid general rule
states that the Sandiganbayan retains jurisdiction over municipal mayors, then Binay's
cases should be referred by the anti-graft court to itself, not to the regional trial courts.
With due respect, I believe this rather circumlocutory interpretation renders Section 7
useless. In fact, I daresay that said interpretation or explanation is much more difficult to
understand than the provision itself. Indeed, why should the words "proper courts" be
deemed to include the Sandiganbayan? The majority's ruling leads to the absurdity of
the Sandiganbayan's being required to refer to itself a criminal case already pending
before it, one in which trial has not yet begun. I would rather rest on the most
fundamental rule in statutory construction: Interpretation is needed only when the law is
vague, not when it is clear and unambiguous, 1 as in the case of Section 7, RA 7975.
Consequently, I vote (1) to GRANT the Petition in G.R. Nos. 120681-83, because
Binay's cases fall under the exception stated in Section 7, RA 7975; and (2) to DISMISS
the Petition in G.R. No. 128136, because Petitioner Magsaysay's cases were filed after
RA 7975 had taken effect; they are thus covered by the general rule that the
Sandiganbayan has jurisdiction over municipal mayors.
SANCHEZ AND MANAGAY VS SANDIGANBAYAN
(G.R. No. 120011, September 07, 1999)

FACTS:
1. Sandiganbayan denied petitioners’ joint motion to dismiss the criminal case against them
on the ground that there is lack of jurisdiction over the offense charged because they were
previously charged before the court martial with violation of the Articles of War involving
the same facts as the charge of violation of Section 3(e) of the Anti-Graft and Corrupt
Practices Act before the Sandiganbayan.
2. Petitioners now filed a special civil action for certiorari with preliminary injunction
assailing the resolution of the Sandiganbayan.
3. The Supreme Court grants the petition.
4. Petitioners are officers of the Philippine Army (PA). Lt. Col. Sanchez at times material
hereto was Commanding Officer, 9 Post Engineer Detachment, Headquarters and
th

Headquarters Support Group (HHSG), PA. Major Managay was G-4, HHSG, PA.
5. The pre-trial investigating officer submitted a report to the Commanding General stating
that there was a prima facie case against petitioners for violation of Article 95 of the
Articles of War for causing wrongful release of P599,547.00 for payment of
repair/renovation of G-10 Office, Philippine Army, equivalent to 88.55% completion of
the work when in truth and in fact, only 25% of the work had been completed, to the
damage and prejudice of the government.
6. On the basis of the report, the Judge Advocate of the PA, initiated court martial proceedings
against petitioners before the Philippine Army Permanent General Court Martial No. 2. In
addition, he referred the findings to the Provincial Prosecutor of Rizal, recommending the
filing of an information with the Sandiganbayan against petitioners and Gaudencio
Romualdez for violation of R. A. No. 3019.
7. The Provincial Prosecutor of Rizal endorsed the records to the Ombudsman.
8. When they were arraigned before General Court Martial No. 2 of the Philippine Army,
they pleaded not guilty.
9. April 18, 1994: The Ombudsman then filed with the Sandiganbayan an information against
Sanchez and Managay for violation of R. A. No. 3019, Section 3 (e).
10. Petitioners filed a joint motion to dismiss the case before the Sandiganbayan on the ground
that it has no jurisdiction over the case.
11. The Sandiganbayan denied the motion to dismiss, for lack of merit, ruling that the offenses
charged in the court martial and the information before the Sandiganbayan are distinct and
separate from each other.
12. Petitioners were arraigned before the Sandiganbayan. They pleaded not guilty.
13. Petitioners filed a motion for reconsideration of the denial reiterating that the
Sandiganbayan had no jurisdiction over the case as the court martial had acquired original
and exclusive jurisdiction over the case, pursuant to R. A. No. 7055, and that the acts
complained of in the charge sheet in the court martial and the Information before the
Sandiganbayan were the same or identical.
14. Again, the Sandiganbayan denied the motion for reconsideration for lack of merit.

ISSUE: W/N the Sandiganbayan has jurisdiction over the case.

HELD: Although the Sandiganbayan had jurisdiction at the time the charge was filed on April 18,
1994, it no longer has jurisdiction over the case under Republic Act No. 7975, enacted on March
30, 1995, or even under Republic Act No. 8249, enacted on February 5, 1997.

The Special Prosecutor, Office of the Ombudsman, acknowledged the recent enactment of
Republic Act No. 7975 under which the Sandiganbayan "lost" its jurisdiction over the case
primarily because the public officials charged, petitioners herein, were officers of the Philippine
Army below the rank of full colonel. Hence, in the words of the Special Prosecutor, "necessarily
the herein criminal case No. 20461 against petitioners should be referred to the proper court as
trial has not begun in the public respondent Sandiganbayan." Regrettably, the Sandiganbayan
denied petitioners' motion for reconsideration even when at the time it was resolved Republic Act
No. 7975 was in effect, and its enactment was precisely to declog its docket of "small fry" cases.

In denying petitioners' motion for reconsideration seeking to dismiss the information against them,
the Sandiganbayan acted without jurisdiction.

The Court granted the petition for certiorari and annulled the resolutions of the Sandiganbayan.
The Court orders the Sandiganbayan to refer the criminal case to the proper court.

LACSON VS EXECUTIVE SECRETARY


G.R. 128096 January 20, 1999

FACTS:
On May 1995, 11 members of the Kuratong Baleleng gang were killed along
Commonwealth Ave., QC by the Anti-Bank Robbery and Intelligence Task Group
(ABRITG) headed by Chief Supt. Jewel Canson of PNP. One of the components of
ABRITG is the Presidential Anti-Crime Commission-Task Force Habagat (PACC-TFH)
headed by petitioner Chief Supt. Panfilo Lacson. SPO2 Eduardo delos Reyes told the media
that what happened was a summary execution (or a rub out) and not a shoot-out between
the Kuratong Baleleng gang members and the ABRITG. The panel of investigators headed
by Omb. Blancaflor found that the incident was a legitimate police operation. However, a
review board led by Overall Deputy Omb. Villa recommended the indictment for multiple
murder against 26 respondents, including Lacson. Lacson was among those charged as
principal in information for murder before the Sandiganbayan. All the accused filed
separate motions questioning its jurisdiction.
Sandiganbayan ordered the cases transferred to the QC RTC which has original and
exclusive jurisdiction under RA 7975 (An Act to Strengthen the Functional and Structural
Organization of the Sandiganbayan, Amending for that Purpose PD 1606, as Amended),
as none of the principal accused has the rank of PNP Chief Supt. or higher (Lacson is NOT
a PNP Chief Supt.)
The Office of the Special Prosecutor moved for a reconsideration. While it is pending, RA
8249 (An Act Further Defining thr Jurisdiction of the Sandiganbayan, Amending for the
Purpose PD 1606, as Amended, Providing Funds Therefor, and for Other Purposes) was
enacted to law which deletes the word “principal” from the phrase “principal accused”.
Thus, the Sandiganbayan takes cognizance again of the case.
ISSUE/S:
Whether or not Sandiganbayan has jurisdiction.
RULING:
NO. QC RTC that has jurisdiction. The jurisdiction of a court is defined by the Constitution
or statute. The elements of that definition must appear in the complaint or information so
as to ascertain which court has jurisdiction over a case. Hence, the elementary rule that the
jurisdiction of a court is determined by the allegations in the complaint or information, and
not by the evidence presented by the parties at the trial.
The multiple murder charge falls under Section 4(b) of R.A. 8249, which requires that the
offense charged must be committed by the offender in relation to his office in order for the
Sandiganbayan to have jurisdiction over it.
An offense is said to have been committed in relation to the office if it (the offense) is
‘intimately connected’ with the office of the offender and perpetrated while he was in the
performance of his official functions. While the information states that Lacson, et al
committed the crime of murder in relation to their public office, there is, however, no
specific allegation of facts that the shooting of the victim by the said principal accused was
intimately related to the discharge of their official duties as police officers. Likewise, the
amended information does NOT indicate that the said accused arrested and investigated
the victim and then killed the latter while in their custody.
While there is the allegation in the amended information that the said accessories
committed the offense “in relation to office as officers and members of the (PNP),” the
Court do not see the intimate connection between the offense charged and the accused’s
official functions. That phrase is merely a conclusion between of law, not a factual
averment that would show the close intimacy between the offense charged and the
discharge of the accused’s official duties.
What is controlling is the specific factual allegations in the information that would indicate
the close intimacy between the discharge of the accused’s official duties and the
commission of the offense charged, in order to qualify the crime as having been committed
in relation to public office.
RIZAL COMMERCIAL BANKING CORPORATION (RCBC), petitioner,
vs.
HON. LUCIA V. ISNANI, PRESIDING JUDGE OF BRANCH 59, RTC, MAKATI, HON. FELICIDAD Y.
NAVARRO-QUIAMBAO, PRESIDING JUDGE, BRANCH 65, MTC, MAKATI, AND LOLITA ENCELAN,
respondents.
G.R. No. 117383 March 6, 1995
FACTS: In a complaint filed, on April 27, 1994 (a few days after the effectivity of Republic Act No. 7691,
amending Batas Pambansa Blg. 129), with the Makati Regional Trial Court (RTC), Private respondent Lolita
Encelan sought to recover from petitioner Rizal Commercial Banking Corporation (RCBC) actual damages
of $5,000.00 or its Philippine peso equivalent of approximately P137,675.00.
RCBC moved to dismiss the case for lack of jurisdiction on the ground that the complaint was cognizable
by the MTC of Manila, not the RTC, the principal demand prayed for not being in excess of P200,000.00.
Respondent RTC Judge Lucia V. Isnani, instead of dismissing the complaint, transferred, on July 8, 1994,
the entire records of the case to MTC. The case was assigned to the sala of MTC Judge Felicidad Navarro-
Quiambao. Upon learning of the transfer, RCBC sought with the MTC a reconsideration thereof.
On 16 September 1994, respondent MTC Judge Navarro-Quiambao issued an Order denying the motion.
Hence, this petition by RCBC.
LAW: R.A. No. 7691 provides the ff:
A. Civil actions and settlement of estate proceedings, testate or intestate, including the grant of
provisional remedies when warranted, where the value of the personal property, estate, or amount of the
demand does not exceed One Hundred Thousand Pesos (P100,000.00), or Two Hundred Thousand Pesos
(P200,000.00) in Metro Manila, exclusive of interest, damages of whatever kind, attorney's fees, litigation
expenses, and costs (the amount of which must be specifically alleged), shall, after the effectivity of R.A.
7691, be filed with the metropolitan and municipal trial courts.
B. Civil actions or a settlement of estate proceedings, aforesaid, pending with regional trial courts
which have already reached the pretrial stage at the time of the effectivity of R.A. 7691 shall remain with
said courts for proper disposition. The transfer of pending cases (which have already reached the pretrial
stage) to metropolitan or municipal trial courts may be allowed, however, provided the following conditions
concur; viz.:
a. the case is cognizable by the municipal or metropolitan trial court under the present provisions of
the Act; and
b. the parties agree to the transfer of the case from the regional trial court to the municipal or
metropolitan trial court.
C. R.A. 7691 took effect on 15 April 1994 or fifteen (15) days after its publication on 30 March 1994.
Cases filed on or after such effectivity date must accord with the new jurisdictional mandate; a
disregard thereof shall constitute a ground for the dismissal of the action or proceeding for lack of
jurisdiction.
ISSUE: Whether or not Judge Isani erred in transferring the entire records of the case to MTC.
RULING: No. In this case, the principal demand prayed for in the complaint filed on April 27, 1994, or after
R.A. 7691 had already become effective, with the Makati RTC, is only for US$5,000.00, or approximately
P137,675.00 in Philippine currency, and thusly within the exclusive jurisdiction of the Metro Manila MTCs.
Instead of ordering the transfer of the complaint to the MTC, respondent RTC judge, therefore, should have
dismissed the case prayed for by petitioner for lack of jurisdiction.
Therefore, Motion for reconsideration is granted. The appealed order of RTC Judge Lucia V. Isnani (denying
petitioner's motion to dismiss and transferring the case instead to the MTC) and the order issued by MTC
Judge Felicidad Y. Navarro-Quiambao (to whom the case was transferred), (denying petitioner's motion for
reconsideration), are hereby set aside.

REPUBLIC v. ASUNCON
G.R. No. L-108208, March 1994
FACTS: An information was filed with RTC Quezon City accusing Manio, a member of the PNP,
of the crime of homicide for the shooting of a certain T/Sgt. Romeo Sadang during an operation
in response to a complaint that a person was creating trouble at Dumalay Street in Novaliches.

RTC Judge Asuncion required the prosecution and the defense to comment on whether the
court should proceed with the trial of the case in view of the case of Deloso v. Domingo where
the SC rules that the Sandiganbayan has jurisdiction over offenses committed by public officials
when the penalty prescribed by law is higher than prision correccional and the murder charge
against the petitioner carries the penalty of reclusion temporal in its maximum period of death
(Art. 248, Revised Penal Code), hence, it is cognizable by the Sandiganbayan, and the
Ombudsman has primary jurisdiction to investigate it.

On the ground of lack of jurisdiction, Judge Asuncion dismissed the criminal case for the refiling
of the same with Sandiganbayan. The motion for reconsideration was also denied. The Office of
the Ombudsman, as intervenor, argued that the term “regular courts” in R.A. No. 6975 refers to
“civilian courts”, which include Sandiganbayan. The petitioner insisted the contrary, and claimed
that if it were the intention of R.A. No. 6975 to grant to Sandiganbayan the jurisdiction over PNP
members, then it would have explicitly used the term “civil courts” as distinguished from “court
martial”.

The petitioner also said that since there is an irreconcilable conflict between RA 6975 (current
law) and PD 1606 (former law), the former law should be repealed by the current law.
ISSUE: WON the Sandiganbayan is included in civil or regular courts.
WON there is an irreconcilable conflict between RA 6975 and PD 1606
RULING: 1. During the Marcos regime, it was the court-martial that has jurisdiction over police
officers, policemen, firemen, and jail guards pursuant to PD 1850. The next administration was
clearly against that idea. Hence, the overwhelming sentiment of the framers of the 1987
Constitution against the martial law regime and the militarization of the police forces prompted
them to explicitly direct the establishment and maintenance of one police force, which shall be
national in scope and civilian in character. This civilian character is unqualified and
unconditional and is, therefore, all-embracing.

The mandate of RA 6975 is to divest courts-martial of any jurisdiction over criminal cases
involving PNP members and to return or transfer that jurisdiction to the civil courts which is often
interchangeably used by the Legislative as regular courts. The question now is Whether or not
the Sandiganbayan is included in civil or regular courts.

Regular courts are those within the judicial department of the government, namely, the Supreme
Court and such lower courts as may be established by law. The Sandiganbayan was created by
P.D. No. 1486 pursuant to the mandate of Section 5, Article XIII of the 1973 Constitution which
was later on became PD 1606 and lastly, due to amendments, became PD 1861. Under the
amendments introduced by P.D. No. 1861, the Sandiganbayan has jurisdiction over the
following cases:

Sec. 4. Jurisdiction. — The Sandiganbayan shall exercise:

(a) Exclusive original jurisdiction in all cases involving:

(1) Violations of Republic Act No. 3019, as amended, otherwise known as the Anti-Graft and Corrupt Practices Act,
Republic Act No. 1379, and Chapter II, Section 2, Title VII of the Revised Penal Code;

(2) Other offenses or felonies committed by public officers and employees in relation to their office, including those
employed in government-owned or controlled corporations, whether simple or complexed with other crimes, where
the penalty prescribed by law is higher than prision correccional or imprisonment for six (6) years, or a fine of
P6,000.00: PROVIDED, HOWEVER, that offenses or felonies mentioned in this paragraph where the penalty
prescribed by law does not exceed prision correccional or imprisonment for six (6) years or a fine of P6,000.00 shall
be tried by the proper Regional Trial Court, Metropolitan Trial Court, Municipal Trial Court and Municipal Circuit Trial
Court.

(b) Exclusive appellate jurisdiction:

(1) On appeal, from the final judgments, resolutions or orders of the Regional Trial Courts in cases originally decided
by them in their respective territorial jurisdiction.

(2) By petition for review, from the final judgments, resolution or orders of the Regional Trial Courts in the exercise of
their appellate jurisdiction over cases originally decided by the Metropolitan Trial Courts, Municipal Trial Courts and
Municipal Circuit Trial Courts, in their respective jurisdiction. . . .

Undoubtedly, it was ruled that Sandiganbayan is a regular court thus included in the term
regular court in RA 6975. The contention of the Petitioner that it is not because it is regarded as
a special court and not a regular does not hold water because they are not opposite of one
another. Special means "designed for a particular purpose; confined to a particular purpose,
object, person, or class," and is, therefore, the antonym of general. On the other hand, regular
means "steady or uniform in course, practice, or occurrence," as opposed to casual or
occasional. The Sandiganbayan is a court with special jurisdiction because its creation as a
permanent anti-graft court is constitutionally mandated and its jurisdiction is limited to certain
classes of offenses. Courts of special jurisdiction, which are permanent in character, are also
regular courts.
2. There is no merit in the theory of petitioner that Section 46 of R.A. No. 6975 impliedly
repealed Section 4 of P.D. No. 1606, as amended by P.D. No. 1861, as regards the jurisdiction
of the Sandiganbayan over members of the PNP. Both provisions are not irreconcilable and the
presumption against an implied repeal has not been overcome. Implied repeal may be indulged
in only if the two laws are inconsistent, or the former law must be repugnant as to be
irreconcilable with the latter law. Contrary to the issue, the two provisions can well go together
with full and unhampered effect to both and without doing violence to either, thereby giving spirit
to the maxim that every statute must be so construed and harmonized with other statutes as to
form a uniform system of jurisprudence.

FERDINAND CUNANAN, petitioner, vs.


HON. HERMIN E. ARCEO, as Presiding Judge of Branch 43 of the Regional Trial Court of San
Fernando, Pampanga; HON. PEDRO M. SUNGA, JR., as Presiding Judge of the Regional Trial
Court of Branch 42 of San Fernando, Pampanga; and THE PEOPLE OF THE
PHILIPPINES, respondents.
[G.R. No. 116615, March 1, 1995, FELICIANO, J.]

FACTS:
1. Cunanan was on a mission on that day at Candaba, Pampanga when he went out of the Police
Station after hearing the commotion. He fired a warning shot was a clear indication that his intention
was to restore peace and order disturbed and broken by the fight between the victim and Rogelio
Agustin and later on between the deceased and Pfc. Efren Bass.
2. On 5 April 1991, an information for Murder was filed against petitioner Ferdinand Cunanan before
Branch 46 of the Regional Trial Court ("RTC") of San Fernando, Pampanga, presided over by Judge
Norberto C. Ponce. The Information alleged that Cunanan was a member of the Philippine National
Police; it contained no averment that he had committed the offense charged in relation to his public
office. On 4 November 1993, the parties having presented their evidence, Judge Arceo required them
to submit memoranda. Note: The case was later transferred to Brach 43 with Arceo as presiding judge.
3. On 11 March 1994, the Supreme Court promulgated its En Banc Decision in Republic v. Hon. Asuncion,
et al., which laid down the rule that the Sandiganbayan has exclusive and original jurisdiction to take
cognizance of offenses committed by public officers in relation to their office, where the penalty
prescribed by law is higher than prision correccional or imprisonment of six (6) years or more or a fine
of P6,000.00. The Supreme Court further held that in the event an Information failed to allege that
the accused-public officer had committed the offense charged in relation to his office, the RTC
hearing the criminal case, pending at the time of the promulgation of the Asuncion rule, shall conduct
a preliminary hearing to determine the existence or absence of this material fact.
• If this material fact is found to be present, the RTC shall order the transfer of the case to
the Sandiganbayan for docketing, and the latter shall proceed to hear the case as if the same
had been originally instituted with it.
• If it be determined that that fact is absent, the RTC seized with the case shall proceed with
the trial and render judgment on the case.
4. Judge Arceo proceeded to apply these holdings by conducting a hearing solely to ascertain if
petitioner had committed the offense charged in relation to his office. He ruled that petitioner had
committed the offense charged while in the performance of his official functions. He then held that
the RTC had no jurisdiction to try this case and that, accordingly, any decision it may render thereon
would be null and void. Judge Arceo dismissed Criminal Case No. 5708 "for refiling with the
Sandiganbayan," pursuant to the Asuncion ruling.
5. In a further Order dated 23 May 1994, Judge Arceo modified his earlier order and ordered the case
forwarded to the Sandiganbayan and the complete records of the same transmitted therewith as if it
was originally filed with the said Court.
6. In an Order dated 24 May 1994, upon motion by the prosecution, Judge Arceo inhibited himself
from further hearing the case. The case was then raffled to the sala of Branch 42, the RTC of San
Fernando, Pampanga, co-public respondent Judge Pedro M. Sunga Jr. presiding. He denied, among
other things, petitioner's Opposition to the Order directing the transmittal of the records of his case
to the Sandiganbayan.

Petitioner’s Contentions:
• Jurisdiction over the case was fixed in the RTC by the terms of the Information for Murder dated 5
April 1991, which contained no averment that he had committed the felony in relation to his office,
and that such jurisdiction is not determined by the result of the evidence presented at the trial. He
also cites the Court's pronouncement in Sanchez v. Hon. Demetriou, et al. that the use or abuse of
public office does not inhere in the crime of Murder as an element.
• The RTC judge in Asuncion who had decreed a transfer of the case to the Sandiganbayan did so when
the prevailing case law was Deloso v. Domingo, which did not require that an Information contain an
averment that the accused public officer had committed the offense charged in relation to his office,
before the Sandiganbayan can take cognizance of the case. In contrast, the public respondents here
had decreed a transfer of the case to the Sandiganbayan when the new, prevailing case law was 19

already in force and which now requires the presence of such material averment in an Information
before a case can be taken cognizance of by the Sandiganbayan. 20

• The Asuncion ruling is inapplicable to the present case, since here trial had already ended and the case
was already submitted for decision when the Asuncion ruling was promulgated. A transfer of his case
to the Sandiganbayan at this late stage will, accordingly, expose him (petitioner) to double jeopardy of
punishment for the same offense.
• Petitioner believes Judge Arceo's Order dated 21 April 1994 dismissing the case for lack of
jurisdiction over the offense charged amounts to an acquittal of petitioner.

ISSUE: Whether or not the fact that the crime was committed in relation to public office should be
contained in the Information is material in determining jurisdiction

RULING: No, it is not material.


1. Section 4 (a-2) of P.D. No. 1602 as amended by P.D. No. 1861 provides as follows:
Sec. 4. Jurisdiction. — The Sandiganbayan shall exercise:
(a) Exclusive original jurisdiction in all cases involving:
xxx xxx xxx
(2) Other offenses or felonies committed by public officers and employees in relation to their office,
including those employed in government-owned or controlled corporations, whether simple
or complexed with other crimes, where the penalty prescribed by law is higher than prision correccional
or imprisonment for six (6) years, or a fine of P6,000.00; PROVIDED, HOWEVER, that offenses
or felonies mentioned in this paragraph where the penalty prescribed by law does not exceed prision
correccional or imprisonment for six (6) years or a fine of P6,000.00 shall be tried by the proper Regional
Trial Court, Metropolitan Trial Court, Municipal Trial Court and Municipal Circuit Trial Court.
xxx xxx xxx

2. It is firmly settled that jurisdiction over the offense charged is a matter that is conferred by
law.
3. Whenever the above two (2) requisites are present, jurisdiction over the offense is vested in the
Sandiganbayan. This is true even though the information originally filed before the RTC did not aver that the
accused public officer had committed the offense charged in relation to his office.
4. The RTC's initial assumption of jurisdiction over the offense charged in this case did not, therefore,
prevent it from subsequently declaring itself to be without jurisdiction, that lack of jurisdiction having
become apparent from subsequent proceedings in that case.
5. Maintenance of peace and order is one of the duties of a policeman. And, that was what the accused
was doing when the deceased was killed. Thus, it is clear that the offense is intimately connected with the
office of the accused and perpetuated while he was in the performance of his official functions.
6. The principle set out in Montilla v. Hilario is that an offense may be considered as committed in
relation to the accused's office if "the offense cannot exist without the office" such that "the office [is] a
constituent element of the crime.
It may be noted, once more, that the absence in the information filed on 5 April 1991 before Branch 46 of
the RTC of San Fernando, Pampanga, of an allegation that petitioner had committed the offense charged in
relation to his office, is immaterial and easily remedied. Respondent RTC judges had forwarded petitioner's
case to the Sandiganbayan, and the complete records transmitted thereto in accordance with the directions of
this Court set out in the Asuncion case: ". . . As if it was originally filed with [the Sandiganbayan]."

RAMON G. CUYCO, vs.THE HONORABLE SANDIGANBAYAN, Fifth Division and


THE HONORABLE OFFICE OF THE SPECIAL PROSECUTOR, G.R. No. 137017-18
February 8, 2000

FACTS: On April 18, 1995, Graft Investigation Officer Ma. Lourdes M. Vilaria-Yap found
probable cause for the indictment of petitioner Ramon G. Cuyco, Generoso P. Germino and Melcy
V. Wee for violation of Section 3(a), Republic Act No. 3019, and petitioner Ramon G. Cuyco
together with Rolando R. Madarang for violation of Section 3(e) of the same Act, and
recommended the filing of two information against petitioner, together with the other respondents.
On October 30, 1995, the Ombudsman approved the recommendation, and on November
2, 1995, the prosecution filed with the Sandiganbayan two information against petitioner for the
offenses aforesaid. 2

On June 20, 1997, petitioner filed with the Sandiganbayan a motion to quash the
information for lack of jurisdiction, contending that the Sandiganbayan had no jurisdiction over
the cases under Republic Act No. 7975, which was subsequently amended by Republic Act No.
8249.
On June 8, 1998, Prosecutor Jacqueline J. Ongpauco-Cortel filed with the Sandiganbayan
her comment stating that "the prosecution interposes no objection to the remanding of the case to
the Regional Trial Court of Zamboanga City."
On August 5, 1998, the Sandiganbayan issued a resolution denying petitioner's motion to
quash and on September 21, 1998, issued another resolution ordering the preventive suspension of
petitioner and his co-accused for ninety (90) days.
On September 23, 1998, petitioner filed with the Sandiganbayan a motion for
reconsideration seeking to set aside the resolutions in question and to dismiss the criminal cases
for want of jurisdiction.
On December 16, 1998, the Sandiganbayan issued a resolution denying petitioner's motion
for reconsideration. 6

Petitioner contends that at the time of the commission of the offense in 1992, he was
occupying the position of Director II, Salary Grade 26, hence, jurisdiction over the cases falls
with the Regional Trial Court
Hence, this petition. 7
ISSUE: Whether or not at the time of the filing of the information on November 2, 1995 the
Sandiganbayan had jurisdiction over the cases.

RULING: No.
The Sandiganbayan has jurisdiction over offenses and felonies, whether simple or
complexed with other crimes committed by public officers and employees mentioned in subsection
(a) of Section 4, Republic Act No. 7975, as amended by Republic Act No. 8249 in relation to their
office, where the accused holds a position with salary grade "27" and higher under the
Compensation and Position Classification Act of 1989.
The Sandiganbayan has no jurisdiction over violations of Section 3(a) and (e), Republic
Act No. 3019, as amended, unless committed by public officials and employees occupying
positions of regional director and higher with Salary Grade "27" or higher, under the Compensation
and Position Classification Act of 1989 (Republic Act No. 6758) in relation to their office.

ERNESTO MORALES y DELA CRUZ, Petitioner, v. COURT OF APPEALS


FACTS:
Morales was accused of selling and delivering .46 grams of shabu. Such violation of
RA 7691 as amended by RA 7659 is punishable by imprisonment not exceeding
prision correccional or six (6) years. Petitioner filed petition to dismiss on the ground
that MTC has jurisdiction over the case. RTC dismissed the petition since it has
concurrent original jurisdiction over all cases involving offenses punishable under
the Dangerous Drugs Act.
Morales filed in CA a petition for certiorari but was also dismissed for lack of
jurisdiction.
ISSUE:
Whether or not RTC has jurisdiction over violation of Dangerous Drugs Act
punishable by less than 6 years.
RULING:
Yes. Amended of Section 32 of B.P. Blg. 129 by R.A. No. 7691 provides:
SEC. 32. Jurisdiction of Metropolitan Trial Courts, Municipal Trial Courts and
Municipal Circuit Trial Courts in Criminal Cases. — Except in cases falling within
the exclusive original jurisdiction of Regional Trial Court and of the Sandiganbayan,
the Metropolitan Trial Courts, Municipal Trial Courts, and Municipal Circuit Trial
Courts shall exercise:chanrob1es virtual 1aw library

(2) Exclusive original jurisdiction over all offenses punishable with imprisonment
not exceeding six (6) years irrespective of the amount of fine, and regardless of other
imposable accessory or other penalties, including the civil liability arising from such
offender or predicated thereon, irrespective of kind, nature, value or amount thereof:
Provided, however; That in offenses involving damage to property through criminal
negligence, they shall have exclusive original jurisdiction thereof.
The exclusive original jurisdiction of the Metropolitan Trial Courts, Municipal Trial
Courts, and Municipal Circuit Trial Courts in criminal cases does not cover those
cases which by provision of law fall within the exclusive original jurisdiction of
Regional Trial Courts and of the Sandiganbayan regardless of the prescribed penalty.
ALFREDO CUYOS y TULOR, petitioner, vs.HON. NICOLAS P. GARCIA, Presiding Judge, Municipal Court, San
Fernando, Pampanga and THE PEOPLE OF THE PHILIPPINES, respondents.
G.R. No. L-46934 April 15, 1988

Facts:
Petitioner was charged before the Municipal Court of San Fernando, Pampanga, with homicide with multiple
serious physical injuries and damage to property, through reckless imprudence. Petitioner was driver of a cargo
truck which had collided with a Volkswagen automobile in a vehicular accident which resulted in the death of
one (1) person and physical injuries to four (4) other people.

Petitioner entered a plea of not guilty at his arraignment. After arraignment, respondent Judge set the case for
trial. Before trial could commence, however, petitioner filed a " Motion to Remand the Case to the Court of
First Instance for Trial", alleging lack of jurisdiction over the case on the part of the Municipal Court. Petitioner's
argument was that the amended criminal complaint alleged that the Volkswagen car had suffered damages
amounting to P18,000.00, and that under Art 365 of the RPC, the crime with which he was charged would carry
a fine in an amount ranging from the amount of the damage to three (3) times the value of the damage alleged
(i.e. 3 x P18,000.00 or P54,000.00). He further contended that under the Judiciary Act of 1948, the Municipal
Court had jurisdiction only over offenses punishable by a fine not exceeding P6,000.00. On the same date,
petitioner filed an Urgent Motion to Postpone the trial of the case relying on the same grounds set out in his
Motion to Transfer the Case to the Court of First Instance.

The respondent Municipal judge issued an order denying the Motion to transfer the Case to the Court of First
Instance and set the trial of the case. A verbal Motion for Reconsideration by petitioner was denied. Hence,
this petition.

Issue:
Whether or not the respondent Municipal Court of San Fernando, Pampanga has jurisdiction to try the criminal
case against petitioner.

Ruling:
No, the SC held that the respondent Municipal Court has no jurisdiction to try the criminal case against
petitioner on the ground that the penalty involved is beyond the jurisdiction of the Municipal Court.

In imposing the corresponding penalty, to the quasi-offense of reckless imprudence resulting in physical
injuries and damage to property, Article 48 of the Revised Penal Code should be applied. However, there may
be cases, as the one at bar, where the imposable penalty for the physical injuries charged would come within
the jurisdiction of the municipal or justice of the peace court while the fine, for the damage to property, would
fall on the Court of First Instance. As the information cannot be split into two, one for damages and another
for the physical injuries, the jurisdiction of the court to take cognizance of the case must be determined not by
the corresponding penalty for the physical injuries charged but by the fine imposable for the damage to
property resulting from the reckless imprudence.

Since the maximum fine imposable in the present case is P54,000.00, and the maximum imprisonment
imposable (for the homicide through reckless imprudence) is six (6) years, clearly, the criminal charge involved
falls outside the jurisdiction of the Municipal Trial Court and consequently within the jurisdiction of the
Regional Trial Court of San Fernando, Pampanga.

PEOPLE OF THE PHILIPPINES vs SANDIGANBAYAN AND VICTORIA AMANTE


G.R. No. 167304 August 25, 2009
Facts:
Victoria Amante was a member of the Sangguniang Panlungsod of Toledo City, Province of Cebu
at the time pertinent to this case. On January 14, 1994, she was able to get hold of a cash advance
in the amount of P71,095.00 in order to defray seminar expenses of the Committee on Health and
Environmental Protection, which she headed. No liquidation was made after almost two years and
so on December 22, 1995, a demand letter was issued by the City Auditor asking respondent to
settle her unliquidated cash advance within 72 hours from receipt of the demand. Upon the
recommendation of the Commission on Audit (COA), the Office of the Deputy Ombudsman for
Visayas (OMB-Visayas) resolved to file an Information for Malversation of Public Funds. The
Office of the Special Prosecutor (OSP) found probable cause to indict respondent Amante and thus
on May 21, 2004, the Office of the Special Prosecutor (OSP) filed an Information with the
Sandiganbayan accusing Victoria Amante of violating Section 89 of P.D. No. 1445 (The Auditing
Code of the Philippines).
Respondent Amante in her MOTION TO DEFER ARRAIGNMENT AND MOTION FOR
REINVESTIGATION dated November 18, 2004 stated that the Sandiganbayan had no jurisdiction
over the said criminal case because respondent Amante was then a local official who was
occupying a position of salary grade 26, whereas Section 4 of Republic Act (R.A.) No. 8249
provides that the Sandiganbayan shall have original jurisdiction only in cases where the accused
holds a position otherwise classified as Grade 27 and higher, of the Compensation and Position
Classification Act of 1989, R.A. No. 6758.
The Sandiganbayan, in its Resolution dated February 28, 2005, dismissed the case against Amante
for lack of jurisdiction. The dismissal, however, is without prejudice to the filing of this case to
the proper court.
Issue/s:
Whether or not a member of the Sangguniang Panlungsod under Salary Grade 26 who was charged
with violation of The Auditing Code of the Philippines falls within the jurisdiction of the
Sandiganbayan.
Ruling:
The present case falls under P.D. No. 1606 as amended by R.A. No. 8249. Under Section 4(a) of
said law, the following offenses are specifically enumerated: violations of R.A. No. 3019, as
amended, R.A. No. 1379, and Chapter II, Section 2, Title VII of the Revised Penal Code. In order
for the Sandiganbayan to acquire jurisdiction over the said offenses, the latter must be committed
by, among others, officials of the executive branch occupying positions of regional director and
higher, otherwise classified as Grade 27 and higher, of the Compensation and Position
Classification Act of 1989. However, the law is not devoid of exceptions. Those that are classified
as Grade 26 and below may still fall within the jurisdiction of the Sandiganbayan provided that
they hold the positions thus enumerated by the same law. Particularly and exclusively enumerated
are provincial governors, vice-governors, members of the Sangguniang Panlalawigan, and
provincial treasurers, assessors, engineers, and other provincial department heads; city mayors,
vice-mayors, members of the Sangguniang Panlungsod, city treasurers, assessors, engineers ,
and other city department heads; officials of the diplomatic service occupying the position as
consul and higher; Philippine army and air force colonels, naval captains, and all officers of higher
rank; PNP chief superintendent and PNP officers of higher rank; City and provincial prosecutors
and their assistants, and officials and prosecutors in the Office of the Ombudsman and special
prosecutor; and presidents, directors or trustees, or managers of government-owned or controlled
corporations, state universities or educational institutions or foundations. In connection therewith,
Section 4(b) of the same law provides that other offenses or felonies committed by public officials
and employees mentioned in subsection (a) in relation to their office also fall under the jurisdiction
of the Sandiganbayan.

By simple analogy, applying the provisions of the pertinent law, respondent Amante, being a
member of the Sangguniang Panlungsod at the time of the alleged commission of an offense in
relation to her office, falls within the original jurisdiction of the Sandiganbayan. The provision of
the law shows that those public officials enumerated in Section 4(a) of P.D. No. 1606, as amended,
may not only be charged in the Sandiganbayan with violations of R.A. No. 3019, R.A. No. 1379
or Chapter II, Section 2, Title VII of the Revised Penal Code, but also with other offenses or
felonies in relation to their office. The said other offenses and felonies are broad in scope but are
limited only to those that are committed in relation to the public official or employee's office.

In the offenses involved in Section 4(a), public office is essential as an element of the said offenses
themselves, while in those offenses and felonies involved in Section 4(b), it is enough that the said
offenses and felonies were committed in relation to the public officials or employees' office.
Moreover, Section 4(b) does not mention any qualification as to the public officials involved. It
simply stated, public officials and employees mentioned in subsection (a) of the same section.
Therefore, it refers to those public officials with Salary Grade 27 and above, except those
specifically enumerated. It is a well-settled principle of legal hermeneutics that words of a statute
will be interpreted in their natural, plain and ordinary acceptation and signification, unless it is
evident that the legislature intended a technical or special legal meaning to those words.

The Petition was GRANTED and the Resolution of the Sandiganbayan (Third Division)
NULLIFIED and SET ASIDE. Consequently, case was REMANDED to the Sandiganbayan for
further proceedings.
HANNAH EUNICE D. SERANA vs. SANDIGANBAYAN and PEOPLE OF THE
PHILIPPINES
[G.R. No. 162059, January 22, 2008, REYES, R.T.]

CAN the Sandiganbayan try a government scholar accused, along with her brother, of swindling government funds?

FACTS:
1. Petitioner Hannah Eunice D. Serana was a senior student of the University of the Philippines-Cebu.
She was appointed by then President Joseph Estrada on December 21, 1999 as a student regent of
UP, to serve a one-year term starting January 1, 2000 and ending on December 31, 2000.
2. In the early part of 2000, petitioner discussed with President Estrada the renovation of Vinzons Hall
Annex in UP Diliman.
3. On September 4, 2000, petitioner, with her siblings and relatives, registered with the Securities and
Exchange Commission the Office of the Student Regent Foundation, Inc. (OSRFI).
4. One of the projects of the OSRFI was the renovation of the Vinzons Hall Annex. President Estrada
gave Fifteen Million Pesos (P15,000,000.00) to the OSRFI as financial assistance. The source of the
funds, according to the information, was the Office of the President.
5. The renovation of Vinzons Hall Annex failed to materialize. The succeeding student regent, Kristine
Clare Bugayong, and Christine Jill De Guzman, Secretary General of the KASAMA sa U.P., a
system-wide alliance of student councils within the state university, consequently filed a complaint
for Malversation of Public Funds and Property with the Office of the Ombudsman.
6. On July 3, 2003, the Ombudsman found probable cause to indict petitioner and her brother Jade Ian
D. Serana for estafa at the Sandiganbayan. The Information reads:

“xxx wilfully, unlawfully and feloniously defraud the government by falsely and fraudulently representing to former
President Joseph Ejercito Estrada that the renovation of the Vinzons Hall of the University of the Philippines
will be renovated and renamed as "President Joseph Ejercito Estrada Student Hall," and for which purpose
accused HANNAH EUNICE D. SERANA requested the amount of FIFTEEN MILLION PESOS
(P15,000,000.00), Philippine Currency, from the Office of the President, and the latter relying and believing
on said false pretenses and misrepresentation gave and delivered to said accused Land Bank Check No. 91353
dated October 24, 2000 in the amount of FIFTEEN MILLION PESOS (P15,000,000.00), which check was
subsequently encashed by accused Jade Ian D. Serana on October 25, 2000 and misappropriated for their
personal use and benefit, and despite repeated demands made upon the accused for them to return aforesaid
amount, the said accused failed and refused to do so to the damage and prejudice of the government in the
aforesaid amount. CONTRARY TO LAW. (Underscoring supplied)

7. Petitioner moved to quash the information.


a. Sandiganbayan does not have any jurisdiction over the offense charged or over her person, in
her capacity as UP student regent. RA 3019, as amended by R.A. No. 8249, enumerates the crimes or
offenses over which the Sandiganbayan has jurisdiction. It has no jurisdiction over the crime of estafa. It only
has jurisdiction over Crimes Committed by Public Officers. Estafa falls under Crimes Against Property, and is
not within the Sandiganbayan’s jurisdiction.
b. It was Estrada, and not the government, that was duped. Even assuming that she received
the P15,000,000.00, that amount came from Estrada, not from the coffers of the government.
c. The Sandiganbayan had no jurisdiction over her person. As a student regent, she was not a
public officer since she merely represented her peers, in contrast to the other regents who held their positions
in an ex officio capacity. She addsed that she was a simple student and did not receive any salary as a student
regent.
d. She had no power or authority to receive monies or funds. Such power was vested with the
Board of Regents (BOR) as a whole. Since it was not alleged in the information that it was among her
functions or duties to receive funds, or that the crime was committed in connection with her official
functions, the same is beyond the jurisdiction of the Sandiganbayan citing the case of Soller v. Sandiganbayan. 11

8. The Ombudsman opposed the motion.


. Section 4(b) of Presidential Decree (P.D.) No. 1606 clearly contains the catch -all phrase "in relation to
office," thus, the Sandiganbayan has jurisdiction over the charges against petitioner.
a. The source of the money is a matter of defense. It should be threshed out during a full-blown trial.
b. Petitioner, despite her protestations, was a public officer. As a member of the BOR, she has the
general powers of administration and exercises the corporate powers of UP. Based on Mechem’s definition of
a public office, petitioner’s stance that she was not compensated, hence, not a public officer, is erroneous.
Compensation is not an essential part of public office. Parenthetically, compensation has been interpreted to
include allowances. By this definition, petitioner was compensated.

Sandiganbayan Disposition: Denied petitioner’s motion for lack of merit


1. Section 4(b) of R.A. 8249 provides that the Sandiganbayan also has jurisdiction over other offenses
committed by public officials and employees in relation to their office. From this provision, there is
no single doubt that this Court has jurisdiction over the offense of estafa committed by a public
official in relation to his office.
2. Section 4 of Republic Act No. 8249 which provides:

Sec. 4. Jurisdiction – The Sandiganbayan shall exercise exclusive original jurisdiction in all cases involving:
(A) x x x
(1) Officials of the executive branch occupying the positions of regional director and higher, otherwise classified as Grade "27"
and higher, of the Compensation and Position Classification Act of 1989 (Republic Act No. 6758), specifically
including:
xxxx
(g) Presidents, directors or trustees, or managers of government-owned or controlled corporations, state universities or educational institutions or
foundations. (Italics supplied)

It is very clear from the aforequoted provision that the Sandiganbayan has original exclusive jurisdiction over all
offenses involving the officials enumerated in subsection (g), irrespective of their salary grades, because the
primordial consideration in the inclusion of these officials is the nature of their responsibilities and functions.

3. Is accused-movant included in the contemplated provision of law? It is unmistakably evident that the
Board of Regents of the University of the Philippines is performing functions similar to those of the Board of
Trustees of a non-stock corporation. This draws to fore the conclusion that being a member of such board,
accused-movant undoubtedly falls within the category of public officials upon whom this Court is vested with
original exclusive jurisdiction, regardless of the fact that she does not occupy a position classified as Salary
Grade 27 or higher under the Compensation and Position Classification Act of 1989.
4. Finally, this court finds that accused-movant’s contention that the same of P15 Million was received
from former President Estrada and not from the coffers of the government, is a matter a defense that should
be properly ventilated during the trial on the merits of this case.

ISSUE: W/n the Sandiganbayan has jurisdiction over the UP Student Regent

Ruling: Yes.

1. It is P.D. No. 1606, as amended, rather than R.A. No. 3019, as amended, that determines the
jurisdiction of the Sandiganbayan.
2. Section 4(B) of P.D. No. 1606 reads: “B. Other offenses or felonies whether simple or complexed
with other crimes committed by the public officials and employees mentioned in subsection (a0 of
this section in relation to their office.” Evidently, the Sandiganbayan has jurisdiction over other
felonies committed by public officials in relation to their office. We see no plausible or sensible
reason to exclude estafa as one of the offenses included in Section 4(bB) of P.D. No. 1606.
3. The jurisdiction is simply subject to the twin requirements that (a) the offense is committed by public
officials and employees mentioned in Section 4(A) of P.D. No. 1606, as amended, and that (b) the
offense is committed in relation to their office.
4. It is not only the salary grade that determines the jurisdiction of the Sandiganbayan. The
Sandiganbayan also has jurisdiction over other officers enumerated in P.D. No. 1606. Section
4(A)(1)(g) of P.D. No. 1606 explictly vested the Sandiganbayan with jurisdiction over Presidents,
directors or trustees, or managers of government-owned or controlled corporations, state universities
or educational institutions or foundations. Petitioner falls under this category. As the Sandiganbayan
pointed out, the BOR performs functions similar to those of a board of trustees of a non-stock
corporation. By express mandate of law, petitioner is, indeed, a public officer as contemplated by
P.D. No. 1606.
5. In the case at bench, the information alleged, in no uncertain terms that petitioner, being then a
student regent of U.P., "while in the performance of her official functions, committing the offense in
relation to her office and taking advantage of her position, with intent to gain, conspiring with her
brother, JADE IAN D. SERANA, a private individual, did then and there wilfully, unlawfully and
feloniously defraud the government x x x."

MAJOR GENERAL CARLOS F. GARCIA, AFP (RET.), Petitioner, versus

SANDIGANBAYAN and the REPUBLIC OF THE PHILIPPINES, Respondents.

FACTS

Gen. Garcia was charged with violation of RA 6713 (Ethical Standards for Public
Employees), Art 183 of RPC (Perjury), and violation of Civil Service Law. His wife, Clarita Garcia, and
their three sons, Ian Carl, Juan Paolo and Timothy Mark, all surnamed Garcia, were impleaded in the
complaint for violation of R.A. No. 1379 insofar as they acted as conspirators, conduits, dummies and
fronts in receiving, accumulating, using and disposing of his ill-gotten wealth.

Office of the Ombudsman, filed before the Sandiganbayan, a Petition with Verified Urgent Ex
Parte Application for the Issuance of a Writ of Preliminary Attachment against Garcia, his wife, and
three sons, seeking the forfeiture of unlawfully acquired properties under Sec. 2 of R.A. No. 1379. The
Office of the Ombudsman, after conducting an inquiry similar to a preliminary investigation, has
determined that a prima facie case exists Gen. Garcia and family who hold such properties for, with,
or on behalf of, Gen. Garcia, since during his incumbency as a soldier and public officer he acquired
huge amounts of money and properties manifestly out of proportion to his salary as such public officer
and his other lawful income, if any.

The writ of preliminary attachment was subsequently issued on 2 November 2004 upon the filing of a
bond by the Republic. On 17 November 2004, Garcia filed a Motion to Dismiss on the ground of lack
of jurisdiction of the Sandiganbayan over forfeiture proceedings under R.A. No. 1379.

Garcia argued that Sandiganbayan, under P.D. No. 1606 (law creating it), was intended principally as
a criminal court, with no jurisdiction over separate civil actions

ISSUE
W/N Sandiganbayan has Jurisdiction over forfeiture cases which are civil proceedings.
RULING (DISMISSED)
Yes, Sandiganbayan has Jurisdiction over the case.

( LAW) Under R.A. No. 8249(Defines JD of SB),

the Sandiganbayan is vested with exclusive original jurisdiction in all cases involving violations of R.A.
No. 3019, R.A. No. 1379, and Chapter II, Sec. 2, Title VII, Book II of the Revised Penal Code, where
one or more of the accused are officials occupying the following positions whether in a permanent,
acting or interim capacity, at the time of the commission of the offense:

(d) Philippine army and air force colonels, naval captains, and all officers of higher rank;

(JURISPRUDENCE) Forfeiture is Penal in nature.


Cabal v. Kapunan, Jr.
In a strict signification, a forfeiture is a divestiture of property without compensation, in consequence
of a default or an offense, and the term is used in such a sense in this article. A forfeiture, as thus
defined, is imposed by way of punishment not by the mere convention of the parties, but by the
lawmaking power, to insure a prescribed course of conduct. It is a method deemed necessary by the
legislature to restrain the commission of an offense and to aid in the prevention of such an offense.
The effect of such a forfeiture is to transfer the title to the specific thing from the owner to the sovereign
power.

Republic v. Agoncillo and Katigbak v. Solicitor General.


Section 12 of the law provides a penalty but it is only imposed upon the public officer or employee who
transfers or conveys the unlawfully acquired property, it imposes the penalty of forfeiture of the
properties unlawfully acquired upon the respondent public officer or employee.
violations of R.A. No. 1379 are placed under the jurisdiction of the Sandiganbayan, even though the
proceeding is civil in nature, since the forfeiture of the illegally acquired property amounts to a penalty.
The soundness of this reasoning becomes even more obvious when we consider that the respondent
in such forfeiture proceedings is a public officer or employee and the violation of R.A. No. 1379 was
committed during the respondent officer or employee’s incumbency and in relation to his office. This
is in line with the purpose behind the creation of the Sandiganbayan as an anti-graft court—to address
the urgent problem of dishonesty in public service.

People vs. Magallanes

Facts: In 1994, two informations for kidnapping for ransom with murder were filed with
the RTC of Bacolod City against 14 persons, 5 of whom are members of the PNP. Each
of the accused pleaded not guilty upon arraignment. After, the prosecution rested its case,
the trial court started to receive the evidence for the accused.

Later, the private prosecutors moved for the transmittal of the records of the cases to the
Sandiganbayan on the ground that the trial court has no jurisdiction over the cases
because the offenses charged were committed in relation to the office of the accused
PNP officers.

The trial court, thru respondent Judge, ruled that the Sandiganbayan does not have
jurisdiction over the subject cases because the informations do not state that the offenses
were committed in relation to the office of the accused PNP officers. It held that the
allegation in the informations that the accused PNP officers took advantage of their office
in the commission of the offense charged is merely an allegation of an aggravating
circumstance. It further stated that a public office is not a constituent element of the
offense of kidnapping with murder nor is the said offense intimately connected with the
office. It then denied the motion for transfer of the records to the Sandiganbayan and
declared that the trial of the case should continue.

The prosecution filed a petition for certiorari, prohibition and mandamus with a prayer for
a temporary restraining order, challenging the refusal of the judge to transfer the cases
to the Sandiganbayan. The petitioner submits that the crimes charged in the subject
cases were connected with public office because the accused PNP officers, together with
the civilian agents, arrested the two swindling suspects in the course of the performance
of their duty and not out of personal motive, and if they demanded from the two suspects
the production of the money of the Dumancas spouses and later killed the two; they did
so in the course of the investigation conducted by them as policemen. The petitioner
further asserts that the allegations in the informations reading "taking advantage of his
position as Station Commander of the Philippine National Police" and "taking advantage
of their respective positions" presuppose the exercise of the functions attached to the
office of the accused PNP officers and are sufficient to show that the offenses charged
were committed in relation to their office.

Issues:
1.Who has jurisdiction over the cases?

2. Were the offense committed in relation to the office of the accused PNP officers?

Held:

1. The jurisdiction of a court may be determined by the law in force at the time of the
commencement of the action. When the informations in the cases were filed, the law
governing the jurisdiction of the Sandiganbayan was P.D. 1861, which provides that the
Sandiganbayan shall have exclusive original jurisdiction over cases involving: 1)
violations of the Anti-Graft and Corrupt Practices Act; 2) offenses committed by public
officers in relation to their office, where the penalty prescribed is higher than prision
correccional or imprisonment of six (6) years, or a fine of P 6,000.00. If the penalty for the
offense charged does not exceed imprisonment of six (6) years or a fine of P6,000.00, it
shall be tried by the Regional Trial Court, Metropolitan Trial Court, Municipal Trial Court
or the Municipal Circuit Trial Court.

2. It is an elementary rule that jurisdiction is determined by the allegations in the complaint


or information, and not by the result of evidence after trial.

The informations in the court below do not indicate that the accused arrested and
investigated the victims and then killed the latter in the course of the investigation. The
informations merely allege that the accused, for the purpose of extracting or extorting the
sum of P353,000.00, abducted, kidnapped, and detained the two victims, and failing in
their common purpose, they shot and killed the said victims. For the purpose of
determining jurisdiction, it is these allegations that shall control, and not the evidence
presented by the prosecution at the trial.

The allegation of "taking advantage of his position" or "taking advantage of their


respective positions" incorporated in the informations is not sufficient to bring the offenses
within the definition of "offenses committed in relation to public office." In Montilla vs.
Hilario, such an allegation was considered merely as an allegation of an aggravating
circumstance, and not as one that qualifies the crime as having been committed in relation
to public office.

Also, in Bartolome vs. People of the Philippines, despite the allegation that the accused
public officers committed the crime of falsification of official document by "taking
advantage of their official positions," this Court held that the Sandiganbayan had no
jurisdiction over the case because "[t]he information [did] not allege that there was an
intimate connection between the discharge of official duties and the commission of the
offense."

Accordingly, for lack of an allegation in the informations that the offenses were committed
in relation to the office of the accused PNP officers or were intimately connected with the
discharge of the functions of the accused, the subject cases come within the jurisdiction
of the Regional Trial Court and not of the Sandiganbayan as insisted by the petitioner.

Moreover, the Sandiganbayan has partly lost its jurisdiction over cases involving
violations of R.A. 3019, as amended in R.A. 1379 because it only retains jurisdiction on
cases enumerated in subsection (a) when the public officers rank is classified as Grade
“27” or higher. In the case at bar, none of the PNP officers involved occupy a position
classified as Grade “27” or higher.

Lastly, the courts cannot be divested of jurisdiction which was already acquired before
the subsequent enactment of R.A. 7975 which limited the Sandiganbayan‟s jurisdiction
to officers whose rank is Grade“27” or higher. Jurisdiction once acquired is not affected
by subsequent legislative enactment placing jurisdiction in another tribunal. It remains
with the court until the case is finally terminated. Hence, cases already under the
jurisdiction of the courts at the time of the enactment of R.A. 7975 are only referred to the
proper courts if trial has not yet begun at that time.

G.R. No. Nos. 111771-77 November 9, 1993

ANTONIO L. SANCHEZ, Petitioner, vs. The Honorable HARRIET O. DEMETRIOU (in her
capacity as Presiding Judge of Regional Trial Court, NCR, Branch 70, Pasig), et al.

Facts:
• Seven informations were filed before RTC Calamba against the Mayor of Calamba,
Laguna (Antonio Sanchez) and six others for the rape-slay of Mary Eileen Sarmenta and
the killing of Alan Gomez. However, Secretary of Justice expressed apprehension that
the trial of said cases might result in miscarriage of justice due to the partisan
atmosphere in Calamba. Court thereupon ordered transfer of the venue of seven cases
to RTC Pasig which where raffled to herein respondent Judge Demetriou.
• Petitioner filed a motion to quash the information on several grounds one of which is that
as a public officer, Sanchez can be tried for the offense only by the Sandiganbayan.
• Respondent Judge denied the motion. Hence, this petition for certiorari and prohibition
with prayer of TRO/writ of injunction.
ISSUE:
Whether or not petitioner Sanchez, as a public officer can be tried for the offense only by the
Sandiganbayan.
RULING:
• No, Sandiganbayan has no jurisdiction over the said case. Section 4, paragraph (a) of
P.D. No, 1606, as amended by P.D. No.1861 (Amendment Relative To The Jurisdiction
Of The Sandiganbayan And For Other Purposes), provides that Sandiganbayan has
exclusive original jurisdiction in all cases involving:
• (1) Violations of Ant-Graft and Corrupt Practices Act (RA 3019), RA 1379 (An
Act Declaring Forfeiture In Favor Of The State Any Property Found To Have
Been Unlawfully Acquired By Any Public Officer Or Employee And Providing For
The Proceedings Therefor), and Chapter II Section2, Title VII of the RPC (Crimes
Committed by Public Officers);
• (2) Other offenses or felonies committed by public officers and employees in
relation to their office, including those employed in government-owned or
controlled corporations, whether simple or complexed with other crimes, where
the penalty prescribed by law is higher than prision correccional or imprisonment
for six (6) years, or a fine of P6,000.00
• In this case, the accused was charged with the crime of rape with homicide which does
not fall under paragraph (1). Neither it is covered by paragraph (2) because it is not an
offense committed in relation to the office of the petitioner.
• There is no direct relation between the commission of the crime of rape with homicide
and the petitioner's office as municipal mayor because public office is not an essential
element of the crime charged. The offense can stand independently of the office.
Moreover, it is not even alleged in the information that the commission of the crime
charged was intimately connected with the performance of the petitioner's official
functions.
• It follows that the said crime, being an ordinary offense, is triable by the regular courts
and not the Sandiganbayan.
DP: Wherefore, petition is DISMISSED. The respondent judge is DIRECTED to continue with the
trial of Criminal Cases

UY VS SB
(Unavailable)
BARTOLOME V. PEOPLE
G.R. NO. L- 64548 and G.R. No. L-64559, 7 July 1986

FACTS

Sandiganbayan convicted the petitioners (Bartolome and Santos) of the crime of falsification of a
public document. The Information alleged that Bartolome, in conspiration with Santos, falsified his
CS Personal Data Sheet by making it appear that Bartolome passed the Career Service
Professional Qualifying Examination and that he was a 4 year AB student at FEU.
th

ISSUE
Whether or not Sandiganbayan has jurisdiction over the subject matter

RULING
No, Sandiganbayan has no jurisdiction over the subject matter of the case.

Under Section 4 of PD 1606, the Sandiganbayan has jurisdiction over


a. Violations of Republic Act No. 3019, as amended, otherwise known as the Anti-Graft and
Corrupt Practices Act, and Republic Act No. 1379;
b. Crime committed by public officers and employees, including those employed in
government-owned or controlled corporations, embraced in Title VII of the Revised Penal Code,
whether simple or complexed with other crimes; and
c. Other crimes or offenses committed by public officers or employees, including those
employed in government-owned or controlled corporations, in relation to their office.

In this case, the petitioners were charged of falsification of public documents, which is punishable
under Article 171, Title IV, Book Two on Crimes against Public Interest. Neither does
Sandiganbayan has jurisdiction pursuant to Section 4(c) as crime charged was not committed in
relation to the office of the accused. In order to qualify as crime “committed in relation to the
office”, it is settled that the offense charged must be intimately connected with their respective
offices and was perpetrated while they were in the performance of their official functions. Such
fact was not alleged in the Information. Also, “public office is not an essential ingredient of the
crim charged such that the offense can exist even without the office”.

As the alleged falsification was not punishable under Title VII of the RTPC, nor was it committed
in relation to the office of the accused, the Sandiganbayan has no jurisdiction over the case.

FERDINAND CUNANAN, petitioner, vs.HON. HERMIN E. ARCEO, as Presiding Judge of Branch 43 of the Regional
Trial Court of San Fernando, Pampanga; HON. PEDRO M. SUNGA, JR., as Presiding Judge of the Regional Trial
Court of Branch 42 of San Fernando, Pampanga; and THE PEOPLE OF THE PHILIPPINES, respondents.
G.R. No. 116615 March 1, 1995

Facts:
An information for Murder was filed against petitioner Ferdinand Cunanan before Branch 46 of the RTC of San
Fernando, Pampanga, presided over by Judge Norberto C. Ponce. The Information alleged that petitioner was a
member of the Philippine National Police; it contained no averment that he had committed the offense charged in
relation to his public office. Petitioner entered a plea of not guilty at arraignment and trial proceeded thereafter.
The parties having presented their evidence, Judge Arceo required them to submit memoranda, after which the case
was deemed submitted for decision.

The Supreme Court promulgated its En Banc Decision in Republic v. Hon. Asuncion, et al., which laid down the rule
that the Sandiganbayan has exclusive and original jurisdiction to take cognizance of offenses committed by public
officers in relation to their office, where the penalty prescribed by law is higher than prision correccional or
imprisonment of six (6) years or more or a fine of P6,000.00. The Supreme Court further held that in the event an
Information failed to allege that the accused-public officer had committed the offense charged in relation to his
office, the RTC hearing the criminal case, pending at the time of the promulgation of the Asuncion rule, shall conduct
a preliminary hearing to determine the existence or absence of this material fact. If this material fact is found to be
present, the RTC shall order the transfer of the case to the Sandiganbayan for docketing, and the latter shall proceed
to hear the case as if the same had been originally instituted with it. If it be determined that that fact is absent, the
RTC seized with the case shall proceed with the trial and render judgment on the case.

Judge Arceo proceeded to apply these holdings by conducting a hearing solely to ascertain if petitioner had
committed the offense charged in relation to his office. He then ruled that the petitioner had committed the offense
charged in relation to his office, hence he ordered to forward to the Sandiganbayan the case of petitioner and the
complete records of the same transmitted therewith as if it was originally filed with the said Court on the ground of
lack of jurisdiction.

Judge Arceo inhibited himself from further hearing the case. The case was then raffled and was presided by co-public
respondent Judge Pedro M. Sunga Jr., who denied, among other things, petitioner's Opposition to the Order
directing the transmittal of the records of his (petitioner's) case to the Sandiganbayan. Hence, this petition.

Petitioner contends that the Asuncion ruling is inapplicable to the present case, since trial had already ended and
the case was already submitted for decision when the Asuncion ruling was promulgated. A transfer of his case to the
Sandiganbayan at this late stage will, accordingly, expose him (petitioner) to double jeopardy of punishment for the
same offense. Petitioner believes Judge Arceo's Order dismissing the case for lack of jurisdiction over the offense
charged amounts to an acquittal of petitioner.

Issue:
Whether or not the public respondent RTC judges had correctly applied the doctrine laid down in Asuncion to this
case, considering that here the absence of jurisdiction on the part of the RTC became apparent to the RTC after
completion of the trial and submission of the case for decision.

Ruling:
Yes, the SC held that the public respondent RTC judges had correctly ruled that they have no jurisdiction on the
petitioner’s case.

To determine whether an offense falls under the exclusive jurisdiction of the Sandiganbayan, two requisites must
concur: (a) the offense must have been committed by the accused public officer in relation to his office; and (b) the
penalty prescribed for the offense charged is higher thanprision correccional or imprisonment for six (6) years or a
fine of Six Thousand Pesos (P6,000.00).

Whenever the above two (2) requisites are present, jurisdiction over the offense is vested in the Sandiganbayan
even though the information originally filed before the RTC did not aver that the accused public officer had
committed the offense charged in relation to his office. Hence, the absence in the old information filed before the
RTC of an allegation that petitioner Cunanan had committed the offense charged in relation to his office, is
immaterial insofar as determination of the locus of jurisdiction is concerned.
The RTC's initial assumption of jurisdiction over the offense charged in this case did not, therefore, prevent it from
subsequently declaring itself to be without jurisdiction, that lack of jurisdiction having become apparent from
subsequent proceedings in that case. Here, the RTC found after a hearing that petitioner had committed the offense
charged while he was in the performance of his duties as a policeman. It is clear that this Court is bereft of any
jurisdiction to try and decide this case and any decision that may be rendered maybe validly assailed as null and void
for want of jurisdiction.

The absence in the information filed before Branch 46 of the RTC of San Fernando, Pampanga, of an allegation that
petitioner had committed the offense charged in relation to his office, is immaterial and easily remedied. That
information maybe amended at any time before arraignment before the Sandiganbayan, and indeed, by leave of
court at any time before judgment is rendered by the Sandiganbayan, considering that such an amendment would
not affect the juridical nature of the offense charged (i.e., murder), the qualifying circumstances alleged in the
information, or the defenses that petitioner may assert before the Sandiganbayan. The amendment may be made
before the Sandiganbayan without surprising the petitioner or prejudicing his substantive rights.

Petitioner had not been exposed at all to legal jeopardy by the commencement and trial because the RTC was not a
court of competent jurisdiction to try the case in the first place. The dismissal of the Information by the RTC was not
equivalent to, and did not operate as an acquittal of petitioner of that offense. The "dismissal" (later deleted by the
RTC) had simply reflected the fact that the proceedings before the RTC were terminated, the RTC having ascertained
that it had no jurisdiction to try the case at all.

WHEREFORE, the Petition for Certiorari is hereby DISMISSED for lack of merit. The Orders of the public respondent
RTC Judges are hereby AFFIRMED.

LACSON vs. EXECUTIVE SECRETARY


(G.R. No. 128096, January 20, 1999)

FACTS:
On 18 May 1995, 11 members of the Kuratong Baleleng gang, were killed along Commonwealth Ave., QC by the
Anti-Bank Robbery and Intelligence Task Group (ABRITG) headed by Chief Supt. Jewel Canson of PNP. One of the
components of ABRITG is the Presidential Anti-Crime Commission-Task Force Habagat (PACC-TFH) headed by
petitioner Chief Supt. Panfilo Lacson.
SPO2 Eduardo delos Reyes told the media that what happened was a summary execution (or a rub out) and not a
shoot-out between the Kuratong Baleleng gang members and the ABRITG.
Omb. Aniano Desierto formed a panel of investigators headed by the Deputy Omb. for Military Affairs, Bienvenido
Blancaflor, to investigate the incident. This panel found that the incident was a legitimate police operation. However,
a review board led by Overall Deputy Omb. Francisco Villa recommended the indictment for multiple murder against
26 respondents, including Lacson.
Lacson was among those charged as principal in information for murder before the Sandiganbayan. All the accused
filed separate motions questioning its jurisdiction.
Sandiganbayan ordered the cases transferred to the QC RTC which has original and exclusive jurisdiction under RA
7975 (An Act to Strengthen the Functional and Structural Organization of the Sandiganbayan, Amending for that
Purpose PD 1606, as Amended), as none of the principal accused has the rank of PNP Chief Supt. or higher .The
Office of the Special Prosecutor moved for a reconsideration. While it is pending, RA 8249 (An Act Further Defining
thr Jurisdiction of the Sandiganbayan, Amending for the Purpose PD 1606, as Amended, Providing Funds Therefor,
and for Other Purposes) was enacted to law which deletes the word “principal” from the phrase “principal accused”.
Thus, the Sandiganbayan takes cognizance again of the case
ISSUES:
Whether or not Sandiganbayan has jurisdiction. (NO)
HELD: Sandiganbayan has no jurisdiction.
It’s QC RTC that has jurisdiction. The jurisdiction of a court is defined by the Constitution or statute. The elements
of that definition must appear in the complaint or information so as to ascertain which court has jurisdiction over a
case. Hence the elementary rule that the jurisdiction of a court is determined by the allegations in the complaint or
information,and not by the evidence presented by the parties at the trial.
The multiple murder charge falls under Section 4(b) of R.A. 8249, which requires that the offense charged must be
committed by the offender in relation to his office in order for the Sandiganbayan to have jurisdiction over it.
An offense is said to have been committed in relation to the office if it (the offense) is ‘intimately connected’ with the
office of the offender and perpetrated while he was in the performance of his official functions.
While the information states that Lacson, et al committed the crime of murder in relation to their public office, there
is, however, no specific allegation of facts that the shooting of the victim by the said principal accused was intimately
related to the discharge of their official duties as police officers. Likewise, the amended information does NOT indicate
that the said accused arrested and investigated the victim and then killed the latter while in their custody.
While there is the allegation in the amended information that the said accessories committed the offense “in relation
to office as officers and members of the (PNP),” the Court do not see the intimate connection between the offense
charged and the accused’s official functions. That phrase is merely a conclusion between of law, not a factual averment
that would show the close intimacy between the offense charged and the discharge of the accused’s official duties.
What is controlling is the specific factual allegations in the information that would indicate the close intimacy between
the discharge of the accused’s official duties and the commission of the offense charged, in order to qualify the crime
as having been committed in relation to public office.

Pp. vs.ULYSSES M. CAWALING, et al. (G.R. No. 117970 July 28, 1998)

Facts:
Second Assistant Provincial Fiscal Alexander Mortel filed, before the RTC of Odiongan,
Romblon, an Information for murder against former Mayor Ulysses M. Cawaling and Policemen
Ernesto Tumbagahan, Ricardo De los Santos and Hilario Cajilo. Thereafter, a decision of the
RTC Romblon was promulgated finding the accused guilty of murder.
However the appellants argue that the trial court erred when it assumed jurisdiction over the
criminal case. They insist that the Sandiganbayan, not the regular courts, had jurisdiction to try
and hear the case against the appellants, as they were public officers at the time of the killing
which was allegedly committed by reason of or in relation to their office.
Issue: WON the regional trial court, not the Sandiganbayan, has jurisdiction over the case
committed by public officers including a town mayor.
Ruling:
Yes, RTC has jurisdiction over the case.
In order for the Sandiganbayan to exercise exclusive and original jurisdiction over a case, two
requisites must concur: (a) the offense was committed by the accused public officer in relation to
his office; and (b) the penalty prescribed by law is higher than prision correccional or
imprisonment for six (6) years, or higher than a fine of six thousand pesos.
Furthermore, the relation between the crime and the office must be direct and not accidental; that
the offense cannot exist without the office. In other words, the office must be a constituent
element of the crime.
The Information filed against the appellants contains no allegation that appellants were public
officers who committed the crime in relation to their office. In the absence of such essential
allegation, the Sandiganbayan does not have jurisdiction over the present case. Even before
considering the penalty prescribed by law for the offense charged, it is thus essential to
determine whether that offense was committed or alleged to have been committed by the public
officers and employees in relation to their offices.
Jurisdiction is determined by the allegations in the complaint or information. In the absence of
any allegation that the offense was committed in relation to the office of appellants or was
necessarily connected with the discharge of their functions, the regional trial court, not the
Sandiganbayan, has jurisdiction to hear and decide the case.
______________________________________________________________________________

Sec. 4. Jurisdiction. — The Sandiganbayan shall exercise original jurisdiction in all cases involving:

a. Violations of Republic Act No. 3019, as amended, otherwise known as the Anti-Graft and Corrupt Practices Act, Republic Act
No. 1379, and Chapter II, Section 2, Title VII of the Revised Penal Code, where one or more of the principal accused are officials
occupying the following positions in the government, whether in a permanent, acting or interim capacity, at the time of the
commission of the offense:

(1) Officials of the executive branch occupying the positions of regional director and higher, otherwise classified as grade 27 and
higher, of the Compensation and Position Classification Act of 1989 (Republic Act No. 6758), specifically including:

(a) Provincial governors, vice-governors, members of the sanggunian panlalawigan, and provincial treasurers, assessors,
engineers, and other provincial department heads;

(b) City mayors, vice-mayors, members of the sangguniang panlungsod, city treasurers, assessors, engineers, and other city
department heads;

(c) Officials of the diplomatic service occupying the position of consul and higher;

(d) Philippine army and air force colonels, naval captains, and all officers of higher rank;

(a) PNP chief superintendent and PNP officers of higher rank;

(f) City and provincial prosecutors and their assistants and officials and prosecutors in the Office of the Ombudsman and special
prosecutor;

(g) Presidents, directors or trustees, or managers of government-owned or -controlled corporations, state universities or
educational institutions or foundations;

(2) Members of Congress and officials thereof classified as Grade "27" and up under the Compensation and Position
Classification Act of 1989;

(3) Members of the judiciary without prejudice to the provisions of the Constitution;

(4) Chairmen and members of Constitutional Commissions, without prejudice to the provisions of the Constitution; and

(5) All other national and local officials classified as Grade "27" and higher under the Compensation and Position Classification
Act of 1989.

b. Other offenses or felonies committed by the public officials and employees mentioned in subsection (a) of this section in
relation to their office.

c. Civil and criminal cases filed pursuant to and in connection with Executive Order Nos. 1, 2, 14 and 14-A.

In cases where none of the principal accused are occupying the positions corresponding to salary grade "27" or higher, as
prescribed in the said Republic Act No. 6758, or PNP officers occupying the rank of superintendent or higher, or their
equivalent, exclusive jurisdiction thereof shall be vested in the proper Regional Trial Court, Metropolitan Trial Court, Municipal
Trial Court, and Municipal Circuit Trial Court, as the case may be, pursuant to their respective jurisdictions as provided in Batas
Pambansa Blg. 129.

The Sandiganbayan shall exercise exclusive appellate jurisdiction on appeals from the final judgments, resolutions or orders of
regular courts where all the accused are occupying positions lower than grade "27," or not otherwise covered by the preceding
enumeration.

xxx xxx xxx

In case private individuals are charged as co-principals, accomplices or accessories with the public officers or employees,
including those employed in government-owned or — controlled corporations, they shall be tried jointly with said public officers
and employees in the proper courts which shall have exclusive jurisdiction over them. (emphasis supplied).

TATAD VS SB
(UNAVAILABLE)

PEOPLE OF THE PHILIPPINES, Petitioner,


vs.
HON. SANDIGANBAYAN, FIRST DIVISION & THIRD DIVISION, HERNANDO BENITO
PEREZ, ROSARIO PEREZ, RAMON ARCEO and ERNEST ESCALER, Respondents.

FACTS:
Congressman Wilfrido B. Villarama of Bulacan delivered a privilege speech in the House of
Representatives denouncing acts of bribery allegedly committed by a high ranking government official
whom he then called the "2 Million Dollar Man." The Presidential Anti- Graft and Commission (PAGC)
conducted an inquiry on the exposé of Cong. Villarama and later confirmed it as DOJ Secretary Justice
Hernando B. Perez. Cong. Jimenez delivered a privilege speech in the House of Representatives
confirming and accusing Secretary Perez of extorting US$2 Million.
On November 25, 2002, then Ombudsman Simeon Marcelo requested PAGC to submit documents
relevant to the exposé Cong. Jimenez submitting his complaint-affidavit to the Office of the Ombudsman
on December 23, 2002.
The complaint-affidavit of Jimenez was redocketed for the criminal case in which the respondents were
Secretary Perez, Ernest L. Escaler and Ramon C. Arceo, Jr.; and as OMB-C-A-02-0631L, for the
administrative case involving only Secretary Perez as respondent.
The filing was amended continuously on separate days until April 18, 2008, when the Office of the
Ombudsman filed in the Sandiganbayan four informations against respondents, namely:
1. for violation of Sec. 3 (b) of Rep. Act 3019, as amended;
2. for Robbery (Art. 293, in relation to Art. 294, Revised Penal Code;
3. for Falsification of Public/Official Document under Art. 171 of the Revised Penal Code; and
4. for violation of Section 7, Rep. Act 3019, as amended, in relation to Section 8, Rep. Act 6713.
Issue: W/N the dismissal of the case is proper on the ground that it violates the accused rights to
just and speedy disposition of cases enshrined in the Constitution.
Ruling: Yes. The dismissal of the case is proper.
The long wait of the accused is without valid cause or justifiable motive and has unnecessarily trampled
upon their constitutional prerogatives to a speedy disposition of the case. This is an impermissible course
of action that our fundamental law loathes.
The Constitution of the Philippines provides:
Art. 3 Sec. 16: All persons shall have a right to a speedy disposition of their cases before all judicial(,)
quasijudicial or administrative bodies.
Thus under our present fundamental law, all persons are entitled to a speedy resolution of their cases be it
civil, administrative or criminal cases. It is, in criminal cases however where the need to a speedy
disposition of their cases is more pronounced. It is so, because in criminal cases, it is not only the honor
and reputation but even the liberty of the accused.
The charge is a simple case for Robbery. Certainly it does not involve complicated and factual issues that
would necessitate painstaking and gruelling scrutiny and perusal on the part of the Ombudsman. It may
have its novel, and to it, valid reason for departing from the established procedure and rules, but virtually
in doing so, it has failed to discharge its duty as mandated by the Constitution to promptly act on
complaints filed in any form or manner against public officers and employees.
The totality of the facts and the surrounding circumstances bears unmistakably the earmarks of inordinate
delay, making the applicability of the doctrine enunciated in Anchangco Jr. and Duterte cases cited in the
parties’ pleadings irrefragable.
Accordingly, there being a clear violation of the constitutional right of the accused, the prosecution is
ousted of any authority to file the information and we hereby order the quashing of the information and
the consequent dismissal of this case.
With the Ombudsman’s finding that the extortion (intimidation) was perpetrated on February 13, 2001
and that there was transfer of Mark Jimenez US $1,999,965.00 to Coutts Bank Account HO 133706 on
February 23, 2001 in favor of the accused, there is no reason why within a reasonable period from these
dates, the complaint should not be resolved. The act of intimidation was there, the asportation was
complete as of February 23, 2001 why was the information filed only on April 18, 2008. For such a
simple charge of Robbery there is nothing more to consider and all the facts and circumstances upon
which to anchor a resolution whether to give due course to the complaint or dismiss it are on hand. The
case is more than ripe for resolution. Failure to act on the same is a clear transgression of the
constitutional rights of the accused. A healthy respect for the constitutional prerogative of the accused
should have prodded the Ombudsman to act within reasonable time.
In fine, the Office of the Ombudsman transgressed the respondents' right to due process as well as their
right to the speedy disposition of their case.
WHEREFORE, the Court DISMISSES the petitions for certiorari for their lack of merit.

VENUS V DESIERTO
(G.R. No. 130319, October 21, 1998)

FACTS:
1. Eriberto Venus was the Municipal Mayor of New Washington, Aklan, while Mars C. Regalado
and Harry P. Abayon were members of the Sangguniang Bayan (SB) of said municipality.
2. The SB passed a Resolution authorizing Venus to negotiate and/or enter into a contract with
the Board of Liquidators (BOL), Office of the President in Manila in the acquisition of a certain
parcel of land. (The Garcia-Diapo Enterprise).
3. Venus went to Manila and submitted to the BOL the resolution and offered to buy the lot on a
government-to-government basis at a price mutually acceptable.
4. 8 Sept 1988: BOL rejected the offer and advised that a public bidding will be conducted
instead
5. Petitioner returned to his municipality and reported the same to the SB. He also presented a
list of his expenses during the authorized trip for reimbursement of P1,400 which he
subsequently received.
6. Petitioner sought the opinion of the Provincial Auditor regarding the requirements for the
participation of the municipality in the bidding. In view of the numerous requirements, the SB
doubted whether New Washington could participate in the public bidding. (timing and
resources limitations)
7. Auction Day: Nevertheless, Petitioner, on his own initiative and expenses went to Manila and
asked the BOL if they could postpone the auction so that his municipality could attend it.
However, this was denied and the auction proceeded. During the auction, petitioner, acting in
his personal capacity, submitted his personal bid for the land and won the bid.
a. He introduced improvements on the land at his expense.
b. During his incumbency as Mayor, he allowed a portion of the lot to be used, without
charge, as a garage for the municipality’s fire truck and for the municipality’s
mushroom culture laboratory.
8. OFFICE OF THE PROVINCIAL PROSECUTOR: Private respondents filed a letter-complaint
charging petitioner with violation of par (h) of Sec 3 of RA 3019 (Anti-Graft and Corrupt
Practices Act)
. Complaint affidavit: alleged that in contravention of the resolution and authority, in evident
bad faith and for the sole purpose of self-interest, petitioner bought the lot in his name and for
personal gain
a. Counter-affidavit: as told in the facts; filed for harassment
b. forwarded to office of deputy ombudsman
9. OFFICE OF THE DEPUTY OMBUDSMAN FOR THE VISAYAS: recommended
DISMISSAL
. No violation: what is contemplated in Section 3(h) of the anti-graft law is the actual
intervention in the transaction in which one has financial or pecuniary interest in order that liability
may attach
10. OMBUDSMAN VASQUEZ: disapproved resolution
. Consider possible liability for violation of Sec 3(e) of RA 3019: on BAD FAITH in
maneuvering to acquire the property for himself
a. Re-raffled; now OFFICE OF THE DEPUTY OMBUDSMAN FOR VISAYAS
recommended approval of filing of information
11. OFFICE OF THE SPECIAL PROSECUTOR: on review, Special Prosecutor Ines found
reasonable ground to charge petitioner and also prepared the Information
12. OMBUDSMAN: Ombudsman Desierto approved Ines resolution and the filing of Information
. Info in violation of 3(e) of RA 3019: “while in the performance of his official functions,
taking advantage of his position, and committing the offense in relation to his office, through
evident bad faith, did then and there wilfully, unlawfully and criminally cause undue injury to the
government… did then and there maneuver said sales deal and enter into a Contract of Sale with
the said Board of Liquidators in his own name instead, and purchased the aforementioned lot for
and in his own behalf despite the fact that he knew that said lot is badly needed by the Municipality
which is proposed to be the site of the Fire Fighting Station of the Municipality; to the damage and
prejudice of the Municipality of New Washington, Aklan and public interest
13. SANDIGANBAYAN: issued an order noting that a copy of the resolution directing the filing
of the information was sent by registered mail to petitioner only on 3 May 1996, or a day after
the filing of the information, thereby effectively depriving accused of his statutory right to file
a motion for reconsideration.
14. OFFICE OF SPECIAL PROSECUTOR: Upon a subsequent re-assessment of the evidence as
a consequence of petitioners motion for reconsideration, another Special Prosecution Officer,
Victor Pascual, found that petitioner had not violated Sec. 3(e) of R.A. No. 3019, as amended.
He thus recommended dismissal of the case for want of probable cause and the filing of the
corresponding manifestation to inform the Sandiganbayan of the result of the motion for
reconsideration. In this instance, the Special Prosecutor himself concurred with the finding.
However, the Ombudsman disapproved the recommendation as he found that probable cause
existed, but opted to allow the court to find absence of bad faith.
15. SANDIGANBAYAN: Noted Pascual’s Manifestation and set petitioner’s arraignment on Oct
20
16. SUPREME COURT: Petitioner filed In this is petition for prohibition under Rule 65, with
application for a temporary restraining order and writ of preliminary injunction to annul
and set aside approval or resolution finding probable cause and the information and prohibit
the Ombudsman from further prosecuting the case; and prohibit the Sandiganbayan from acting
on and trying Criminal Case No. 23332. SC issued TRO. Sandiganbayan granted motion to
defer arraignment.
ISSUE: W/N the case should be dismissed on the ground of lack of probable cause despite the
finding of the Ombudsman that there is.
HELD: Yes. Ordinarily SC does not interfere with discretion of the Ombudsman in the
determination of probable cause. However there are certain exceptions, one of which is: where
there is clearly no prima facie case against the accused and motion to quash has been denied.
Ratio:
Conformably with the general rule that criminal prosecutions may not be restrained either through
a preliminary or final injunction or a writ of prohibition, this Court ordinarily does not interfere
with the discretion of the Ombudsman to determine whether there exists reasonable ground to
believe that a crime has been committed and that the accused is probably guilty thereof and,
thereafter, to file the corresponding information with the appropriate courts. Exceptions
enumerated in Brocka v Enrile:
a. To afford protection to the constitutional rights of the accused (Hernandez vs. Albano, et
al.)
b. When necessary for the orderly administration of justice or to avoid oppression or
multiplicity of actions (Dimayuga, et al. vs. Fernandez, 4; Hernandez vs. Albano, supra;
Fortun vs. Labang)
c. When there is a prejudicial question which is sub judice (De Leon vs. Mabanag, 70 Phil.
202);
d. When the acts of the officer are without or in excess of authority (Planas vs. Gil, 67 Phil.
62);
e. Where the prosecution is under an invalid law, ordinance or regulation (Young vs.
Rafferty)
f. When double jeopardy is clearly apparent (Sangalang vs. People and Alvendia, 109 Phil.
1140);
g. Where the court has no jurisdiction over the offense (Lopez vs. City Judge, L-25795)
h. Where it is a case of persecution rather than prosecution (Rustia vs. Ocampo,
i. Where the charges are manifestly false and motivated by the lust for vengeance (Recto vs.
Castelo, 18 L.J. (1953), cited in Raoa vs. Alvendia, CA G.R. No. 30720-R, October 8,
1962; Cf. Guingona, et al. vs. City Fiscal, L-60033, April 4, 1984, 128 SCRA 577);
j. Where there is clearly no prima facie case against the accused and a motion to quash
on that ground has been denied (Salonga vs. Pao, et al., L-59524, February 18, 1985,
134 SCRA 438); and
k. Preliminary injunction has been issued by the Supreme Court to prevent the threatened
unlawful arrest of petitioners (Rodriguez vs. Castelo, L-6374, August 1, 1953) (cited in
Regalado, Remedial Law Compendium, p. 188, 1988 Ed.)
GR: The rule is based not only upon respect for the investigatory and prosecutory powers granted
by the Constitution to the Office of the Ombudsman but upon practicality as well. Otherwise, the
functions of the courts will be grievously hampered by innumerable petitions assailing the
dismissal of investigatory proceedings conducted by the Office of the Ombudsman with regard to
complaints filed before it, in much the same way that the courts would be extremely swamped if
they would be compelled to review the exercise of discretion on the part of the fiscals or
prosecuting attorneys each time they decide to file an information in court or dismiss a complaint
by a private complainant.
EXC: SC may interfere with the discretion of the Ombudsman in case of clear abuse of discretion.

On bad faith
Good faith is presumed. To be a violation of 3(e) of RA 3019, bad faith must be evident. On the
basis alone of the finding and conclusion of Special Prosecution Officer III Victor Pascual, with
which the Special Prosecutor concurred, there was no showing of bad faith on the part of
petitioner. It was, therefore, error for the Ombudsman to pass the buck, so to speak, to the
Sandiganbayan to find absence of bad faith.
Question of GF: revolves around the proper application/interpretation of the resolution issued by
the SB of New Washington. In no way then may petitioner be deemed to have acted with bad
faith in not submitting a bid for and in behalf of the municipality since the resolution did not
authorize him to do so and the municipality was in no position to submit a bid and only wanted to
enter into a negotiated contract of sale.
Whether the SC can order the Sandiganbayan to dismiss the case
The Sandiganbayan may agree or disagree with the findings of the Ombudsman based on its
independent assessment of the merits of the case therefore the SC can remand the case to the
Sandiganbayan Nevertheless, where the innocence of an accused is manifest from the evidence as
in the case at bar, there is no reason to merely remand the case. A court should never play into the
hands of the prosecution and blindly comply with its erroneous manifestations. Faced with an
information charging a manifestly non-existent crime, the duty of a trial court is to throw it out. Or,
at the very least and where possible, make it conform to the law. (JPE vs Salazar)
WHEREFORE, the instant petition is GRANTED. For want of reasonable ground to believe
that petitioner violated Section 3(e) of R.A. No. 3019, as amended, or for absence of probable
cause therefor, the Sandiganbayan is hereby ORDERED to forthwith DISMISS Criminal Case
No. 23332, entitled People of the Philippines versus Eriberto L. Venus, and to SUBMIT to this
Court a report of its compliance within ten (10) days from such dismissal. The temporary
restraining order issued on 12 January 1998 is hereby made permanent.
MAGSUCI VS SANDIGANBAYAN
G.R. L-101545 January 3, 1995
FACTS:

BFAR and Dexter Company, represented by Ancla, entered into an agreement for
the

construction of certain structures. During the course of the undertaking, petitioner


was

designated as the BFAR regional director having the duty, among others, to supervise such

construction. Enriquez, the then BFAR Central Engineer charged with certifying the progress

of Dexter, together with Ancla, presented their Accomplishment Report and Certification of

Completion attesting that the undertaken construction has been completed. Petitioner

reviewed their reports, affixed his signature on the disbursement voucher releasing
the

public funds to Dexter, and co-signed the checks for disbursement. However, it eventually

appeared that the project was still underway and had still yet to be completed. Because of

this, petitioner was charged for falsification of public documents in conspiracy with Enriquez

and Ancla

BFAR and Dexter Construction, represented by its Manager, Ancla, entered into an
agreement for the construction of a 40 ton ice making plant, 150-ton ice storage, and a 350-
ton cold storage facility in Surigao City. A MOA was also entered to purchase and install three
distribution transformers and a steel elevated tank. BFAR Central Office Engineer Enriquez was
charged with the duty to render accomplishment reports on the progress of the construction, and
this was prepared and bore the signature of Ancla. The newly designated BFR Regional Dir
Magsuci read the report and affixed his signature thereon and directed the issuance of the
voucher, signed it, the voucher carrying the standard printed certification that the expenses were
necessary, lawful, and incurred under his supervision. He likewise signed four checks, payable to
Dexter Construction.

As it turned out, the additional work so represented to have been accomplished in the report and
certification has yet to be undertaken. It would appear that the work has been completed in
December 1983. The Sandiganbayan rendered judgement, finding Magsuci guilty of estafa
through falsification of public documents.
ISSUE/S:

WON Magsuci should be held guilty for the crime of Estafa

RULING:

NO. Magsuci should be acquitted. It is fairly evident is the fact that the actions taken by
petitioner involved the very functions he had to discharge in the performance of his official
duties. However, he had no intimation at all that he had foreknowledge of any irregularity
committed by either Engr. Enriquez or Ancla. Magsuci might have been indeed lax and
administratively remiss in placing too much reliance on the official reports submitted by
Enriquez, his subordinate, but for conspiracy to exist, it is essential that there be a conscious
design to commit an offense. The Sandiganbayan erred in its ruling that there was conspiracy
between Magcusi, Enriquez, and Ancla. Conspiracy is not the product of negligence but of
intentionally on the part of cohorts.

PEDRO G. SISTOZA,, Petitioner, v. ANIANO DESIERTO in his capacity as Ombudsman, and


ELISEO CO, Respondents.
G.R. No. 144784. September 3, 2002

FACTS: On 10 August 1999 the Pre-Qualification, Bid and Awards Committee (PBAC) of the Bureau of
Corrections offered for public bidding the supply of tomato paste in addition to other food items for
consumption in the month of September.
Among the bidders were RBJJ, PMS Trading Enterprises, Filcrafts Industries, Inc., and Elias General
Merchandising. Elias General Merchandising offered a bid of P1,350.00 for 100/170 tins-grams to 1 case
while RBJJ and PMS Trading Enterprises tendered their respective bids for the same quantity at the higher
prices of P1,380.10 and P1,380.05 per case. On the other hand, Filcrafts Industries, Inc., offered P539.00
for the quantity of 48/198 tins-grams to 1 case.
It appears that the bid tender form executed by Elias General Merchandising and submitted to PBAC
already indicated a change in the quantity specification from which PBAC approved as shown by the initials
of the chairman and members thereof. In the same extent, PBAC rejected the bid of Filcrafts Industries,
Inc., for offering a non-registered brand of tomato paste in the Philippines and its failure to specify in the
bid tender form the country of origin of the tomato paste it would supply. Elias General Merchandising won
the bidding with its offer of P1,350.00 for 100/170 tins-grams to 1 case.
On August 13, 1999 the Supply Division of the Bureau of Corrections prepared the purchase order for the
one-month supply of tomato paste in favor of Elias General Merchandising. Petitioner Sistoza received the
purchase order and its supporting documents, briefly read them and thereafter affixed his signature on the
purchase order.
The Justice Department observed that the award to the supplier with only the second lowest bid was not
adequately justified in the resolution of the PBAC. The winning bidder replied to the Supply Division and
expressed its willingness to "meet the price of the lowest bidder for item No. 55, tomato paste which is more
or less P1,120.00/box for 100 cans/170 grams." The Supply Division proposed in return the price of
P964.12/box of 100 cans/170 grams supposedly matching the lowest bid of Filcrafts Industries Inc. Elias
General Merchandising rejected the counter-offer and pegged its price offer at P1,120.00 for 100 cans/170
grams.
On November 29 1999 Sistoza endorsed for the third time the purchase order of tomato paste in favor Elias
General Merchandising to the Department of Justice. he Pre-Qualification, Bid and Awards Committee in
its resolution dated September 2, 1999 states that Item No. 55 (tomato paste) was awarded to Elias General
Merchandise in spite of being the 2nd lowest bidder due to the fact that the offer of Filcrafts Industries, Inc.
does not conform to the specification provided for in the purchase orders.
Respondent Eliseo Co, bidder for supply of food items of the New Bilibid Prison, filed an affidavit-complaint
with the Office of the Ombudsman alleging criminal and administrative charges for violation of Sec. 3, par.
(e), RA 3019, otherwise known as the Anti-Graft and Corrupt Practices Act, against petitioner Pedro G.
Sistoza (Director of the Bureau of Corrections) and officers and members of its Supply Division and PBAC.
He claimed that Sistoza and his staff conspired with each other to cause undue injury to the government
and the inmates of the New Bilibid Prison by giving undue advantage to Elias General Merchandise
although its bid was higher in price and lower in quantity than that offered by Filcrafts Industries, Inc.
Office of the Ombudsman dismissed the administrative proceedings against petitioner Sistoza and some of
his co-respondents on the ground that their actions in awarding the supply of tomato paste to Elias General
Merchandising were merely recommendatory and that they were effectively scrutinized and validated when
the award was eventually approved by the Department of Justice.
On June 22, 2000 Sistoza filed with the Sandiganbayan a motion for reinvestigation and suspension of
proceedings therein. Petitioner Sistoza argues that he had no active participation in the award of the supply
of tomato paste to Elias General Merchandising and that his involvement was limited to signing the
purchase order for this food item. He concludes that as a matter of law his signature on the purchase order,
without more, does not prove any violation of Sec. 3, par. (e), RA 3019.
LAW: Section 3, par. (e), RA No. 3019 defines "corrupt practices of public officers." It provides-
In addition to acts or omissions of public officers already penalized by existing law, the following shall
constitute corrupt practices of any public officer and are hereby declared to be unlawful x x x x (e) Causing
any undue injury to any party, including the Government, or giving any private party any unwarranted
benefits, advantage or preference in the discharge of his official administrative or judicial functions through
manifest partiality, evident bad faith or gross inexcusable negligence. This provision shall apply to officers
and employees of offices or government corporations charged with the grant of licenses or permits or other
concessions.

The elements of the offense are: (a) The accused is a public officer or a private person charged in
conspiracy with the former; (b) The public officer commits the prohibited acts during the performance of his
or her official duties or in relation to his or her public functions; (c) That he or she causes undue injury to
any party, whether the government or a private party; (d) Such undue injury is caused by giving unwarranted
benefits, advantage or preference to such parties; and, (e) That the public officer has acted with manifest
partiality, evident bad faith or gross inexcusable neglect.
ISSUE: whether or not there is reasonable ground to believe that the accused is guilty of the offense
charged and should be subjected to the expense, strictness and embarrassment of trial, is the function of
the prosecution.
RULING: No. The Information against petitioner Sistoza, while specifying manifest partiality and evident
bad faith, does not allege gross inexcusable negligence as a modality in the commission of the offense
charged. In the instant case, there is no direct evidence that petitioner Sistoza acted in conspiracy with the
officers and members of the PBAC and the other implicated public officials. He did not himself participate
in the bidding procedures nor was he involved in the award of the supply of tomato paste to Elias General
Merchandising. As defined, the acts charged against petitioner do not amount to manifest partiality, evident
bad faith nor gross inexcusable negligence which should otherwise merit a prosecution for violation of Sec.
3, par. (e), RA 3019.
The instant case brings to the fore the importance of clearly differentiating between acts simply negligent
and deeds grossly and inexcusably negligent punishable under Sec. 3, par. (e), of the Anti-Graft and
Corrupt Practices Act.
Since petitioner had no reason to doubt the validity of the bidding process and given the urgency of the
situation since the tomato paste had by then been delivered and consumed by the inmates of the New
Bilibid Prison, we certainly cannot infer malice, evident bad faith or gross inexcusable negligence from his
signing of the purchase order and endorsing the same to the Department of Justice.
While it is the function of the Ombudsman to determine whether or not the petitioner should be subjected
to the expense, rigors and embarrassment of trial, he cannot do so arbitrarily.
Petition is Granted.

Department of Justice vs. Liwag


G.R No. 149311, February 11, 2005

FACTS: Mary Ong, a former undercover agent of the Presidential Anti-Organized Crime Task
Force (PAOCTF) and the Philippine National Police (PNP) Narcotics Group, filed a complaint-
affidavit of kidnapping and murder on January 8, 2001 before the Ombudsman against PNP
General Panfilo M. Lacson, PNP Colonel Michael Ray B. Aquino, other high-ranking officials of
the PNP, and several private individuals. Subsequently, on March 9, 2001, Mary Ong and other
witnesses executed sworn statements before the NBI, alleging the same facts and
circumstances revealed by Mary Ong in her complaint-affidavit before the Ombudsman. NBI
Director Reynaldo Wycoco, in a letter dated May 4, 2001 addressed to then Secretary of Justice
Hernando Perez, recommended the investigation of Lacson, Aquino, other PNP officials, and
private individuals for the following alleged crimes. On May 2001, a panel of prosecutors from
the DOJ sent a subpoena to Lacson, Aquino and the other persons named in the witnesses’
sworn statements. Lacson and Aquino, through their counsel, manifested in a letter dated May
18, 2001, that the DOJ panel of prosecutors should dismiss the complaint filed therewith by
Mary Ong since there are complaints pending before the Ombudsman alleging a similar set of
facts against the same respondents. They further claimed that in the case of Uy vs
Sandiganbayan, the Ombudsman has primary jurisdiction over criminal cases cognizable by the
Sandiganbayan and, in the exercise of this primary jurisdiction, he may take over, at any stage,
from any investigatory agency of Government, the investigation of such cases involving public
officials, including police and military officials such as private respondents. DOJ interpreted the
letter as a motion to dismiss and immediately denied the same. On the very same day that the
DOJ issued the aforesaid Order, the Solicitor General received a copy of a petition for
prohibition filed by Lacson and Aquino before the Regional Trial Court (RTC) of Manila. Judge
Liwag, judge of the RTC of Manila issued an order prohibiting the DOJ from conducting the
preliminary investigation. Hence the instant petition.
ISSUE: WON the DOJ has jurisdiction to conduct the preliminary investigation despite an
existing pendency before the Ombudsman showing the same facts.
RULING: NO. Section 15, Republic Act No. 6640, known as the Ombudsman Act of 1989,
provides that the Office of the Ombudsman shall have the following power to investigate and
prosecute on its own or on complaint by any person, any act or omission of any public officer or
employee, office or agency, when such act or omission appears to be illegal, unjust, improper or
inefficient. It has primary jurisdiction over cases cognizable by the Sandiganbayan and, in the
exercise of this primary jurisdiction, it may take over, at any stage, from any investigatory
agency of Government, the investigation of such cases. RTC Judge Liwag, in his ruling,
mentioned that in a preliminary investigation conducted by the Ombudsman itself, the other
investigative agencies of the Government have no power and right to add an input into the
Ombudsman’s investigation. Only in matters where the other investigative agencies are
expressly allowed by the Ombudsman to make preliminary investigation may such agencies
conduct the investigation, subject to the final decision of the Ombudsman.

Furthermore, Section 13, Article XI of the Constitution specifically vests in the Office of the
Ombudsman the plenary power to investigate any malfeasance, misfeasance or non-feasance
of public officers or employees. To discharge its duty effectively, the Constitution endowed the
Office of the Ombudsman with special features which puts it a notch above other grievance-
handling, investigate bodies.

The power to take over a case at any time is not given to other investigative bodies. All this
means that the power of the Ombudsman to investigate cases cognizable by the
Sandiganbayan is not co-equal with other investigative bodies, such as the DOJ.

The DOJ has general jurisdiction to conduct preliminary investigation of cases involving
violations of the Revised Penal Code, this general jurisdiction cannot diminish the plenary power
and primary jurisdiction of the Ombudsman to investigate complaints specifically directed
against public officers and employees. The Office of the Ombudsman is a constitutional
creation. In contrast, the DOJ is an extension of the executive department, bereft of the
constitutional independence granted to the Ombudsman.

The subsequent assumption of jurisdiction by the DOJ in the conduct of preliminary


investigation over the cases filed against the respondents would not promote an orderly
administration of justice. To allow the same complaint to be filed successively before two or
more investigative bodies would promote multiplicity of proceedings. It would also cause undue
difficulties to the respondent who would have to appear and defend his position before every
agency or body where the same complaint was filed.

There is also the distinct possibility that the two bodies exercising jurisdiction at the same time
would come up with conflicting resolutions regarding the guilt of the respondents. Lastly, a
second investigation would entail an unnecessary expenditure of public funds, and the use of
valuable and limited resources of Government, in a duplication of proceedings already started
with the Ombudsman. PETITION DISMISSED
CARMELO F. LAZATIN, MARINO A. MORALES, TEODORO L. DAVID and ANGELITO A.
PELAYO, Petitioner, v.
HON. ANIANO A. DESIERTO as OMBUDSMAN, and SANDIGANBAYAN, THIRD
DIVISION, Respondents.
[G.R. NO. 147097, June 5, 2009, PERALTA, J.]

FACTS:
1. On July 22, 1998, the Fact-Finding and Intelligence Bureau of the Office of the Ombudsman filed a
Complaint-Affidavit charging Lazatin, et al. with Illegal Use of Public Funds under the Revised
Penal Code and the Anti Graft and Corrupt Practices Act.
2. The complaint alleged that there were irregularities in the use by then Congressman Carmelo F.
Lazatin of his Countrywide Development Fund (CDF) for the calendar year 1996, i.e., he was both
proponent and implementer of the projects funded from his CDF; he signed vouchers and
supporting papers pertinent to the disbursement as Disbursing Officer; and he received, as claimant,
eighteen (18) checks amounting to P4,868,277.08. Lazatin, with the help of co-petitioners was
allegedly able to convert his CDF into cash.
3. The Evaluation and Preliminary Investigation Bureau (EPIB) recommending the filing of fourteen
(14) counts each of Malversation of Public Funds and violation of Section 3 (e) of R.A. No. 3019.
Said Resolution was approved by the Ombudsman. Twenty-eight (28) Informations were filed
against the petitioners before the Sandiganbayan.
4. Lazatin and his co-petitioners then filed their respective Motions for Reconsideration/
Reinvestigation, which were granted by the Sandiganbayan (Third Division). The Sandiganbayan also
ordered the prosecution to re-evaluate the cases against petitioners.
5. Subsequently, the Office of the Special Prosecutor recommended the dismissal of the cases for lack
or insufficiency of evidence. The Ombudsman ordered the Office of the Legal Affairs (OLA) to
review the OSP Resolution. The OLA recommended that the OSP Resolution be disapproved and
the OSP be directed to proceed with the trial of the cases against petitioners. The Ombudsman
adopted the OLA Memorandum. The cases were then returned to the Sandiganbayan for
continuation of criminal proceedings.

Petitioners’ Contention:
• The Ombudsman had no authority to overturn the OSP's Resolution because, under Section 13,
Article XI of the 1987 Constitution, the Ombudsman is clothed only with the power to watch,
investigate and recommend the filing of proper cases against erring officials, but it was not granted
the power to prosecute. They point out that under the Constitution, the power to prosecute belongs
to the OSP (formerly the Tanodbayan), which was intended by the framers to be a separate and
distinct entity from the Office of the Ombudsman. Petitioners maintain that R.A. No. 6770 (The
Ombudsman Act of 1989), which made the OSP an organic component of the Office of the
Ombudsman, should be struck down for being unconstitutional.

ISSUE: Whether or not the Ombudsman had the authority to overturn the decision of the Office of the
Special Prosecutor

RULING: Yes, the Ombudsman has the authority to overturn decisions of the OSP.
1. It has long been settled that the provisions of R.A. No. 6770 granting the Office of the Ombudsman
prosecutorial powers and placing the OSP under said office have no constitutional infirmity. The
Court held that giving prosecutorial powers to the Ombudsman is in accordance with the
Constitution as paragraph 8, Section 13, Article XI provides that the Ombudsman shall "exercise
such other functions or duties as may be provided by law."
2. The constitutionality of Section 3 of R.A. No. 6770, which subsumed the OSP under the Office of
the Ombudsman, was likewise upheld by the Court in Acop. It was explained, thus:
x x x Section 7 of Article XI expressly provides that the then existing Tanodbayan, to be henceforth known as the Office
of the Special Prosecutor, "shall continue to function and exercise its powers as now or hereafter may be
provided by law, except those conferred on the Office of the Ombudsman created under this Constitution."
The underscored phrase evidently refers to the Tanodbayan's powers under P.D. No. 1630 or subsequent
amendatory legislation. It follows then that Congress may remove any of the Tanodbayan's/Special
Prosecutor's powers under P.D. No. 1630 or grant it other powers, except those powers conferred by the
Constitution on the Office of the Ombudsman.

Pursuing the present line of reasoning, when one considers that by express mandate of paragraph 8, Section 13, Article XI
of the Constitution, the Ombudsman may "exercise such other powers or perform functions or duties as may
be provided by law," it is indubitable then that Congress has the power to place the Office of the Special
Prosecutor under the Office of the Ombudsman. In the same vein, Congress may remove some of the
powers granted to the Tanodbayan by P.D. No. 1630 and transfer them to the Ombudsman; or grant the
Office of the Special Prosecutor such other powers and functions and duties as Congress may deem fit and
wise. This Congress did through the passage of R.A. No. 6770.

3. In Office of the Ombudsman v. Valera, the Court, basing its ratio decidendi on its ruling in Acop and
Camanag, declared that the OSP is "merely a component of the Office of the Ombudsman and may only act
under the supervision and control, and upon authority of the Ombudsman." In Perez v. Sandiganbayan, it was
held that the power to prosecute carries with it the power to authorize the filing of informations.

JUDGE ADORACION G. ANGELES, vs. HON. MA. MERCEDITAS N. GUTIERREZ,


Ombudsman
G.R. Nos. 189161 & 189173 March 21, 2012

FACTS: On 20 February 2007, petitioner Judge Angeles filed a criminal Complaint against
respondent Velasco with the Ombudsman and sought his indictment before the Sandiganbayan for
the following acts allegedly committed in his capacity as a prosecutor:
1. Giving an unwarranted benefit, advantage or preference to the accused in a criminal case
for smuggling by failing to present a material witness; (Velasco was the trial prosecutor in
a criminal case involving the smuggling of jewelry, failed to present a material witness in
4

the aforesaid case)


2. Engaging in private practice by insisting on the reopening of child abuse cases against
petitioner; (Velasco’s filing of two Petitions to reopen the child abuse cases filed against
petitioner Judge Angeles. Petitioner who was previously charged with inflicting physical
and psychological abuse on Maria Mercedes Vistan, her 13-year-old grandniece.)
3. Falsifying a public document to make it appear that a clarificatory hearing on the child
abuse Complaint was conducted. (The alleged falsification of public document arose from
the same preliminary investigation conducted by respondent in the child abuse cases
mentioned above. According to petitioner Judge Angeles, respondent Velasco made it
appear that he had conducted a clarificatory hearing when in fact the supposedly witness
Leonila Vistan, , was seriously sick and could not have appeared at the alleged
clarificatory hearing)
The Ombudsman dismissed the charges against respondent Velasco. It found that after evaluation
of the facts and evidence presented by complainant, there was no cause to conduct a preliminary
investigation or an administrative adjudication with regard to the charges.

ISSUE: WON the OMBUDSMAN committed grave abuse of discretion amounting to lack or
excess of jurisdiction in dismissing the Complaint against respondent Velasco.

HELD: NO.
As a general rule, the Court does not interfere with the Ombudsman’s exercise of its
investigative and prosecutorial powers without good and compelling reasons. Such reasons are
clearly absent in the instant Petition.
The Ombudsman is empowered to determine whether there exists reasonable ground to
believe that a crime has been committed and that the accused is probably guilty thereof and,
thereafter, to file the corresponding information with the appropriate courts. Settled is the rule that
the Supreme Court will not ordinarily interfere with the Ombudsman’s exercise of his investigatory
and prosecutory powers without good and compelling reasons to indicate otherwise. Said exercise
of powers is based upon the constitutional mandate and the court will not interfere in its exercise.
The rule is based not only upon respect for the investigatory and prosecutory powers granted by
the Constitution to the Office of the Ombudsman, but upon practicality as well. Otherwise,
innumerable petitions seeking dismissal of investigatory proceedings conducted by the
Ombudsman will grievously hamper the functions of the office and the courts, in much the same
way that courts will be swamped if they had to review the exercise of discretion on the part of
public prosecutors each time they decided to file an information or dismiss a complaint by a private
complainant.
The Rules of Procedure of the Office of the Ombudsman (Ombudsman Rules of
Procedure), specifically Section 2 of Rule II, states:
34

Evaluation. — Upon evaluating the complaint, the investigating officer shall recommend whether
it may be: a) dismissed outright for want of palpable merit; b) referred to respondent for comment;
c) indorsed to the proper government office or agency which has jurisdiction over the case; d)
forwarded to the appropriate officer or official for fact-finding investigation; e) referred for
administrative adjudication; or f) subjected to a preliminary investigation.
Thus, the Ombudsman need not conduct a preliminary investigation upon receipt of a complaint.

EMILIO A. GONZALES III, vs. OFFICE OF THE PRESIDENT OF THE PHILIPPINES,


acting through and represented by EXECUTIVE SECRETARY PAQUITO N. OCHOA,
JR

FACTS:
Rolando Mendoza was charged with Grave Misconduct (robbery, grave threats, robbery extortion
and physical injuries). Petitioner, as the deputy Ombudsman, recommended that Mendoza is guilty
of the charges. Mendoza was then involved in a hostage taking incident in a bus which resulted to
the death of 8 HongKong Nationals. Due to the public outcry, an investigation was conducted by
Incident Investigation and Review Committee(IRRC). IRRC found Deputy Ombudsman Gonzales
committed serious and inexcusable negligence and gross violation of their own rules of procedure
by allowing Mendoza's motion for reconsideration to languish for more than nine (9) months
without any justification, in violation of the Ombudsman prescribed rules to resolve motions for
reconsideration in administrative disciplinary cases within five (5) days from submission.
ISSUE:
Whether or not the Office of the President has jurisdiction over a Deputy Ombudsman and exercise
administrative disciplinary power
RULING:
Yes. The Power of the President to Remove a Deputy Ombudsman and a Special Prosecutor is
Implied from his Power to Appoint. In giving the President the power to remove a Deputy
Ombudsman and Special Prosecutor, Congress simply laid down in express terms an authority that
is already implied from the President's constitutional authority to appoint the aforesaid officials in
the Office of the Ombudsman.

ANTONINO vs. DESIERTO


(G.R. NO. 144492, December 18, 2008)

Facts: Pres. Macapagal issued Presidential Proclamation No. 168 withdrawing from sale or settlement and
reserve for recreation and health resort site purposes some land of the public domain in General Santos. The
lands were subdivided into 3 lots: Lot Y-1, Lot X and Lot Y-2. Marcos amended the proclamation and
opened for disposition Lot Y-1 and Lot Y-2. Heris of Kuspo applied for Free Patent and Certificate of Title
with regards to the 2 lots in which the government of General Santos filed two cases against the heirs.
Subsequently, the parties entered into a compromise agreement where the heirs would only be entitled to
20,000 sqm. of the total area of 37,658 sqm. granting the government the excess to have a total of 32,678
sqm. including Lot X.

The compromise agreement was approved and the cases were dismissed. A writ of execution was issued. Judge
Adre issued Order for the exclusion of Lot X in the compromise agreement and that the heirs are enjoined
to donate to the city government. Subsequently, private respondents applied for Miscellaneous Sales Patents
over Lot X (which was still covered under Proclamation no. 168 as reserved for recreational and health resort
site purposes). The lot was subdivided into 16 where the City Mayor Nuez (respondent) did not object to
whatever legal proceedings with regards to Lot X. An appraisal was submitted by other respondents and a
notice of sale and Certificates of Titles were issued. The owners of the 16 lots transferred, sold, exchanged
the lots to the AFPRetirement and Separation Benefits System.

Petitioners then filed a complaint for malversation of public funds or property through falsification of
public documents (16 applications for miscellaneous sales patents) with the Office of the Ombudsman. The
latter dismissed the case with regards to other respondents who he only found to have regularly performed their
office functions in good faith. More than a year after the resolution (Jan. 20, 1999), the petitioners filed for
a Motion for Reconsideration on Feb. 4, 2000 contending that the Ombudsman committed grave abuse of
discretion in dismissing charges against respondents.

Issue: w/n the Ombudsman committed grave abuse of discretion, amounting to lack or in excess of
jurisdiction in the exercise of his prosecutor functions, by dismissing the charges against the respondents.

Held: Under Sections 12 and 13, Article XI of the 1987 Constitution, and pursuant to R.A. No. 6770, the
Ombudsman has the power to investigate and prosecute any act or omission of a public officer or
employee when such act or omission appears to be illegal, unjust, improper or inefficient. Well-settled is the
rule that this Court will not ordinarily interfere with the Ombudsman's exercise of his investigatory and
prosecutory powers without good and compelling reasons that indicate otherwise. A contrary rule would
encourage innumerable petitions seeking dismissal of investigatory proceedings conducted by the
Ombudsman, which would grievously hamper the functions of the office and the courts, in much the same
way that courts would be swamped by a deluge of cases if they have to review the exercise of discretion
on the part of public prosecutors each time they decide to file an information or dismiss a complaint by a
private complainant.

Of course, this rule is not absolute. The aggrieved party may file a petition for certiorari under Rule 65
of the Rules of Court when the finding of the Ombudsman is tainted with grave abuse of discretion amounting
to lack or excess of jurisdiction. This Court laid down the following exceptions to the rule:
1.When necessary to afford adequate protection to the constitutional rights of the accused;
2.When necessary for the orderly administration of justice or to avoid oppression or multiplicity of actions;
3.When there is a prejudicial question that is sub judice;
4.When the acts of the officer are without or in excess of authority;
5.Where the prosecution is under an invalid law, ordinance or regulation;
6.When double jeopardy is clearly apparent;
7.Where the court has no jurisdiction over the offense;
8.Where it is a case of persecution rather than prosecution;
9.Where the charges are manifestly false and motivated by the lust for vengeance;
10. When there is clearly no prima facie case against the accused and a motion to quash on that ground has
been denied.

Grave abuse of discretion exists where a power is exercised in an arbitrary, capricious, whimsical or despotic
manner by reason of passion or personal hostility so patent and gross as to amount to evasion of positive
duty or virtual refusal to perform a duty enjoined by, or in contemplation of law. The alleged grave abuse
of discretion imputed to the Ombudsman is found wanting in this case. Thus, this Court finds no reason
to deviate from the general rule.

ENEMECIO v. OFFICE OF THE OMBUDSMAN and BERNANTE


G.R. No. 146731, January 13, 2004

Facts:
Petitioner Enemecio is a utility worker at the Cebu State College of Science and Technology,
College of Fisheries Technology (CSCST-CFT). Respondent Bernante is an Assistant Professor
of CSCST-CFT.

Enemecio filed an administrative complaint for gross misconduct, falsification of public


documents, malversation, dishonesty and defamation against Bernante. Enemecio also filed with
the Ombudsman a criminal complaint against Bernante for falsification of public document. The
Ombudsman jointly tried the two cases.

Enemecio alleged Bernante caused the painting of obscene words against her on the walls of the
campus, that Bernante shouted defamatory words against her, and that Bernante made it appear in
his leave application that he was on forced leave from 15 May 1996 to 21 May 1996 and on
vacation leave from 22 May 1996 to 31 May 1996. In truth, Bernante was serving a 20-day prison
term because of his conviction of the crime of slight physical injuries. Bernante was still able to
receive his salary during his incarceration since the superintendent approved Bernante’s
application for leave. Enemecio contended that Bernante was not entitled to receive salary for that
period because of his “falsified leave applications.”

On 13 January 2000, the Ombudsman rendered a decision dismissing the administrative complaint
against Bernante, holding that he evidence is insufficient to prove that respondent was the person
responsible for such.

On the same date, the Ombudsman dismissed the criminal complaint against Bernante, finding no
probable cause to indict Bernante for falsification of public document, since there is no regulation
or law against the utilization of leave credits for purposes other than recreation. The Ombudsman
denied Enemecio’s motion to reconsider the dismissal of the criminal complaint

Enemecio filed a special civil action for certiorari before the CA. Applying the ruling in Fabian v.
Desierto, the CA dismissed Enemecio’s petition for having been filed out of time. The appellate
court also stated that the proper remedy available to Enemecio was a petition for review under
Rule 43 and not a petition for certiorari under Rule 65.

In dismissing the petition, the CA stated that Fabian held that appeals in administrative
disciplinary cases from the Ombudsman to the CA must be brought by petition for review under
Rule 43. The CA stated that a petition for review must be filed within 15 days from notice of the
assailed final order or resolution. Since Enemecio received on 22 March 2000 a copy of the
Ombudsman’s Order denying her motion for reconsideration, the appellate court ruled that
Enemecio had only until 6 April 2000 to file a petition for review. Enemecio filed her petition
only on 8 May 2000. The appellate court further stated that Enemecio’s allegation in the petition
that there is no appeal or other plain, speedy or adequate remedy in the ordinary course of law is
false. The proper remedy available to Enemecio is a petition for review

In denying Enemecio’s motion for reconsideration, the CA held Fabian does not apply to
Enemecio’s petition assailing the dismissal of the criminal complaint against Bernante. Fabian
declared void Section 27 of RA 6770, which allows appeals to the SC from decisions of the
Ombudsman in administrative disciplinary cases. Under the Fabian ruling, the appellant should
take such appeal in administrative disciplinary cases to the Court of Appeals under Rule 43. The
Court of Appeals added that it follows that the power to review decisions of the Ombudsman in
criminal cases is retained by the Supreme Court under Section 14 of RA 6770. Thus, the appellate
court dismissed the petition for lack of jurisdiction.

Issue:
Whether a petition for certiorari under Rule 65 filed before the Court of Appeals is the proper
remedy to question the dismissal of a criminal complaint filed with the Ombudsman? (NO)

Ratio:
Enemecio filed before the Court of Appeals a petition for certiorari under Rule 65 questioning the
Ombudsman’s Resolution dated 13 January 2000 and Order dated 28 February 2000 dismissing
the criminal case against Bernante.

The appellate court dismissed Enemecio’s petition and denied her motion for
reconsideration. Enemecio now comes to this Court via this petition for review, claiming that
“what was involved in the petition before the appellate court was the administrative, not the
criminal case.” However, it is clear from the records that petitioner filed with the Court of Appeals
a certiorari petition assailing the Ombudsman’s Resolution and Order dismissing the criminal
case, not the administrative case against Bernante. For this reason, the appellate court in its 7
December 2000 Resolution rectified itself and stated that Fabian does not apply to Enemecio’s
petition as the Fabian ruling applies only to administrative disciplinary actions.

The action must also fail. Appeals from decisions of the Ombudsman in administrative
disciplinary actions should be brought to the Court of Appeals under Rule 43. The only provision
affected by the Fabian ruling is the designation of the Court of Appeals as the proper forum and
of Rule 43 as the proper mode of appeal. All other matters in Section 27 of RA 6770, including
the finality or non-finality of decisions of the Ombudsman, remain valid.
Jurisprudence now holds that where the findings of the Ombudsman on the existence of probable
cause in criminal cases is tainted with grave abuse of discretion amounting to lack or excess of
jurisdiction, the aggrieved party may file a petition for certiorari with the Supreme Court under
Rule 65. Since Enemecio filed a certiorari petition before the Court of Appeals, instead of the
Supreme Court, she availed of a wrong remedy in the wrong forum. Hence, the instant petition
should be dismissed outright.

MANUEL BAVIERA, petitioner,


vs.
ROLANDO B. ZOLETA, in his capacity as Graft Investigation and Prosecution Officer II; MARY
SUSAN S. GUILLERMO, in her capacity as Director, Preliminary Investigation and Administrative
Adjudication Bureau-B; PELAGIO S. APOSTOL, in his capacity as Assistant Ombudsman, PAMO;
ORLANDO C. CASIMIRO, in his capacity as Assistant Ombudsman for the Military and Other
Law Enforcement Offices; and MA. MERCEDITAS N. GUTIERREZ (Then) Undersecretary,
Department of Justice, respondents.
[G.R. No. 169098, October 12, 2006, CALLEJO, SR., J.]

FACTS:
1. Manuel V. Baviera filed several complaints against officers or directors of the Standard Chartered
Bank (SCB), Philippine Branch, including Sridhar Raman, an Indian national who was the Chief
Finance Officer of the bank, as respondents with the Securities and Exchange Commission
(SEC), Bangko Sentral ng Pilipinas (BSP), Anti-Money Laundering Council (AMLC), National Labor
Relations Commission (NLRC), and the Department of Justice.
2. Baviera claimed that he was a former employee of the bank, and at the same time, an investor who
was victimized by the officers or directors of SCB, all of whom conspired with one another in
defrauding him as well as the investing public by soliciting funds in unregistered and unauthorized
foreign stocks and securities.
3. On September 18, 2003, Baviera, through counsel, requested the Secretary of Justice for the issuance
of a Hold Departure Order (HDO) against some of the officers and directors of SCB, including
Raman. On September 26, 2003, then Secretary of Justice Simeon Datumanong issued HDO No.
0193. A copy of the order was served on the Bureau of Immigration (BI) for implementation. On the
same day, the BI issued an Order implementing that of the Secretary of Justice. Meanwhile, Secretary
Datumanong went to Vienna, Austria, to attend a conference. Undersecretary Merceditas Navarro-
Gutierrez was designated as Acting Secretary of the DOJ.
4. On September 28, 2003, a Sunday, Raman arrived at the Ninoy Aquino International Airport (NAIA)
for his trip to Singapore but was apprehended by BI agents and NAIA officials. However, the next
day, September 29, 2003, Raman was able to leave the country via Singapore Airlines-SQ-71 at an
8:15 a.m. flight. He was to attend a conference in Singapore and to return to the Philippines on
October 2, 2003. Acting Secretary of Justice Merceditas N. Gutierrez had verbally allowed the
departure of Raman.
5. On the same day, Raman, through counsel, wrote Secretary Datumanong for the lifting of the HDO
insofar as his client was concerned. Acting Secretary Gutierrez issued an Order allowing Raman to
leave the country. In said Order, she stated that the Chief State Prosecutor had indicated that he
interposed no objection to the travel of Raman to Singapore.
6. On October 3, 2003, Baviera filed a Complaint-Affidavit with the Office of the Ombudsman
charging Undersecretary Ma. Merceditas N. Gutierrez for violation of Section 3(a), (e), and (j) of
Republic Act (RA) No. 3019, as amended. The complainant alleged that the actuations of respondent
Gutierrez were illegal, highly irregular and questionable for the following reasons:
a. DOJ Sec. Datumanong issued a Hold Departure Order (HDO) against three foreign
nationals, including Raman, on September 26, 2003;
b. Also on September 26, 2003, BID Commissioner Danilo Cueto issued the necessary order
and notification to all airports, seaports and exit points for the implementation of the
aforesaid HDO;
c. Raman went to the NAIA for departure out of the Philippines on Sunday, September 28,
2003;
d. Raman was stopped by Immigration officials from leaving the country on Sunday on the
strength of the HDO;
e. Usec. Gutierrez admitted having interceded on behalf of the Indian national, thus allowing
him to leave the country for Singapore at about 8:15 a.m. of Monday, September 29, 2003;
f. Obviously, the appeal of Raman to be allowed to leave the country was made verbally either
by him or thru counsel;
g. There is no written application for temporary stay of the HDO in respect to Raman’s
departure;
h. There is likewise no written order by Usec. Gutierrez allowing Raman to leave;
i. Usec. Gutierrez claims that she cleared the matter with DOJ Sec. Datumanong who was in
Vienna, Austria;
j. If she did so, then she could have made the consultation only either by telephone or e-mail
i. If she consulted Sec. Datumanong by telephone, then she must have gone out of
her way to go to the Department of Justice on a Sunday to use the DOJ
telephone;
ii. If she did not go to the DOJ on a Sunday, then she must have used her own
telephone and shouldered the expense to call Sec. Datumanong on behalf of her
beloved Indian national or the latter’s counsel;
iii. If she cleared the matter with Sec. Datumanong by e-mail, then the burden is on
her to prove that she did so by that means;
k. It is obvious that Usec. Gutierrez went out of her way to accommodate an Indian national or
the latter’s lawyer on a Sunday (verbally, secretly, and when nobody was looking) to allow
the Indian national to leave the country –despite an existing HDO- thus giving the Indian
national unwarranted, undue preference, benefit and advantage, to the damage and prejudice
of complainant.
l. There are indications that Usec. Gutierrez will also allow the other Indian national (Ajay
Kanwal) to leave for permanent posting outside the Philippines despite the existing HDO.
7. In her Counter-Affidavit, respondent Gutierrez averred that she did not violate any law or rule, in
allowing Raman to leave the country. She merely upheld his rights to travel as guaranteed under the
Constitution. Moreover, the DOJ may allow persons covered by HDOs to travel abroad, for a
specific purpose and for a specific period of time. She further averred that:
. I allowed Mr. Raman to leave the Philippines on September 29, 2003 in my capacity as Acting
Secretary, not as Undersecretary as alleged in the Complaint-Affidavit. An Acting Secretary has the power and
authority to perform all official acts that a Department Secretary, if personally present, could lawfully do and
to exercise sound discretion under certain circumstances. In the case of an Acting Secretary of Justice, the
authority extends to allowing the travel of a person subject of an HDO, like Mr. Raman, whose attendance in
an official business abroad was urgent and necessary. Although I could have lifted the HDO on the ground
that there was no ground for its continued enforcement, I did not do so in deference to the Secretary who
issued it but, instead, allowed Mr. Raman to travel for a specific purpose and period. Secretary Datumanong
eventually lifted the HDO and, therefore, ratified my act.
a. An individual subject of an HDO issued by the Department may be allowed to travel abroad. Even
the court that issued an HDO may authorize the subject person to travel for a specific purpose and for a
certain period. If the person already charged in court may be authorized to travel, there is more reason to
allow the person, like Mr. Raman, who was still subject of a preliminary investigation by a prosecutor, to
travel abroad. He continues to enjoy the constitutional presumption of innocence. Thus, his rights under the
law should not be unreasonably curtailed.
b. I allowed Mr. Raman to travel to Singapore because he, as Chief Finance Officer of Standard
Chartered Bank (an international bank with good reputation), was invited and required to attend the
Wholesale Bank International Accounting Standards Conference from September 29 to October 2, 2003. The
travel was not meant to have him transferred to another branch of the bank abroad and frustrate the results
of the investigations, which were the cited reasons for the HDO application. Indeed, he returned to the
Philippines on October 2, 2003.
c. Allowing Mr. Raman to travel abroad under the circumstances would send a positive message to
foreigners engaged in banking and business activities in the Philippines that the Government consistently
upholds the rule of law and respects human rights, thereby boosting investors’ confidence in the Philippines.
d. In allowing Mr. Raman to travel abroad, I relied on my oath as a lawyer and as a government official
to support and defend the Constitution. I also relied on the first Whereas Clause of the above-mentioned
Department Circular No. 17 dated March 19, 1998, which cites Section 6, Article III of the present
Constitution that, in part, reads: "xxx Neither shall the right to travel be impaired except in the interest of
national security, public safety, or public health, as may be provided by law." Relevantly, in Kant Kwong v.
Presidential Commission on Good Government, the Supreme Court En Banc held: “xxx. The right to travel and to
freedom of movement is a fundamental right guaranteed by the 1987 Constitution and the Universal
Declaration of Human Rights to which the Philippines is a signatory. The right extends to all residents
regardless of nationality. And "everyone has the right to an effective remedy by the competent national
tribunals for acts violating the fundamental rights granted him by the Constitution or by law."

8. Respondent Gutierrez requested the Office of the Ombudsman to dismiss the complaint against her,
thus:
a. There is no basis for the complaint for violation of Section 3(a) of RA No. 3019, as amended,
because I never persuaded, induced nor influence any public officer to violate the rules and regulations duly
promulgated by competent authority.
b. The complaint for violation of Section 3(e) of RA No. 3019 is baseless. The complainant has not
sustained any injury by reason of the travel order, as Mr. Raman immediately returned to the Philippines after
his official business. I authorized Mr. Raman to travel in recognition of his right thereto under the
Constitution and existing international human rights law instruments. In so doing, I did not give him
unwarranted benefit, advantage or preference in the discharge of my official functions through manifest
partiality, evident bad faith or gross inexcusable negligence. Indeed, had I denied him the right, I would be
held liable under such provision, in addition to other liabilities under the Civil Code.
c. Neither is there any basis for the complaint for violation of Section 3(j) of RA No. 3019, as amended.
I permitted Mr. Raman to leave the country on September 29, 2003 because he had an important official
business abroad and he was legally entitled to the right to travel and the grounds mentioned in the
Constitution for the impairment of the right did not exist.

9. In his Reply-Affidavit, Baviera alleged that:


a. The main issue against her is NOT an individual’s constitutional right to travel nor the wide
discretionary powers of the DOJ to grant special permits to travel to individuals subject of HDO BUT her
abuse of such discretionary powers.
b. When she allowed the Indian National to leave the country on a mere verbal plea by Raman or his
well-connected lawyer on a Sunday and without a proper Motion for Reconsideration yet being filed by
Raman or his lawyer, she undoubtedly gave the latter unwarranted benefit, advantage or preference in the
discharge of her official duty as Acting Secretary. The undisputable fact, which respondent herself admitted
proudly, was both plea and the Order were done verbally.
c. It was only much later that her Order dated 29 September 2003 was belatedly released long after
Raman had left the country on an early morning flight to Singapore. Respondent went out of her way to
accommodate a foreign national by hurriedly allowing the latter to leave without going through proper
procedures. Paragraph V of DOJ Circular No. 17 provides the following procedure in appealing or lifting an
HDO, to wit: "A copy of the HDO implemented by the Commissioner shall be sent to the person subject of
the order, if his postal address is known, so that he may, if he so desires, file a MOTION FOR
RECONSIDERATION with the Secretary. (Underscoring supplied).
d. Respondent tried to justify her indiscretion by attaching as Annex "4" of her Counter-Affidavit a
letter from Raman’s lawyer dated 29 September 2003 requesting that Raman be allowed to travel.
Conspicuously, the letter was stamped received by respondent’s office and allegedly signed and received by
her staff on Monday, 29 September 2003 at 6:15 a.m. Obviously, respondent is trying to cover up her actions,
albeit to no avail. Who could possibly believe that respondent’s office would be open at 6:15 in the morning
of a Monday when the normal office hours is at 8 a.m.? Worse, assuming arguendo that the letter-request was
received at 6 a.m., how come Raman was able to board Singapore Airlines Flight No. SQ-71 which left at
about 8:15 a.m. or barely two (2) hours upon the receipt of the request?
e. Res Ipsa Loquitor. It is either respondent Gutierrez, Secretary Datumanong or the Chief State
Prosecutor (whom she claimed to have consulted before giving the order) reports to their offices at 6 a.m.
and buckle down to work immediately or that respondent Gutierrez’s allegations in her defense are all
concocted lies.
f. The belated documentation of respondent’s action was further proven by records showing that the
Motion for Reconsideration and the Supplement thereof were dated 5 October and 7 October 2003,
respectively, or six (6) days after Raman was allowed by respondent to leave the country.
g. Respondent cannot deny the fact that she admitted in a hurriedly-called press conference later on 29
September 2003 before the DOJ press that she was the one who verbally gave instructions to immigration
and NAIA officials to allow Raman to leave the country.
h. Even her claims that she has not benefited from her actions cannot be made as a defense because the
provisions of the Anti-Graft law charged against her do not require as a pre-condition that the public officer
receive (sic) any gift, present, or benefit.
i. Her decision to grant special permission to Raman (which she proudly admits) is irregular and illegal
because there is no specific law or rules of the DOJ granting special permission or exception to the HDO.

10. On October 17, 2003, Justice Secretary Simeon Datumanong issued an Order lifting the HDO and
ordered the BI to delete the names of the officials of the bank, including Raman, from its Watchlist.
11. On June 22, 2004, Graft Investigation and Prosecutor Officer Rolando Zoleta signed a Resolution
recommending that the criminal complaint against respondent Gutierrez for violation of RA No. 3019 be
dismissed for insufficiency of evidence. Zoleta’s findings are as follows:
a. With respect to the charge of violation of Section 3(a) of Republic Act 3019, there is no evidence,
documentary or testimonial, to show that respondent GUTIERREZ has received material remuneration as a
consideration for her alleged use of influence on her decision to allow Mr. RAMAN to travel abroad.
b. As to the charge of violation of Section 3(e) of Republic Act 3019, no actual or real damage was
suffered by any party, including the government as Mr. Raman immediately returned to the Philippines, the
truth of which was not rebutted by the herein complainant in his Reply-Affidavit. Thus, the herein
complainant also did not suffer undue injury as an element required by the law. By the same token, the
essential ingredient of manifest partiality, evident bad faith or gross inexcusable negligence required for the
commission of such offense has not been proven in the instant case.
c. With regard to the charge of Violation of Section 3(j) of Republic Act 3019, as above discussed, the
respondent, as Acting Secretary of Justice, is authorized or empowered not only to allow the travel abroad of
Mr. RAMAN under specific conditions but also to order the lifting of such Hold Departure Order.

12. On November 16, 2004, Baviera filed a petition for certiorari under Rule 65 of the Rules of Civil
Procedure in the CA, assailing the resolutions of the Ombudsman.
a. However, on January 7, 2005, the CA issued a Resolution dismissing the petition on the ground that
the proper remedy was to file a petition for certiorari with the Supreme Court under Rule 65 of the Rules of
Court.
b. Petitioner filed a motion for reconsideration, insisting that his petition for certiorari in the CA under
Rule 65 was in accordance with the ruling in Fabian v. Desierto. He insisted that the Office of the Ombudsman
is a quasi-judicial agency of the government, and under Batas Pambansa Bilang 129, the CA has concurrent
jurisdiction with the Supreme Court over a petition for certiorari under Rule 65 of the Rules of Court.
c. On July 20, 2005, CA issued a Resolution denying the motion, holding that the ruling in Fabian v.
Desierto is not applicable, as it applies only in appeals from resolutions of the Ombudsman in administrative
disciplinary cases.

ISSUE: W/N the petition for certiorari filed by petitioner in the CA was the proper remedy to assail
the resolution of the Office of the Ombudsman

RULING: No, petitions assailing resolutions of the Ombudsman in criminal or non-administrative


cases shall be filed before the Supreme Court.
1. In Fabian vs. Desierto, the appellate court correctly ruled that its jurisdiction extends only to decisions
of the Office of the Ombudsman in administrative cases. We ruled that appeals from decisions of the
Office of the Ombudsman in administrative disciplinary cases should be taken to the Court of Appeals
under Rule 43 of the 1997 Rules of Civil Procedure.
2. The remedy of aggrieved parties from resolutions of the Office of the Ombudsman finding probable
cause in criminal cases or non-administrative cases, when tainted with grave abuse of discretion, is to
file an original action for certiorari with this Court (SC) and not with the Court of Appeals.
3. On the merits of the petition, the Court finds that petitioner failed to establish that the respondent
officials committed grave abuse of discretion amounting to excess or lack of jurisdiction. The Court
has reviewed the assailed resolutions of the Office of the Ombudsman, and finds that petitioner
likewise failed to establish probable cause for violation of Sections 3(a), (e) and (j) of RA No. 3019.

A.M. No. MTJ-05-1581 February 28, 2005


PETER L. Sesbreño, Sesbreño,vs.JUDGE GLORIA B. AGLUGUB, Metropolitan Trial
Court, Branch 2, San Pedro, Laguna, Judge Aglugub.

FACTS
Peter L. Sesbreño filed 3 separate complaints against Enrique Marcelino, Susan Nuñez,
Edna Tabazon and Fely Carunungan, all from the Traffic Management Unit of San Pedro,
Laguna, for Falsification, Grave Threats and Usurpation of Authority. The 3 cases were
assigned to judge Aglugub. After conducting a preliminary examination, Judge dismissing
the cases for Falsification and Grave Threats for lack of probable cause, and setting for
arraignment the case for Usurpation of Authority. Subsequently, Sesbreño filed an Urgent
Manifestation alleging that the accused were also charged with violation of (R.A. 10) and
praying that warrants of arrest be likewise issued against all of the accused.
Acting upon this manifestation, Judge Aglugub issued an Order stating that a
charge for violation of R.A. 10 was indeed alleged in the complaint for Usurpation of
Authority but was not resolved due to oversight. However, since the statute only applies
to members of seditious organizations engaged in subversive activities, Judge Aglugub
judge found no probable cause and dismissed the charge for violation of R.A. 10 and
denied prayer for the issuance of warrants of arrest against the accused and ordered the
records forwarded to the Provincial Prosecutor’s Office (PPO) for review.1a\^/phi1.n
etSesbreño’s counsel, Atty. Raul Sesbreño (Atty. Sesbreño), filed a Motion for
Reconsideration and Urgent Ex-Parte Motion for Issuance of Warrant of Arrest Against
Non-Appearing Accused. Judge Aglugub, however, did not act on these motions because
the court had already lost jurisdiction over the case by then. The PPO affirmed Judge
Aglugub’s order and remanded the case to the court for further proceedings on the charge
of Usurpation of Authority.
The foregoing circumstances brought about the filing of the instant administrative
complaint.
Sesbreño contends that Judge Aglugub judge violated Sec. 6(b), Rule 112 of the Rules
when she refused to issue warrants of arrest against the accused. Sesbreño also faults
Judge Aglugub judge for allegedly motu proprio reconsidering her Consolidated
Resolution dated May 6, 2003 and failing to order its transmittal to the Office of the
Ombudsman within ten (10) days.
ISSUE
whether Judge Aglugub should have transmitted her Order dismissing the charge of
violation of R.A. 10 to the Office of the Ombudsman instead of the PPO.
RULING (DISMISSED)
NO, This issue is answered by Administrative Order No. 8 entitled Clarifying and
Modifying Certain Rules of Procedure of the Ombudsman, which provides "that all
prosecutors are now deputized Ombudsman prosecutors." Moreover, "Resolutions in
Ombudsman cases against public officers and employees prepared by a deputized
assistant prosecutor shall be submitted to the Provincial or City Prosecutor concerned
who shall, in turn, forward the same to the Deputy Ombudsman of the area with his
recommendation for the approval or disapproval thereof. The Deputy Ombudsman shall
take appropriate final action thereon, including the approval of its filing in the proper
regular court or the dismissal of the complaint, if the crime charged is punishable by
prision correccional or lower, or fine of not more than ₱6,000.00 or both. Resolutions
involving offenses falling within the jurisdiction of the Sandiganbayan shall be forwarded
by the Deputy Ombudsman with his recommendation thereon to the Office of the
Ombudsman."
Thus, Judge Aglugub judge did not err and was, in fact, merely acting in accordance with
law when she forwarded the case for violation of R.A. 10 to the PPO. The fact that the
PPO remanded the case to the court for further proceedings instead of forwarding the
same to the Deputy Ombudsman as required by Administrative Order No. 8 is quite
another matter. In any event, Judge Aglugub should have taken the necessary steps to
remedy the lapse in order to preclude delay in the disposition of the case.
Judge Aglugub Judge Gloria B. Aglugub is ADMONISHED to be more circumspect in the
performance of her duties in the future.
ENRIQUEZ vs. CAMINADE

IMELDA S. ENRIQUEZ, Complainant,


vs. JUDGE ANACLETO L. CAMINADE, Respondent
(A.M. No. RTJ-05-1966 | March 21, 2006)

FACTS: Imelda S. Enriquez (mother of the murder victim) filed a verified complaint with the Office
of the Court Administrator (OCA) charging Judge Anacleto Caminade with Gross Misconduct,
Knowingly Rendering an Unjust Judgment and Gross Ignorance of the Law and Procedure
relative the Criminal Case entitled ‘People of the Philippines versus Sherwin Que Bungol,
Anthony John Apura,’ for Murder.
There was no preliminary investigation completed on accused Alvin Taggart Pimentel Alvez
and Alvin John Apura [as] they were denied the opportunity to file a motion for
reconsideration or a petition for review before the information was filed in court. Because of
this, Judge Caminade issued an order on March 31, 2004 which:
1. denies the motion for the issuance of the warrant of arrest against the accused-movants;
2. sets aside the assailed Resolution of the City Prosecutor on the basis of which the latest
amended information was filed;
3. quashes the latest amended information; and
4. remands this case to the City Prosecutor for completion of the preliminary investigation.’
Due to respondent’s ruling the investigating prosecutor cannot file a criminal information before
the expiration of the 15-day period within which the accused are allowed by the Revised Rules of
Court to move for reconsideration or petition for review of an adverse ‘Resolution.’
Respondent cited Sales vs Sandiganbayan where it was held that:

1. ‘the filing of motion for reconsideration is an integral part of the preliminary investigation
proper’
2. information ‘filed without first affording x x x accused his right to file motion for
reconsideration’ is tantamount to a denial of the right itself to a preliminary investigation.

ISSUE: WON the Sales vs Sandiganbayan ruling is applicable to the case at bar

HELD: NO, the Sales vs Sandiganbayan is not applicable to the criminal case because of
significant factual and procedural distinctions between the two cases:
(1) the Sales case proceeded under the Rules of Procedure of the Ombudsman, while subject
criminal case was conducted under the Rules of Court;
(2) there was no completed preliminary investigation in the Sales case but there was a
completed full-blown panel preliminary investigation on the accused in the subject
criminal case; and
(3) it is only under the Rules of Procedure of the Ombudsman that the preliminary investigation is
deemed completed and terminated upon the lapse of the period to file a motion for reconsideration
from the resolution of the Ombudsman while there is nothing in the Rules of Court which
states that a person investigated has the right to file a motion for reconsideration or
reinvestigation before the information can be filed in court.
WHEREFORE, Judge Anacleto L. Caminade is found guilty of gross ignorance of the law, for
which he is FINED in the amount of twenty thousand pesos (P20,000). He is STERNLY WARNED
that a repetition of the same or similar acts shall be dealt with more severely in the future.
G.R. No. 171175 October 30, 2009
PEOPLE OF THE PHILIPPINES, Petitioner,
vs.
ARTURO F. DUCA, Respondent.

FACTS:
• Respondent Arturo Duca, together with his mother(Cecilia), were charged of the crime of
Falsification of Official Document punished under the Secs 171 and 172 of the Revised
Penal Code. It was alleged that they falsified a declaration of Real Property over a
bungalow residential house in Pangasinan by making it appear that the alleged owner
(Aldrin Duca – brother of Arturo) affixed his signature thereon when in fact he could not
have done so since he was abroad at the time the document was made
• Calayanan having been injured by the execution of such document, filed an ejectment
suit which was granted by the MCTC. Cecilia Duca filed a declaration of nullity of such.
When the case was heard, Arturo admitted that he was the one who signed the document
interposing that he was duly authorized by his brother Aldrin to procure tax declaration.
• MCTC found Duca guilty which was affirmed by RTC. Dissatisfied with the ruling, Duca
Elevated the case to CA. CA acquitted Duca giving credence to his testimony.
• Hence, this instant petition for certiorari. Petitioner argues that the prosecution was denied
due process when the CA resolved the respondent’s appeal without notifying the People
of the Philippines, through the Solicitor General, of the pendency of the same and without
requiring the Solicitor General to file his comment. Petitioner contends that once the case
is elevated to the CA or this Court, it is only the Solicitor General who is authorized to bring
or defend actions on behalf of the People.
ISSUE:
Whether or not CA gravely abused its discretion and had acted without jurisdiction when it
resolved private respondent-Duca’s appeal without giving the People of the Philippines through
the Office of the Solicitor General the opportunity to be heard thereon.
RULING:
• YES. The authority to represent the State in appeals of criminal cases before the CA and
the Supreme Court is solely vested in the Office of the Solicitor General under the 1987
Administrative Code.
• Indeed, in criminal cases, as in the instant case, the Solicitor General is regarded as the
appellate counsel of the People of the Philippines and as such, should have been given
the opportunity to be heard on behalf of the People. The records show that the CA failed
to require the Solicitor General to file his Comment on Duca’s petition. It was not shown
that the Solicitor General was furnished with a copy of the CA resolution. The failure of
the CA to require the Solicitor General to file his Comment deprived the prosecution of a
fair opportunity to prosecute and prove its case.
• The respondent was mandated under Section 1, Rule 42 of the Rules of Court to serve
copies of his petition for review upon the adverse party, in this case, the People of the
Philippines through the OSG. Respondent failed to serve a copy of his petition on the OSG
and instead served a copy upon the Assistant City Prosecutor of Dagupan. The
respondent’s failure to have a copy of his petition served on the People of the Philippines,
through the OSG, is a sufficient ground for the dismissal of the petition as provided in
Section 3, Rule 42 of the Rules of Court. Thus, the CA has no other recourse but to dismiss
the petition. However, the CA, instead of dismissing respondent’s petition, proceeded to
resolve the petition and even acquitted respondent without the Solicitor General’s
comment. We, thus, find that the CA committed grave abuse of discretion amounting to
lack or excess of jurisdiction in rendering its assailed decision. The assailed decision of
the CA acquitting the respondent without giving the Solicitor General the chance to file his
comment on the petition for review clearly deprived the State of its right to refute the
material allegations of the said petition filed before the CA. The said decision is, therefore,
a nullity.
DP: Wherefore, petition for certiorari is hereby GRANTED. The assailed decision of the CA is
hereby SET ASIDE and the case is REMANDED to the CA for further proceedings.

RP VS IYOY
(UNAVAILABLE)

Você também pode gostar